Sunteți pe pagina 1din 198

ACADEMIA DE STUD IT ECONOMICE

LIANA MANU-IOSIFESCU SORINBAZ


BOGDAN IFTIMIE
IIAllzlMATEncA
CULEIERE IE PRIILEME
PENTRU ANUL I
Coordonator: CONSTANTIN RAiscm
EditUraASE
Bucurefti
2000
ACADEMIA DE STUDII ECONOMICE
FACULTATEA DE CIBERNETICA, STATISTICA
INFORMATICA ECONOMICA
CATEDRA DE MATEMATICA
Lector.dr. Lector dr. Lector.dr.
LIANA MANU-IOSIFESCU SORINBAZ BOGDAN IFTIMIE
ANAlizA MATEMATICA
CUlEGERE DE PROBlEME
PENTRU ANUL I
Coordonator: Conf.dr. CONSTANTIN RAISCHI
Editura ASE

2000
ISBN 973 - 9462 - 55 - 3
Cuprins
Pag.
CAPITOLUL 1 ELEMENTE DE TEORIA MUL TIMILOR
TOPOLOGIE................ ............ .... .. .. 5
A. Probleme rezolvate.............. .. . . . . . . . .. 7
B. Probleme propuse........................ .. 30
CAPITOLUL 2 COMPLEMENTE DE TEORIA
A SERIILOR NUMERICE 43
A. Probleme rezolvate.. .. . .. . . . . .. . . . . .. . ... . . 45
B. Probleme propuse................ ....... ... 72
CAPITOLUL 3 CONVERGENT A SIMPLA,
CONVERGENT A UNIFORMA
A DE FUNCTII..... ... .. .......... 85
3.1. de functii.. .. . ... . . . . . . . . . .. . .. . . . . ... . . . . ... 87
A. Probleme rezolvate..... .. . .. . .. . . . . ..... .... 87
B. Probleme propuse........ ... . . . . . . .. . . . . ... 95
3.2. Serii de functii. Serii de puteri.... ........... 98
A. Probleme rezolvate.. ... .. ....... .. ... . .. . .. 98
B. Probleme propuse.......................... 106
3.3. Serii Taylor......................................... 108
A. Probleme rezolvate......................... 1 08
B. Probleme prop use.......................... 116
CAPITOLUL 4 FUNCTII DE MAl MUL TE VARIABILE...... 119
4.1. Domenii de definitie. Limite.
Continuitate . . . . . .. . .. . .. . .. . . . . .. . . . . . . . .. . .. . . . . . 121
A. Probleme rezolvate......................... 121
B. Probleme propuse.......................... 128
4.2. Derivate partiale. Diferentiabilitatea
functiilor de mai multe variabile.
Derivate de ordin superior. Formula lui
Taylor pentru functii de mai multe
variabile.. .. . .. . . . . . . . . . . .. . .. . .. . . . . . . . .. . . . . . . . .. . . 131
3
A. Probleme rezolvate......................... 131
B. Probleme propuse.. ... .. .... ...... ..... .... 147
4.3. Extremele func1iilor de mai multe
variabile. Extreme cu legaturi............... 152
A.1. Probleme rezolvate.. .. . .. . .. . .. . .. . .. . ... 152
A.2. Aplica1ii in economie..................... 166
B. Probleme propuse........................... 174
4.4. Func1ii implicit definite. Transformiri
regulate. Dependen18 func1ionala.......... 178
A. Probleme rezolvate....................... .. 178
B. Probleme prop use.......................... 194
4
131
147
152
152
166
174
178
178
194
ELEMENTE DE TEORIA MULTIMILOR
$1 TOPOLOGIE
Autor: lector.dr. LIANA MANU-IOSIFECU
8
ELEMENTE DE TEORIA MUL TIMILOR
1 TOPOLOGIE
c...__-----
A. Problema rezolvate
1. Sa se verifice care dintre urmatoarele relatii in R
2
sunt
rela\ii de ordine:
a) (x,y) ID
1
(x' ,y') <:::> x s x' y s y' (ordinea prod us)
b) (x,y) ID
2
(x' ,y') <=> x s x' sau (x = x' y s y') (ordinea
lexicografica)
c) (x,y) ID3 (x' ,y') <=> x
2
+I s x'
2
+ y'
2
d) (x, y) ID4 (x', y') <=> I x I + I y I s I x' I + I y' I
Care este ordine totala?
Care sunt ordinile ce le indue pe multimea
A = { (x, y) I y= 0}?
\ Solutie \
a) Fie (x,y) E R
2
::? (x,y} ID1 (x,y) deoarece x s x, y s y.
Oeci v (x,y) E R
2
, (x,y) ID1 (x,y). Rezulta ca relatia ID
1
este
reflexiva. In mod se arata ca relatiile ID2, ID3, ID4 sunt
reflexive.
Fie (x,y), (x',y'), (x",y") E R
2
(x,y) ID1 (x',y'), (x',y') ID1
(x",y").
Rezulta x s x' y s y', x' s x" y' s y", deci x s x" y s y".
Astfel (x,y) ID
1
(x",y"). Deci V (x,y), (x',y'), (x",y") E R
2
a.i.
(x,y) ID1 (x' ,y'), (x' ,y') ID1 (x" ,y") ::? (x,y) ID1 (x" ,y").
Rezulta ca relatia este tranzitiva. In mod se arata ca
ro
2
, ro3, ID4 sunt relatii tranzitive.
Fie (x,y), (x',y'), E R
2
'a.i. (x,y) ID1 (x',y'), (x',y') ID1 (x,y)::?
::?X s x', y s y' x' s x, y' s y. Deci (x,y) = (x',y') relatia ID1
este antisimetrica. In concluzie, ID1 este o relatie de ordine pe R
2
in mod analog, se arata ca relatia IDz este relatie de ordine pe R
2
.
7
Relatia w3 nu este antisimetrica, deoarece 3 (x,y), (x',y') E R
2
,
(x,y) w3 (x',y') l}i (x',y') w3 (x,y) astfel lnd'it (x,y) * (x',y'). De
exemplu: (x,y) = (1 ,0) l}i (x',y') = (0, 1 ).
Relatia w, determina pe R
2
o relatie de ordine partiala.
Relatia w2 este o relatie de ordine totala pe R
2
:V(x,y), (x',y')E R
2
,
(x,y) w2 (x',y') sau (x',y') w2 (x,y). intr-adevar, fie (x,y) l}i {x',y') e
R
2
. Avem x <X' sau x >X' sau x = x'.
in primele doua cazuri este lndeplinita conditia, iar In ultimul
caz, y :S y' sau y' :S y, deci (x,y) w2 (x',y') sau {x',y') w
2
(x',y').
2. Fie X = {{1}, {1,2}, {2,3,4}, {5}} ordonata prin
incluziune:
a) Sa se determine elementele maximale l}i elementele
minimale.
b) Exista un eel mai mare element?
I Solutie I
a) Se verifica imediat ca {1,2}, {2,3,4} l}i {5} sunt
maximale, iar { 1}, {2,3,4} l}i { 5} sunt minimale. Remarcam
{2,3,4} l}i {5} sunt l}i elemente maximale l}i elemente minimale
{X, c).
b) Daca A c X ar fi eel mai mare element al multimii
ordonata prin incluziune, atunci ar trebui sa avem { 1 ,2,3,4,5} cA.
insa nici un element din X nu are aceasta proprietate.
3. Fie M o multime arbitrara. Sa se arate ca:
a) ('P(M), c) este o structura de ordine paftiala In care <I>
este primul element (eel mai mic), iar M eel mai mare element. (Un
element m EM se numel}te primul element al multimii M daca,
oricare ar fi m' E M, avem m :S m').
b) Oricare ar fi A; E 'P (M), i E I, avem: sup{Ai};EI = UA;
;"
= nA; .
;"
8
a) Din
Olbre ar fi A-
,...xivl,
Mlndu-se fn
81:11 ient sunt
, ln;; . b) Fifll 811

'" d b)

c)
,G
I Solutie I
a) Din proprietatile relatiei de incluziune din <t> c A c M
oricare ar fi A e 'P (M), rezulta i). Evident, relatia de incluziune este
reflexiva, antisimetrica tranzitiva, nu orice doua submultimi
afiAndu-se in relatie de incluziune. Gel mai mic eel mai mare
element sunt elemente extremale.
b) Fie 8 = UA,. Din Ai c 8 Viel, rezulta ca 8 este un
;,.1
majorant al familiei {Ai} in ('P (M), c). Fie C un alt majorant, adica
A; c C, iel. De aici, rezulta ca 8 = UA, c c, adica 8 este eel mai
ieA;
mic majorant. Deci 8 = sup{Ai};EI Analog se arata cealalta
egalitate.
4. Fie x co y <::::> x I y, relatia de divizibilitate pe N* (xI y <::::>
3KeN, x = ky) o submultime a lui N*, X = {3,5,15,
2,4,8, 16,80,195}
a) Sa se arate ca relatia definita este o relatie de ordine
partiala pe N*
b) Sa se cerceteze existenta elementului minim a
elementului maxim pentru multimea X.
c) Sa se determine elementele minimale maximale ale
lui X, infX supX sa se arate ca X este inductiv
ordonata.
I Solutie I
a) Fie xeN. Cum xI x, relatia este reflexiva. Antisimetria
tranzitivitatea se deduc folosind definitia relatiei co.
b) Luand pe rand elementele multimii X, cum VxeX 3yeX
a.i. x{y, multimea nu are element minim (2 nu este minim, intrucat
2{3). In mod se arata ca X nu are element maxim.
c) Fie xeX cu 1951 x. Rezulta x = 195 , deci 195 este un
element maximal. Similar 80 este un element maximal al multimii
X. Multimea elementelor minimale este {2,3,5}.
9
Multimea minorantilor multimi X este { 1}, deci infX == 1, fiind
eel mai mare minorant (infX este eel rna; mare divizor comun al
elementelor multimii X). Multimea majorantilor multimii X este
multimea multiplifor comun; din X, {2
4
3513K I keN*}, dec; supX
== 2
4
3513, adica eel rna; mic multiplu comun al elementelor
mulfimii X. X este inductiv Ordonata: orice parte a sa total ordonata
are margine superioara, care este eel rna; mare element din
respectiva submultime fn sensu/ ordinii naturale. Acest element,
tinand seama de conditia de ordonare totala, va fi divizibil cu orice
aft element. Conform feme; lui Zorn, multimea X are eel pufin un
element maximal (cardinalul multimii elementelor maximale este 2)
reate:
5. Studiati marginirea urmatoarelor multimi de numere
A== (-co, 4)
8 =: [-1' 2)
C == { n" I neN}
[solufie]
Multimea A este majorata deoarece exista beR astfel fncat
oricare a eA, a:s;b (evident orice b;:-:4 satisface aceasta). Marginea
superioara a multimii A este 4 (4 este majorant). Fie s > 0. Luand
2
x. == 4-.!:. eA, acesta satisface conditia 4-s :;; x. :;; 4. Deci, \is > 0,
3 x. eA astfel fncat 4 - s :;; x:;; 4. Prin urmare supA == 4. Multimea A
nu este minorata !li nu are margine inferioara finita, fntrucat
\iCeR, 3ac eA astfel fncat ac <C. fntr-adevar, fie CeR. Exista
ac == C- 1 e A== (-co ,4) !ji ac <C. A vern infA ==-co.
Multimea B este fnsa marginita. Exista M == 3 astfel fncat
\ixeB, I x):;; 3 deoarece [-1 .2) c [-3,3]. Marginea inferioara a
multimii 8 este -1, iar cea superioara este 2. fntr-adevar, -1 este
minorant !ji, pentru s >0, exista xs == -1 + .!:. eB, cu -1 :;; x.:;; -1+s.
2
Deci infB == -1 eB, !li analog supS== 2, dar sup 8 10 B.
10
Mulfimea C
asttet rncat IXM/ > M.
2[M] + 2eC.
A vern IXMI >
6. Fie Z "ou,,,.,
pozifiv. Fie pe Z relafia .
a) sa se arafiii!G
b) ScJ se deteflll
c) Sc'i Seafle
[ Soluflel
a) Fie PeZ. 1-\ve,.
dec; relafia OJ este renex!J
Fie p, qez ~ i
p (mOd m) ~ QOJp,
Fie p,q,reZ, PEq
3k,, k2 eZ asttetrncat
Prin urmare, 3k
~ mlp - r ~ p = r (mOd
i&lafia definite'! este o
b) Glasa de ect:r1Val8
fJ= {S/sez, miP-s} =
c) Rezultc'! ca exisUI
6 = {mklkeZ}, f =
1/KeZ}. Mulfimea cat
Observa(ie. Daca
echivalenta ale cor1gnuen1fe
(indicatorullui Euler).
7. Fie pe R relafia OJ
a) Sa se arate ca
b) Sa se determine
i n c ~ t
Mul(imea C nu este marginiti'i, deoarece, VM>O, 3 XMEC
astfel inca! IXMI > M. fntr-adevar, fie M >0; luam xM = (2[MJ + 2) ,_,,,M,., =
=2[M] + 2EC.
Avem IXM/ > M l?i infC = 0, iar supC = oo.
6. Fie Z mul(imea numerelor fntregi l?i m un numar fntreg
pozitiv. Fie pe Z rela(ia ro definita prin proq <=> p=q (mod m)
a) Sa se arata ca ro este rela(ie de echivalenta pe Z
b) Sa se determine clasele de echivalenta
c) Sa se afle numarul claselor de echivalenta
J Solutie /
a) Fie PEZ. Avem prop <=> P=P (mod m) <=> mJp-p <=> mJO,
deci rela(ia ro este reflexive.
Fie p, QEZ l?i proq <=> p=q (mod m) ~ mJp-q ~ mJq-p => q =
p (mod m) ~ qrop, deci rela(ia ro este simetrica.
Fie p,q,rEZ, p"'q (mod m), q = r (mod m) ::::> mJp-q l?i mJq-r =>
3k,, k2 EZ astfel fncat p-q = mk, ~ ? i q-r = mk2.
Prin urmare, 3k = k1+k2 EZ astfelicat p- q = m(k,+k
2
) = mk
3
::) mJp - r ~ p = r (mod m), deci. rela(ia este tranzitiva. Rezulta ca
rela(ia definita este 0 rela(ie de echivalen(a.
b) Glasa de echivalenta a lui PEZ este:
fi= (S/SEZ, mJp-s} = {p+mkJkEZ} = mZ + p
c) Rezulta ca exista m clase de echivalenta
0 = {mkJkEZ}, l= {mk+1JkEZ}', ... , m-l = {mk + m-
A A A A ...............
1/KEZ). Mui(Imea cat este: { o, 1 , 2, ... , m - 1} = Z/ro.
Observa(ie. Daca m nu este prim, atunci clasele de
echivalenta ale congruen(ei modulo m vor fi fn numar de <p{m)
(indicatorullui Euler).
7. Fie pe R rela(ia ro definita prin xroy <=> sin
2
x + cos2y = 1.
a) Sa se arate ca ro.este o rela(ie de echivalen(a
b) Sa se determine mul(imea cat, R/ro.
II
I Solutie I
a) Fie xeR; avem sin
2
x + cos
2
y = 1 ==:- x m x, lfxeR, deci
relatia m este reflexiva.
Fie x,yeR; avem xmy = sin
2
x + cos
2
y = 1=:-
1-cos2x + 1-sin
2
y = 1 ==:- sin
2
y + cos
2
x = 1, adica ymx, deci
relatia m este simetrica.
Fie x,y,zeR astfel incat xmy, ymz ::> sin
2
x + cos
2
y = 1,
sin
2
y + cos
2
z = 1. Prin adunarea membru cu membru obtinem
sin
2
x + cos
2
y + sin
2
y + cos
2
z = 2 ::> sin
2
x + cos
2
z = 1 ==:- xmz.' deci
m este tranzitiva. A!iadar, relatia m este o relatie de echivalen(a.
b) Fie xeR. Glasa de echivalenta a lui x este:
x = {y!ymx} = {y!sin
2
y + cos
2
x = 1} = {y!sin
2
y = sin
2
x} =
{kn xlkeZ}.
Rezulta ca lfxeR, 3ye(O, 2:.], astfel incat ymx.
2
Multimea cat este R/m = { x !xe[O, 2:.]}.
2
8. Fie f: X ~ Y o functie. Atunci avem:
a) f injectiva => lfA1, A2 c;;; X cu f(A1) c f(A
2
) rezulta A
1
c;;;
A2
b) f surjectiva => lf81, 82 c;;; Y cu f
1
(8
1
) c;;; f
1
(8
2
) rezulta
81c;;;8z.
I Solutie I
a) "==:-" Fie A1,A2 c;;; X fixate !?i f(A1) c f(A2). Atunci lfx
1
eA
1
avem f(A1) c f(A2), deci 3 x2eA2 cu f(x1) = f(x
2
). Cum f este
injectiva, rezulta x1 = X2. Prin urmare A1 c;;; Az.
"<="Fie x1,X2eX cu f(x1) = f(x2). Atunci f({x1}) c;;; f({x
2
}), deci
{x1} c;;; {x2} !li deci x1 = x2, ceea ce inseamna ca f este injectiva.
b) "==:-" Fie 81,82 c;;; Y fixate cu r-
1
(81) c f
1
(8
2
). Atunci
lfy1 e 81. :Jx1eX cu f(x1) = Y1
Cum f
1
({Y1}) c;;; r\81), rezulta ca x1 = r-
1
(81) c;;; f\8
2
) ~ i
deci Y1 = f(x1)e82.
12
Prin urmare, B1c;;;E
. "<=" Fie yeY f i x ~
lfy
1
ef(X), avem 0 = f
1
({]
Rezult!i f
1
({y}) "* 0, <:eel
Rezult!i ca, lfyeX
{x} "* 0, adica f este s u ~
9. Fie f,g: A ~ A c
fog i gof sunt inversabi'
I Solutie I
"==:-" f,g . inversab
bijective, deci inversabih
"<=" Daca fog !li
h
1
,h
2
: A ~ A astfel Tncat
h2o(gof) = 1 A. deci fo(go
= 1 A ::> fo(goh1) = (h2 c
unde rezult!i eli f i g su
10. 0 multime A
o parte strict!i a sa. Sl
naturale este infinit!i.
l Solutie I
"==:-" Fie A =
B = {b1,b2, ... bk}. atune
adica k=n. Aceasta TnsE
"<=" Presupuner
construim inductiv o mi
a
2
eA\{a1}, a3eA\{a1,a2
Fie S = { a1,a2,E
A-A\{a1}, ceea ce con1
\txeR, deci
cos
2
y = 1,
obtinem
xroz, deci
=
rezulta
Prin urmare, 81<;;;:82.
"::" Fie YEY fixat. Presupunem ca f
1
({y}) = 0. Atune;
\ty1ef(X), avem 0 = f
1
({y}) <;;;: f\{y1}),deci {y} <;;;:{y1}, adica y = Y1.
Rezulta f
1
({y}) ""0, ceea ce este absurd.
Rezulta ca, \iyeX, 3 x eX astfeJ incat y = f(x) <::::> f
1
({y}) =
{x}., 0, adica f este surjectiva.
9. Fie f,g: A ~ A doua functii. Atunci f,g sunt inversabile <::::>
f o g ~ ; got sunt inversabile.
/ Solutie /
"=>" f,g inversabile => f,g bijective, deci fog !ii got sunt
bijective, deci inversabile.
"<=" Daca fog ~ ; got sunt inversabile, exista doua functii
h1,hi A ~ A astfeJ incat (fog) o h1 = h1o(fog) = 1A !ii (got) o h
2
=
h20(gof) = 1A, deci fo(goh1) = (h1of) og = 1A !ii go(foh2) = (h
2
og)of =
= fA ::> fo(goh1) = (h2 og) of= 1A !ii go(foh2) = (h1of) og = 1A, de
unde rezulta ca f !ii g sunt inversabile.
10. 0 mulfime A este finita <::::>A nu este echivalenta cu nici
o parte stricta a sa. Sa se demonstreze ca multimea numerelor
naturale este infinita.
/ Solufie)
"=>" Fie A = { a1,a2, ... ,an} ~ ; 8<;;;:A cu 8-A. Daca
8 = {b1,b2, ... bk}, atunci {1,2, ... ,n} - {1,2, ... ,k}, deci nsk !ii ksn,
adica k=n. Aceasta inseamna ca 8 = A.
"<:::" Presupunem prin absurd ca A este infinita. Atunci
construim inductiv o multime numarabila S <;;;: A astfel: a1 eA fixat,
a2eAI{a1}, a3eA\{a1.a2}.
Fie S = {a1.a2,a3, ... }. Atunci A-S-A\S ~ ; S-S l{a1}, deci
A-AI{a1}, ceea ce contrazice ipoteza. Ramane ca A este finita.
13
Functia definita prin f(x) = 2n este o injectie
nesurjectiva. Deci f(N) = 2N, N-f(N) c N, ceea ce arata ca N nu
"
este finita, fiind echipotenta cu o parte stricta a sa.
11. Fie A infinita. Sa se arate ca:
a) Exista multimile B lli C cu 0 'i B, C 'i AA = BuC, BnC =
0. cardC = x o.
b) Oricare ar fi multimea X cu cardX xo avem card(AuX)
= cardA
I Solufie I
a) Fie b1, C1EA, apoi b2, C2EA\{b1,c1}, b3, C3E A\{b1,b2,c1,c2}
!1.a.m.d.
Astfel obtinem multimile numarabile B' = {b1,b2, ... , bn, ... }
C = {c1,C2, ... , Cn, ... }.
Evident 13 = A\C :::J B', de unde A= BuC.
b) Fara a particulariza, putem admite ca XnA = 0. Cum A=
BuC, unde C este numarabila lli AuX = Bu(CuX), unde CuX este
tot numarabila, (ca reuniune a unei multimi numarabile cu o
multime eel mult numarabila), daca:
i) X este numarabila, rezulta X-CuX lli deci A-B. Prin
urmare, AuX-BuX-BuC =A
ii) Y eX lli card Y < x o, atunci card A card(AuY)
card(AuX) = card A, de unde card (AuY) = card A
Observa(ie: Daca multimea X este numarabila, cu AnX =
0, relatia AuX -A se mai poate scrie card A+ xo = cardA, adica
x o este element neutru fata de operatia de adunare a numerelor
cardinale transfinite (numerele Cardinale ale multimilor infinite).
12. Sa se arate ca reuniunea unei familii numarabile
(disjuncte sau nu) de multimi numarabile este o multime
numarabila.
14
., Solutie I
Fie An = {a;,a;, ... ,l
disjuncts de multimi num
prin f(a:) = (i,j) este o
Tntr-adevar, din A. r
i = k lli j = I, adica f estE
Al1adar, card <UA.) =cal
,
Ramane de aratat c
Deoarece funqia f
injectiva, rezulta cardNxN
Pentru a demons
suficient sa gasim o injE
(m + n)(m + n +I) lli (m,n),
2
Daca m+n = 111'+n'
n = n', deci m = m', adil
ipotezei. Prin urmare, g(m
Daca m+n * m'+n',
(m+n+
g(m' ,n') <:
(m+nXm
m + n + 1 + n' >
2
Deci m+n < m'+n' :
Prin urmare, functia g
numellte numarare diag(Jj
Astfel card(NxN) =
Daca familia nu es
deoarece card(UA.) <: Ci
1. Nun
2. Ave
x:=
Fie An= {a;,a:, ... ,a;, ... }, n = 1.2 .... , o familie numarabila
. disjuncta de multimi numarabile. Functia f: UA. --+NxN, definita
'
= (i,j) este o bijectie.
, lntr-adevar, din Ai rAj= 0 pentru i * j, daca a; =a:, atunci
k j = I, adica f este injectiva. Surjectivitatea este evidenta.
card (UA.) =card (NxN).
"
Ramane de aratat ca multimea NxN este numarabila.
Deoarece functia f:N --+ NxN definita prin f(n) = (n,O) este
lnilctiva. rezulta cardNxN :::: cardN.
Pentru a demonstra inegalitatea de sens contrar, este
.J!jcient sa gasim o injectie g: NxN --+ N . Fie g(m,n) = n +
(ltn)(m+n+l) (m,n),. (m', n').
/' 2
l.. Daca m+n = r.n'+n', atunci din g(m,n) = g(m',n'), ar rezulta
Ttl" n', deci m = m', adica (m,n) = (m',n'), ceea ce este contrar
iptezei. Prin urmare, g(m,n) * g(m',n').
'f Daca m+n * m'+n', atunci m'+n' 2: m+n+1, de unde:
. ( , ') > (m+n+1Xm+n.+2) + , (m+n)(m+n+l) +
' gmn_ n=
' 2 . 2
+n+ 1 + n' > (m+n)(;+n+l) + n = g(m,n)
Deci m+n < m'+n' g(m,n) < g(m'+n') g(m,n) * g(m',n').
urmare, functia g definita mai .sus (aceasta functie se
numarare diagonala) este injectiva.
Astfel card(NxN) = x o, deci card(UA.) = x o.
"
Daca familia nu este disjuncts, obtinem card(UA.) s xo
"
"
Observa(ie: 1. Numararea diagonals este surjectie
2. Avem n x o = x o similar (ex.22 propus)
x: = xo.
15
13. Sa se arate ca multimea numerelor algebrice este
numarabila.
Se numar algebric un numar real care este solu(ie
a unei ecua(ii de gradul n, P(x) = 0, cu
I Solutie I
Multimea ecuatiilor de grad n, nEN*, cu coeficienti in 71.
este numarabila intrucat multimea este cardinal echivalenta cu
N. Deoarece fiecare ecuatie de grad n are eel mult n radacini
reale, rezulta ca multimea Sn a solutiilor reale ale acestor ecua(ii
este numarabila (reuniunea numarabila de multimi finite este
numarabila). Multimea numerelor algebrice este A= Us ..
n"'J
Deoarece reuniunea numarabila de multimi numarabile este
numarabila (vezi exercitiul12) rezulta ca A este numarabila.
14. Aratati ca multimea numerelor reale R nu este
numarabila.
I Solutie I
Procedeul diagonal allui Cantor.
Presupunand ca R este numarabila, cum R-[0, 1], rezulta cil
[0, 1] = { ao,a1 ,a2, ... , an, ... }. Orice numar real subunitar pozitiv se
poate scrie sub forma unei fractii zecimale O,xox1 ... , Xn ... , unde X;
sunt cifre cuprinse intre 0 9. Astfel, elementele din [0, 1] se scriu:
-Oa" "a" a"
ao-! Oal 2''' n'''
a1 = ... ...
an= ... ...
F1e numarul real b = O,bob1b2bn unde b;= '
. {2 dacii a' * 2
I daca a:= 2
Deoarece bE[0,1], rezulta ca exista n astfel incat b = an.
Scrierea zecimala a lui b fiind unica, din b = an, rezulta ca bn = a,
ceea ce contrazice defini(ia lui b.
16
;
15. Sa se indice o t
I Solutie I
lntrucat (0,1] = QC
f:(0,1] (0, 1) ca 1
& ( 1 1 J ( 1 .
'k: 2''2' 2''
sau, f: (0,1]
unde A =
(0,1) = -,-,-, ... uA.
{
1 1 1 }
. 2 3 4
16. Sa se gaseasc
multime cu trei elemente.
I Solutie I
Se pot defini 29
topologii distincte f(n) c
elemente este majorat de
Pentru X= {a,b,c},
t3 = {0,{b},X}. t4 = {0,1
't6 = {0,{a}.{c},{a,c}.X}.
ta = {0,{a,b}.X}, tg ={I<
t
1
1 = {0,{a},{a,b},X}, t1
t13 = {0,{a}.{a,c},X}. t1
t1s = {0,{b},{b,c},X}, t1
t
17
= {0,{a}.{a,b}.{a,c},:
t19 = {0,{c},{a,c}.{b,c},;
t21 = {0,{b},{c},{b,c},{a
este solutie
ech.ivaiEmta cu
n radacini
cestor ecuatii
finite este
R nu este
1), rezulta ca
pozitiv se
x .... , unde X;
(0, 1) se scriu:
dacii a: * 2
I dacii a: = 2
incat b = a .
cab =a"
n "'
15. Sa se indice o bijectie intre intervalele (0, 1) (0,1)
I Solutie I
intrucat (0, 1) = O(J.,J.] (0, 1) =O(J.,J.) , definim
'"' 2 2 2 2
f:(0,1)-+ (0,1) ca functia a carei restrictie Ia (;, .;, J este
. "ect" ' ( I I ] ( I I ) d f" . . f ( ) 3
In' 1a1k: -,,-, -+ -,-, em1apnn kx = --x,
' 2 2 2' 2' 2'
sau, f: (0,1)-+(0,1), f(x) = {x,l x kEN,
k+l' k'
unde A = (0,1)\H++ deci, (0,1) = .. -}uA
(0,1) = H+, ..
16. Sa se gaseasca toate topologiile care pot fi definite pe o
multime cu trei elemente.
I Solutie I
Se pot defini 29 de topologii. in general, numarul de
topologii distincte f(n) care se pot defini pe o multime cu n
elemente este majora! de 2n(n-l)_
Pentru X= {a,b,c} -r
1
= {0,X}, -r
2
= {0,{a},X},
t3= {0,{b},X}, 't4 = {0,{c},X}, -rs = {0,{a},{b},{a,b},X},
te = {0,{a},{c},{a,c},X}, -r
7
= {0,{b},{c},{b,c},X},
ts= {0,{a,b},X}, -rs = {0,{b,c},X}, -r10 = {0,{a,c},X},
t11 = {0,{a},{a,b},X}, -r12 = {0,{b},{a,b},X},
t13 = {0,{a},{a,c},X}, 't14 = {0,{c},{a,c},X},
t1s= {0,{b},{b,c},X}, -r1s = {0,{c},{b,c},X},
t17= {0,{a},{a,b},{a,c},X}, -r,
8
= {0,{b},{b,c},{a,b},X},
t,g= {0,{c},{a,c},{b,c},X}, -r
20
= {0,{a},{b},{a,b},{a,c},X},
t21 = {0,{b},{c},{b,c},{a,b},X}, -r22 = {0,{a},{c},{a,c),{b,c},X},
17
1:23 = {0,{a},{b},{a,b},{b,c},X}, 1:24 = {0,{b},{c},{b,c},{a,c},X},
'tzs = {0,{c},{a},{a,c},{a,b),X}, 1:26 = {0,{a},{b,c},X},
1:21 = {0,{b),{a,c},X}, 'tza = {0,{c},{a,b},X},
1:29 = {'P(x)}.
17. Fie , familia de parti ale lui R
2
formata din 0 i
multimile cu proprietatea ca, odata cu un punct, contin un disc
deschis cu centrul in acel punct (Se numete disc deschis cu
centrul in punctul (a,b), de raza r > 0, multimea D = {(x,y)l(x-a)
2
(y-b)
2
< ~ ) . Sa se arate ca ' este o topologie pe R
2
, spatiul
topologic (R
2
, 1:) fiind separat.
I Solutie I
Vom a rata ca axiomele topologiei sunt verificate.
i) Fie A,BE't. Deoarece 0E't prin constructie, dacaAnB =0,
atunci AnBE't.,Daca AnB ,.0, fie (a,b) E AnB._Deoarece A,BEt,
exista un disc deschis 0
1
c A cu centru! in (a, b) i un disc deschis
Dz c B cu centrul-jn ;(a,b),.Cele doua discuri fiind concentrice,
rezulta ca unul este'indus Th celalalt. Fie D1 c Dz.
Ob\inem 0
1
c AnB. Deci exista un disc deschis cu centrul in
(a, b) continut in AnB, de unde AnBE't.
ii) Fie A; E't \tiEl i (a,b)E UA .. Rezulta ca exista kEI
,.,
pentru care (a,b) EAk, deci exista un disc deschis DcAk cu centrul
in (a,b). Dar DcUA, , de unde UA, E't.
iEl JE!
iii) 0E't prin constructie, iar R
2
E't deoarece orice disc
deschis este continut in R
2
.
Cum \iA=(xA,YA), B=(xs,Ys) i A ,. B, (3)DA,Ds astfel inca!
DAnOs ,. 0, (ambele discuri avand, de exemplu, raze egale cu o
treime din distanta de Ia A Ia B), deci spatiul topologic este
separat.
18
18. Fie X o multirr
, = {X,0}u{GIC
(topplogia cofinita). Sa
neseparat.
I Solutie I
Prin constructie, :
Fie G1,G2 E't \{!
cdi sunt finite prin il
'G;t'IG
2
et. Pentru G1 = i
Fie G;et, iel i I'
VG, = c(oo,
no, este finita. ReZI
...
sunt verificate. ,
. Presupunem ca
urmare, exista v . Vy '
Vxf"\Vy * 0 :::) 3 Gx.G,
.plus Gy * 0. Deci Gxe
Gx este finita. Dar X=
Cum GxEt, rez:
c'eea ce contrazice ipo
19. Pentru E c!
'
interioare, E
exterioare, E
frontiera, frE
-
aderente, E
de acumula
izolate, lzE
'
Evident E cE,
lzE = E\E'
Sa se gaseasc
,c},X}.
din 0 toate
un disc
deschis cu
{(x,y)j(x-a)
2
+
R
2
, spa(iul
=0,
A,Ber,
djsc deschis
cu centrul in
cu centrul
orice disc
astfel incat
egale cu o
POIClQIc este
18. Fie X o mul(ime infinita. Sa se arate ca:
' = {X,0}u{GjCG finita} determina o topologie pe x
(topologia cofinita). Sa se arate ca spa(iul topologic (X, ) este
neseparat.
I Solutie j
Prin constructie, X, 0Et
Fie G,,G2 Et \{0}, G,nG2 = qcG,uCG2). Cum CG
1

sunt finite prin ipoteza CG,uCG2 este finita. Rezulta
G,nG2Et. Pentru G, = 0 sau G
2
= 0, evident G,nG
2
= 0Et.
Fie G;Et, iEI {ijiEI, G;"" 0}.
va, = c(oa,). Mul(imile CG;, iEI' sunt finite, deci
nG, este finita. Rezulta ua. Et. axiomele topologiei
llf' iel
sunt verificate.
Presupunem ca (X, t) este separat. Fie x,yEX, x "" y. Prin
urmare, exista Vx, Vy vecinatati ale lui x, respectiv y, astfel incat
Vy{'IVy ot 0 3 Gx,GyEt, CU XEGxcVx, yEGycVy, GxllGy "" 0. in
plus Gy"" 0. Deci GxcCGy cum CGy este finita, deducem ca
Gx este fin ita. Dar X = Gx uCGx.
Cum GxEt, rezulta ca CGx este finita, deci X este finita,
ceea ce contrazice ipoteza.
19. Pentru E c R(R) se definesc r:nul(imile punctelor:
interioare, E = {xER j3UEV(x), UcE}
0
. .......,
extenoare, extE = {xERjxE CE}
frontiera, frE = {xERj'v'UEV(x) ::::> UnE.,.0, UnCE ""0}
aderente, E = {xERi 'v'UEV(x) UnE.,.0}
de acumulare, E' = {xERj'v'UEV(x) ::::> UnE\{x}ot0}
izolate, lzE = {xERI3UEV(x), UnE = {x}}
" "
Evident E cE, frE = frCE, E = EvE' = EufrE = E ufrE
lzE = EIE'
" -
Sa se gaseasca C E .CextE, CfrE, C E, CE', ClzE
19
I Solutie I
0 0
XECE <=> XE R, Xi!' E <=> x E R A XER, :JUEV(x), UcE ~
XE R A (Xi!' R v 'v'U, UEV(x) v UnCE >'0) <=> XER A 'v'UEV(x)::)
0
UnCE o00 <=> XE CE. Deci C E = CE
"
Analog CextE = E, CfrE = E uextE, C E = extE.CE' =
extEulzE i ClzE = extEuE'
' - '
Observa(ie. Din CfrE = E uextE rezulta ca R = E u extE u
frE, deci cele trei multimi (disjuncte) realizeaza o partitie a spatiului
(in cazul in care sunt nevide).
20. Sa se arate ca daca frE = 0, atunci EE { R, 0}
I Solutie I
'
Presupunem ca exista XE E ~ i YE extE. Cum x ., y, avem
x<y sau y<x. Sa admitem ca x<y.
' '
Fie z = sup( E n[x,y]) E[x,y]. Deoarece E, extE sunt
'
deschise, exista intervalele lxEB(x), lyEB(y) astfel incat XEix c E ~ i
yEiyECE, de unde, tinand seama de proprietatHe marginii
superioare a unei multimi avem ca x < z < y. Pe de alta parte, din
' '
ZE E u extE, rezulta ca ZE E sau ZE extE. Prin urmare, exista un
'
interval lzEB(z) astfel incat lzcE c(x,y) sau lz c extE n(x,y),
0
ambele relatii fiind in contradictie cu faptul ca z = sup( E n(x,y)].
'
lpotezele E o00 ~ i ext Eo00 au condus deci Ia o contradictie. A ~ a d a r
' - -
E = 0 sau extE = 0, adica EE { 0, R}. A ~ a d a r R( R) sunt conexe
(nu pot fi descompuse intr-o reuniune de doua multimi deschise,
nevide ~ i disjuncte).
20
21. Fie AcR. Atune
I Solutie I
Relatia este echiv
o:> X E FrA = A II CA
. I) ,J_, i
xeCCA = A <=> XE CA 1...
I
A!iadar, R\FrA = )
Observa(ie. Tn ac
' .
oarecare avem X = Au c
22. Sa se arate 1
conexA.
I Solutie I
Folosim metoda 1
este conexa.
Deci exista B1.B2
81118, = 0.
Notam cu C1 = A
v( AnB
2
) = An(B1vB2;
cAn 8,nAIIB2 = 0 ~ i (
.. c2. deoarece in caz c
AcB2 => A cB, => A,r
A nu este conexa, cat
este conexa.
23. Tn topologia c
a) Orice multimi
b) B = { {x}IXEX
= extE.CE' =
' .
E uextE v
a spatiului
ot. y, avem
marginii
parte, din
n(x,y),
n(x,y)).
"
" ....---.
21. Fie AcR. Atunci R = A v CA vFrA (CA = C!i!. A)
J Solutie I
"
" ....---.
Relatia este echivalenta cu R\FrA = AvCA. Dar xeR\FrA
"
- ....---.
FrA = A n CA <=:> XI!! A v XI!! CA <=:> XE CA = CA v
"
- " ....---. "
XeCCA = A <=:> XE CA V A.
" " ....---.
R\FrA= AvCA.
Observafie. fn acelal?i mod se arata ca pentru AcX, X
" ,.JL,
oarecare avem X = A v CA vFrA (CA = CxA).
22. Sa se arate ca daca A este conexa atunci l?i A este
conexa.
I Solutie I
Folosim metoda reducerii Ia absurd. Presupunem ca A nu
este conexa.
Deci exista 8,,82 "- 0 astfellncat A = 8,v82, B, n8
2
= 0,
B,I"'IB, = 0.
Notam cu C, = An8, C2 = An82. Avem C,vC2 = (An8,)
u( AI"'IB2) = An(8,v82) = An A= 0, c, nC2 = (An8,) n (An82)
CAI"'IB,nAn82 = 0l?i c,nc,= (An8,) n( An82) = 0. Cum c, of. 0
ot. C2, deoarece In caz contrar C
1
= 0 An8
1
=0 An8
2
= A
Ac82 A c B, A, n8, c B, n8, =0, rezulta 8
1
= 0, multimea
A nu este conexa, ceea ce contrazice ipoteza. Prin urmare, A
este conexa.
23. fn topologia discreta (X, -r) sa se arate ca:
a) Orice multime AcX are frontiera vida.
b) 8 = { {x}/xeX} este o baza a spatiului topologic.
21
I Solutie I
'

a) Fie multimea AcX un punct XEX cu x,., CA (care nu
este punct exterior pentru A). Rezulta ca CA nu este o vecinatate
pentru x, deci x,.,CA, deoarece In topologia discreta CAet.
'
Obtinem XEA cum Aer:, avem XE A. Am aratat astfel ca orice
punct care nu este punct exterior pentru A este punct interior
pentru A, deci FrA = 0.
b) Sa aratam ca orice element din -r: se poate scrie ca o
reuniune de elemente din B. Fie AE-r:; avem A= U{x}, deci B este .
o baza a spatiului topologic.
A
24. Fie X o multime cu eel putin doua elemente t
topologia grosiera. Sa se arate ca orice punct XEX este un punct
de acumulare pentru orice multime a spatiului, In afara de 0 de
multimea {x}.
I Solutie I
Fie un punct XEX AcX, A,o0, A,o{x}. Punctul x este punct
de acumulare pentru A, daca orice vecinatate a lui x are
proprietatea 0/x \ {x})llA;r0.
intrucat -r: = {0,X}. singura vecinatate a lui x este X deci
(X \ {x} )nA;r0, multimea A continand eel putin un element al lui X
diferit de x. Rezulta x EA.
25. Fie X= {a,b,c,d,e} o topologie pe X
-r: = {X,0, {a}, {a,b}, {a,c,d}. {a,b,c,d}, {a,b,c}}.
a) Sa se gaseasca:
i) familia multimilor lnchise In (X, -r:);
ii) inchiderile multimilor {a}.{b}.{a,c,e};
iii) interiorul multimilor {b,c,e}, {a,b,c,e};
iv) exteriorul frontiera multimii {b,c,e}
22
b) Sa se arate
i) (X, -r:) este un
ii) multimea { a,c
I Solutie I
a) i) Familia m
complementarelor mult
{c,d,e}, {b,e}, {e}, {c,d}
ii) Folosind faptu
{a}= X= {a,c,e:
iii) Folosind faptl
,......L.,
{ b,c,e} = 0
,........L.,
{ a,b,c,e} =
iv)Ext { b,c,e} = i
'
Fr{b,c,e} = CA
{b,c,d,e}
b) i) Fie a,ceX, '
oricare ar fi Va.Vc veci
(X,-r) este separabil inti
ii) Multi mea { a,c
dar nu este inchisa de1
este separat, orice mu
nu este adevarat), deci
'26. Aratati ca:
A este' deschi
b) A este
c) A este lnchis-
....l,
CA (care nu
o vecinatate
CA e 1:.
astfel ca orice
punct interior
b) Sa se arate ca:
i) (X, 1:) este un spatiu topologic neseparat i separabil;
ii) multimea {a,c} este compacta, dar nu este fnchisa .
I Solutie I
a) i) Familia multimilor fnchise In (X, -r) este familia
complementarelor multimilor deschise: .Y = {0,X, {b,c,d,e),
(c,d,e), (b,e), (e), (c,d));
ii) Folosind faptul ca A= nF, obtinem:
,..,.
AcF
{a} =X= {a,c,e} !?i {ll} =Xn{b,c,d,e} n {b,c};
iii) Folosind faptul ca A= UG' obtinem:
0
,.......___
{ b,c,e} = 0
0

GcA

{a,b,c,e} = 0l:.!{a}u{a,b}u{a,b,e} = {a,b,c}
0 0
iv)Ext {b,c,e} = = {;;:d} = 0u{a}={a}
"
" ---
Fr{b,c,e} = CA n C CA = C0nC{a} = Xn{b,c,d,e} =
{b,c,d,e}
b) i) Fie a,ceX, 'v'Vc, {a,b,c} c Vc ==> {a} cVanVc, deci
oricare ar fi Va.Vc vecinatati ale lui a !?i c, VanVc "'0. A$adar
(X,t) este separabil fntrucat {a} = X ..
ii) Multimea {a,c} este compacta, fiind o multime finita,
dar nu este lnchisa deoarece { a,c} 1'.<7. Daca spatiul topologic
este separat, orice multime compacta este lnchisa (ceea ce
nu este adevarat), deci (X, -r) este neseparat.
26. Aratati ca:
a) A este deschisa daca !?i numai daca frA = A -A
0
b) A este fnchisa daca !?i numai daca frA =A-A
c) A este lnchis-deschisa daca !?i numai daca A= 0.
23
I Solutie I
a) Aer: => CA = CA => frA = A n CA = A n CA = A -A
0
frA = A -A => AnfrA = 0 => A= A-frA = A =>A este
deschisa.
- - 0 0
b) A= A => frA =A n CA= AnCA = AnCA= A-A
'
frA = A -A=> frA => A = AufrA = A
c) Daca A este o multime inchis-deschisa, atunci
frA = AnCA = AnCA= AnCA = 0.
Daca frA =0, atunci AnCA =0 =>AnCA=0 =>Ac A
0 - - 0
Dar AcAcA =>A= A =A
27. Sa se arate ca xeE' => card(UnE) xo
I Solutie j
Este evident ca daca card(UnE) xo, VUeV(x) => xeE'.
Reciproc, daca xeE' ar exista UeV(x) astfel inca!
card(UnE) < x
0
, atunci va exista printre punctele din UnE unul
care sa fie eel mai apropiat de x. Fie acesta y. Prin urmare, luand
0 < E < 1 x-y 1. in W = (X-E, x+E) eV(x) nu va exista nici un punct din
A, in afara, eventual, de x, ceea ce contrazice ipoteza ca x e E'.
28. Fie Ep = { _!_+_!_; neN*}, peN* E = QEp
p n pl
i) Sa se gaseasca Ep'
ii) Sa se a rate ca E nu este inchisa
iii) Sa se demonstreze ca F = Eu{O} este inchisa
I Solutie I
Pentru p fixat, avem:
Ep = {_!_ + t} +_I_, _I_+.!., ... }
p p 2 p 3
24
ln orice interval
suficient sa luam xeN*
1 1 (1 1 ) D . 1
-+-E --E,-+E . eCI.-,
p n p p p
nu este punct de acumulare
avem .Tn concluzi1
ii) Oricare ar fi inter
1 1 I
incAt -+-< E, vom avea-
p n I
0, .!.. eE'. Evident 0 e;E, dec
p
iii) Cum F' = {_I_ , p
p
Deci F' c:F, ceea ce 1
29. Fie A= (0,1) n(
in (0, 1) care acopera pe
.dea exemplu de c: (0
\ Solutie I
'
Deoarece = fj, , a\
rezultA cA c: [0,1) \
Cum in U se afiA n
Deci adicA [0,1) \
egalitatea.
=>XEE'.
astfel Inca!
UnE unul
luand
In orice interval (*-E,*+E), exista puncte din Ep; este
suficient sa luam XE N* astfel I neat ~ < , caci atunci
n
I I (I I ) D . I E ' D - R 1 t . t
-+-e --e,-+E . ec1: - E p. aca xe , x"' -,a unc1 aces a
p n p p p P
nu este punct de acumulare al multimii Ep, deoarece, pentru n>no,
. I ~ I I avem x- >-. In concluzie, Ep' = {- }
p n p
ii) Oricare ar fi intervalul (-,s), luand numerele p,n astfel
I I I I 1 I
incA!-+-< s, vom avea -+- E(-s,s). Cum -+- EEp, va rezulta
p n p n p n
0, .!. eE'. Evident 0 !1' E, deci E' cr:. E, adica E nu este lnchisa.
p
iii) Cum F' = { , pEN*} U{O} = E', din
I
-eE2pcF.
p
p p
Deci F'cF, ceea ce arata ca F este lnchisa.
1 1
= -+- ~
2p 2p
29. Fie A = (0, 1) nQ, t-.r, un ~ i r de multimi deschise incluse
in (0,1) ~ i care acopera peA. Sa se arate ca frt-. = (0,1)\ t-.. Sa se
deaexemplu de ~ i r Lln c (0,1}, AcUt." =I'., cu frt-. 7'0.
' 1 ~ 1
"
Deoarece t-. = t., avem ca frt-. nt-. 7'0 i dim frt-. c ::i c (0, 1];
rezulta ciHrt-. c [0, 1] \ t-. ~ i U EV(x); evident XE UnCt-. 7'0.
Cum In U se afla numere rationale ~ i Act-., avem Unt-. 7'0.
Deci xefrt., adica [0, 1) \ t-.c;frt.. Cele doua incluziuni demonstreaza
egaiHatea.
25
Daca A= {r1.f2 .... , rn, ... } consideram En suficient de mici,
astfel Tncat intervalele Llx = (rn-En, rn+En) sa fie incluse Tn (0,1),
atunciAcil= {0,1} cfril.
n;l
30. Fie A= (0, 1) n Q. Sa se determine A', A frA.
j Solutie I
(V)Xo E[O, 1] E > 0, rezulta ca 1: = (xo-E, xo) n (0, 1) ,.;0 sau
J: = (Xo, Xo+&)n (0,1) ,.;0, deci lnQ ,.;0 sau JnQ ,.;0; urmeaza Ca
(An(V,(xo) \{xo}) ,.;0, unde V,(xo) = (xo-E, Tncat xo E A' =>
[0, 1] cA'.
Reciproc, daca x0 E A' x
0
E[O, 1] avem, de exemplu, Xo<O,
deci [0, 1] nV,(xo) =0 pentru 0 < E < IXol- Rezulta ca A nV.(xo) =0,
deci x
0
lf.A', ceea ce este absurd.
Ramane ca x0 E[O, 1], deci A'r;;, [0, 1]. Prin urmare A'= [0, 1].
A= AuA' = ((0,1) nQ) u[0,1] = [0,1]
" "
Presupunem A,.; 0 fie x
0
E A, fixat. Atunci >0, cu
(Xo -, c AcQ (1)
Cum intervalul deschis (xo -&, contine numere
"
irationale, relatia (1) este absurda. Ramane ca A=0.
--- --
frA = AnCA = AnCA = AnC0 = AnR = 0.
31. Fie A r;;, R o multime arbitrara D r;;, R o multime
deschisa. Atunci:
i) (A\0)' cA' \0, A\ Dr;;, A \0
ii) A nOr;;, AnD, A' nOr;;, (AnD)'
I Solutie I
(A\0)' = (AnCD)' r;;, A' n(CD)'
Cum D este deschisa, rezulta ca CD este Tnchisa, deci
(CD)' r;;, CD;
i) (A\0)' c A' nCO =A' \ D. Similar A I Dr;;, A \0
26
ii) Fie XE Ant
deci, cum
(AnD)nV,.;
, Observa(ie: Daca
$Cleva rate.
i)
ii)
A= (0,1), D.
R\O=Qn
32. Sa se
('fnchise) din R, pentru o
deschisa (inchisa).
I Solutie I
Fie An= (-.!_ ).. ),n;;
n n
A= nA. = {0}, deci
t!N
. I :
F1e An = [-, 1], n
n
A= nA, = (0, 1], deci

33. Fie AcR o
fir de multimi inchise (dE
(A= nA,).
"'"
I Solufie I
A fiind deschisa,
deschise nevi de cu A= l
i
poate scrie ca reuniune a 1
Cum reuniunea u
numarabila, functia f:NxN-
En suficient de mici,
fie incluse Tn (0, 1 ),
Xo) n (0, 1) sau
unneaza ca
Tncat xo E A'
de exemplu, Xo<O,
ca A nV,(xo) =0,
urmare A' = [0, 1].
!?i numere
<;;; R o multime
inchisa, deci
d/:,du.J0Y
------._
ii) Fie XE A nD fixat. Atunci VVEV(x) avem DnVEV(x),
deci, cum XEA, rezulta An(DnV) !?' deci
(AnD)nV 0 . Prin urmare, XE Ar;D.
Observafie: Daca D nu este deschisa, relatiile nu sunt
adevarate.
i) A=(0,1),D=Q
ii) R I Q = Q n( R I Q) <;;;Qn(R IQ)=0
32. Sa se construiasca un !?ir (An)n
1
, , de mulfimi deschise
(Tnchise) din R, pentru care multimea A'= nA" <UA") nu este
deschisa (inchisa).
neN neN
I Solufie I
Fie An= (-l, l. ),n;:: 1. Atunci An este deschisa \! n;:: 1 !?i
n n
A= nA" = {0}, deci multimea A nu este deschisa.
neN
Fie An = f l, 1 ], n :2: 1. Atunci An este Tnchisa \! n ? 1 !?i
n
A= nA. = (0, 1 ], deci multimea A nu este Tnchisa.
...
33. Fie A<;;;R o multime deschisa (inchisa). Atunci exista un
de multimi inchise (deschise) (An)n 1, astfel incat A =UA"
(A= nA. ).
neN
""
I Solufie I
A fiind deschisa, (3)(D;);E1 o familie numarabila de intervale
deschise nevide cu A=UD,. fnsa, evident, fiecare D; (iEI) se
poate scrie ca reuniune a ur\ui !?ir (J; )n >
1
de intervale inchise.
Cum reuniunea unui !?ir de multimi numarabile este
numarabila, functia f:NxN definita prin f(n,m) = a;,, unde {a:,
27
a;, ... } este o enumerare a multimii A, V n 1 fiind bijectie, familia
(J J (n,i)ENxl este numarabila.
Daca A este inchisa, avem R\A deschisa, deci 3(Bn) n 1
un de multimi inchise, cu R\A = UB". Punem An= CBn (n 1);
""
atunci, fiecare An(n 1) este o multime deschisa avem:
A= C(UB,) = n<CBJ= nA" .
p<:l n<:l
34. fn topologia
multimea A = (0,1] u{3}. Sa
componentele conexe ale lui A.
[iolutie \
a dreptei reale se considera
0
se determine A, A ,FrA, A'
0
A= (0, 1 ), deoarece orice punct din (0, 1) are ca vecinatate
multimea (0,1), iar 1 3 nu sunt puncte interioare, orice
vecinatate a lor continand puncte din CA.
A = [0,1] u{3}, deoarece 0 este punct aderent al lui A,
orice vecinatate a sa continand puncte din A; evident, orice punct
x,;[0,1] u{3} nu este punct aderent lui A.
FrA = AnCA= {0,1,3}.
A' = [0, 1], deoarece 3 care este punct aderent al lui A, nu
este punct de acumulare, exista V3 = (2,4) astfelincat (V3 \ {3}) n A=0.
Multimea A are doua componente conexe A1 = (0,1]
A2 = {3}.
35. fn topologia a dreptei reale, sa se dea
exemple de multime:
a) inchisa, pentru care exista o acoperire cu deschise din
care nu se poate extrage o acoperire fin ita;
b) nemarginita, pentru care exista o acoperire cu deschise,
din care nu se poate extrage o acoperire fin ita;
c) compacta, pentru care exista o acoperire cu nedeschise,
din care nu se poate extrage o acoperire fin ita.
28
'
I Solutie I
a) A= (-1,1) cu <
nEN};
b) 8 = [O,oc) cu ace
c) C = [0,1] cu ace
36. fn topologia ol
de o functie continua f: E
a) f nemarginita,
b) f(E) neinchisa
c) f marginita, da
d) f nemarginita,
e) f(E) inchisa
I Solutie \
a) Fie E = (0,1}
elementara nemargi1
este marginita, dar necc
b) Fie E = (0,1)
elementara, iar multimE
deschisa. Multimea f(E)
c) Fie E = (0,'
deoarece 'v'XE E, jf(x}j <
sup f(x) = 1, deci f nu

d) E = [0, oc),
e) E = (0,1], f(x)
, familia
avem:
orice
allui A, nu
r.A=0.
= (0,1]
dea
I Solutie I
I
a) A= (-1,1) cu acoperirea {lnlln = (-1 + -, 1- -),
n+l n+l
nEN};
b) B = [O,oc) cu acoperirea {lniln = (n-2, n), nEN};
c) C = [0, 1] cu acoperirea {ixllx = {x}, XEC}.
36. fn topologia a dreptei reale sa se dea exemplu
de o functie continua f: E c R R, E necompacta astfellncat:
a) f nemarginita, E marginita;
b) f{E) nelnchisa marginita;
c) f marginita, dar nu atinge marginile;
d) f nemarginita, E lnchisa;
e) f(E) lnchisa nemarginita.
I Solutie I
a) Fie E = {0, 1) f(x) = ..!.. . Functia f este continua, fiind
X
elementara nemarginita, deoarece lim f(x) = +oc. Multimea E
.
este marginita, dar necompacta, fiind deschisa.
b) Fie E = {0, 1) f(x) = x. Functia f este continua, fiind
elementara, iar multimea E este marginita, dar necompacta, fiind
deschisa. Multi mea f(E) = E = {0, 1) este nelnchisa.
c) Fie E = {0, 1) f(x) = x. Functia f este marginita,
deoarece \ixEE, lf(x)l <1. Din f(E) = (0, 1) rezulta ca Inff(x) = 0
e'
sup f(x) = 1, deci f nu atinge marginile.
XEE
d) E = [0, oc), f{x) = x.
e) E=(0,1],f(x)=..!...
X
29
B. Probleme propuse
1. Fie X o multime nevida. Sa se verifice ca urmatoarele
relatii Tn 'P(X) (multimea partilor acesteia) sunt relatii de ordine ~ i
sunt echivalente.
1) A ro1 B ~ AcB
2) A ro2 B ~ B = A u B
3) A ro3 B ~ A = A n B
4) A ffi4 B ~ A = A E 'P (B)
In ce conditii incluziunea induce o ordine totala pe multimea
partilor unei multimi.
jlndicatie j
Card X 2 2 ~ 'P (X) partial ordonata ( {a} ct. { b} ~ i { b} ct.
{a})
R. Cele 4 relatii sunt reflexive proprii ~ i tranzitive. Ordinea
este totala daca X= 0 sau X= {a}.
2. 0 diviziune finita a intervalului [a,b] este o multime
d = {x
0
,x1, ... , Xn} c [a,b), astfel Tncat a = Xo < X1 ... < Xn = b,
numarul natural n putand varia de Ia diviziune Ia diviziune. Norma
diviziunii d este numaruiJdl = max {x;- X;-1\.
hoo:;n
D:
Fie D = {d; d diviziune finita a intervalului [a,b) c R}.
Sa se verifice ca urmatoarele relatii sunt relatii de ordine Tn
a) ordinea dupa norma d ro1 d' ~ Jdl :::: Jd'J
b) ordinea dupa finete d ro2 d' ~ d c d'
Sunt acestea ordini totale? Coincid?
R. Relatia ro1 este o relatie de ordine improprie, iar ro2 este
relatie de ordine.
3. In multimea R consideram relatiile:
a) {(x,y); a<x<y}, unde a ER este dat
b) { (x,y); a<x<b}, unde a,b ER sunt date
c) {(x,y); x<y}, ordinea uzuala
30
d) { (x, y); a<X<y<l
e) { (x,y); a<x},
f) { (x,y); xsasy}.
i) Sa se ara1
este ordinE
ii) Sa se fac
A= (0, 1]1r
4. Fie, pe o mul1
x = y, x,y EX. Sa se ara
5. Sa se arate ci
marginita.
R. Cel mai mic 1
eel mai mare un majora
6. Sa se arate
elementele minimale 1
numerele prime.
7. Fie: a) an= 1f
b)'"" \;
Se cer: maxA, rr
jlndicatie \
Aflati limitele exl
8. Sa se cercet
inferioare ~ i superioa
reale:
A= {2n+1J nEN
mul(imea
Ordinea
Xn = b,
Norma
ro2 este
d) {(x,y); a<x<y<b};
e) { (x,y); a<x},
f)
i) Sa se arate ca aceste rela(ii sunt ordini in R. Care
este ordine totala?
ii) Sa se faca analiza structurii de ordine a mul(imii
A= (0, 1] in raport cu fiecare dintre aceste ordini.
4. Fie, pe o mul(ime oarecare X, ordinea triviala x ro y <=:>
x = y, x,y EX. Sa se arate ca orice element allui X este maximal.
5. Sa se arate ca orice mul(ime finita de numere reale este
marginita.
R. Cel mai mic element din mul(ime este un minorant, iar
eel mai mare un majorant al mul(imii.
6. Sa se a rate ca in mul(imea numerelor naturale, n> 1,
elementele minimale in raport cu rela(ia de divizibilitate sunt
numerele prime.
. n+l rut
7. Fie: a) an= 1+--cos-, nEN
2n +I 3
A= {an! nEN}
!
Jn +
2
, daca n par
b)an= nl
n+ d - .
--, aca n tmpar
3n +5
Se cer: maxA, minA, supA, infA.
llndicatie I
Afla(i limitele extreme ale acestor
8. Sa se cerceteze marginea l?i sa se determine marginile
inferioare l?i superioare pentru urmatoarele mul(imi de numere
reale:
A= {2n+1! nEN} C
. n +I N}
={sin--, nE
2
3 I
I
B = { \nEN}
n -3,4
D = 0[-2n,-2n+_!_)
2
R. Multimile B C sunt marginite, iar A D sunt multimi
nemarginite.
9. Sa se verifice daca urmatoarele relatii in multimea
.9fa,b] = {flf: [a,b] arbitrara} sunt relatii de ordine.
1) f OJ
1
g f(x) s g(x), VxE[a,b]
2) f OJ
2
g f(xo) s g(xo), VXoE[a,b]
3) f OJ3 g f-g crescatoare
b b
4) f Ol4 g s J!g(x)!dx
10. Fie multimea ff[0,1] = {f; f:[0,1] urmatoarele
relatii in aceasta multime:
1) f OJ
1
g f-g marginita
2) f OJ2 f-g monotona
3) f OJ
3
g f-g continua
Sa se precizeze care dintre
echivalenta.
4) f OJ4 g lf-gl marginita
5) f OJs g lf-gl monotona
6) f OJ5 g lf-gl continua
acestea este relatie de
11. Sa se verifice ca urmatoarele relatii sunt rela\ii de
echivalenta pe multimea de numere reale:
1) (Xn) 0J1 (Yn) 3 AcN finita, VnEN- A, Xn = Yn
2) (Xn) 0J2 (Yn) (xn - Yn) este un constant
3) (Xn) (1)3 (Yn) (Xn- Yn) este un marginit
4) (xn) OJ4 (Yn) (Xn - Yn) este un convergent
12. Sa se indice o bijectie intre punctele unui cere din care
s-a exclus un punct punctele unei drepte.
R. M(x,O); C:x
2
+ / = 1; CnAM = {M'}; A(0,1) ::::>
M' ( 2x x' -I)
x
2
+ 1' x
2
+ 1
13. Fie o familie fin ita { A1 ,A2, ... , An} de multimi nevide,
disjuncte doua ciHe doua. Sa se arate ca exista o multime E care
32
are in comun cu fiec
axioma alegerii se poa
ltndicatie I
Fiecare a;EA;
E = {X\XE LJA I
1
14. Sa se con
urmatoarele proprietal
a) f
b) f,g nebijecti
c) f sau g nein
go f neinjec
Exemplu: f:R.-
bijective, dar g o f est1
15.FieAomu
a) multimea 'P
b) ecuatia Xu'
R. a) Numarul
b) 0 soluti
PE'P(X)
16. Fie A
card(A\B) = n card
a) Determina1
b) Aratati ca:
t
R. a) cardA =
17. Arata\i ca:
a) AcX ::::> ca
b) \if, f:X
c) \if, f:X
mul(imi
multi mea
relatii de
'1) =>
are In comun cu fiecare A;, i = !,n un singur element. (adica:
axioma alegerii se poate demonstra pentru familii finite).
Jrndicatie J
Fiecare a;EA; p;(x): x = a;
E = {XIXE lJA;, P1(x) vp2(X) v ... vpn(X)}
'
14. Sa se construiasca doua functii cu
urmatoarele proprietati:
a) f nesurjectiva, g surjectiva g o f surjectiva;
b) f,g nebijective g of bijectiva
c) f sau g neinjectiva (respectiv nesurjectiva, nebijectiva)
g o f neinjectiva (respectiv, nesurjectiva, nebijectiva).
Exemplu: f(x) = x, g(x) = x
2
, f,g nu sun!
bijective, dar g o f este bijectiva.
15. Fie A o multime finita cu cardA = n. Atunci:
a) mul(imea 'P(A) are 2" elemente;
b) ecuatia XuY =A poseda 3" solu(ii (X,Y);
R. a) Numarul submultimilor lui A avand k elemente este C:
b) 0 solu(ie a ecuatiei are forma Y = (A\X) uP, cu
PE'P(X)
16. Fie A B doua mul(imi finite cu card(AuB) = m,
card(A\B) = n card (AnB) = p.
a) Determinati cardA cardB;
b) Aratati ca: b1) card(AuB) = cardA + cardB- card(AnB)
b2) card(AuB) = card(AnB) => cardA = cardB
R. a) cardA = p+n; cardB = m-n.
17. Aratati ca:
a) AcX => cardA s; cardX
b) \if, f:X Y ::::> cardf(x) s; card X
c) \if, f:X Y, f injectiva => cardX s; cardY
33
d) \if, f:X Y, f surjectiva :::::> cardY ::; card X
18. Sa se afle cardinalele urmatoarelor multimi:
n' +I
A= {xi XER, x = , n=1 ,2,3, ... , 100}
2n'+n+l
an+b
B= {xi XER, x = --, n=1 ,2, ... , p}, a,b,c,d ER, cd >0.
cn+d
C= {xi XER, x = -n
2
+6n-7, n = 1 ,2, ... }
R. cardA = 99, cardB = , cardC = 2
{
1, daca ad = be
p, daca ad * be
19. Sa se arate ca multimea A = {3n+21 nEN} este
numarabila.
R. Functia f:N f(n) = 3n+2 este bijectiva.
20. Sa se arate ca intervalele din tripletele urmatoare sun!
cardinal echivalente, functiile indicate in triplete fiind bijectii intre
intervalele respective.
a) [O,oc), [0, 1)
b)R,(-1,1)
c) (0, oc), R
d) [0, 1], [a,b]
e) (-1,1), R
f) (a,b), (0, oc)
g) (a,b), (c,d)
f(x) = _x_
x+ I
f(x) = _x_
l+lxl
f(x) = lnx
f(x) = a + x(b-a)
f(x) = tg _Zx , g(x) =
1t
f(x) = a-x
x-b
f()
_ c-d ad-be
X- --x+--
a-b a-b
a<b
c<d
21. Sa se arate ca daca X Y sun! doua multimi
numarabile, astfel inca! XnY = 0, atunci XuY este o multime
numarabila.
34
\lndicatie

g:N
22. Aratati ca pre
este o multime numaral
\lndicatie .\
A = {ao,a1, ... ,
unde An = { (an,bo), (a,
rezolvat nr.12.
23. Sa se arate
a) multimea Z s
b) multimea Q 1
c) multimea P 1
d) multimea po
e) multimea pe
lndica(ie Vom s
a) f:Z N, f(x
sau g:N
b) Q = lJA., J
"''
c) x o::; cardP
eR, cd >0.
urmatoare sunt
bijectii intre
multimi
o mul(ime
/rndicatie

I
f( E_ ), n par
h(n) =
2
n +I .

n 1mpar
g:N bijective.
22. AriHa(i ca produsul cartezian a"doua multimi numarabile
este o mul(ime numarabila.
/rndicatie /
A= {ao,a1, ... , an ... }, 8 = {bo,b1, ... , bm ... }, AxB = UA"
n'"O
unde An = { (an,bo), (an,b1), ... , (an,bm), .. } se utilizeaza exerci!iul
rezolvat nr.12.
23. Sa se arate ca urmatoarele multimi sunt numarabile:
a) mul(imea Z a numerelor intregi;
b) mul(imea Q a numerelor. rationale;
c) mul(imea P a numerelor prime;
d) multimea polinoamelor cu coeficienti ra(ionali;
e) mul(imea perechi/or de numere naturale.
lndica(ie Vom arata ca:
a) f:Z N, f(x) = { 2x, daca x :e.o ,
-I - 2x, daca x < 0
I
E_-1, n par
sau g:N g(n) = 2 n +
1
. este bijectie;
n 1mpar
2 ,
b) Q= QA" ,A1 =Z,A2= Am= {;InEZ}
c) x a cardP prin reducere Ia absurd
35
d) lnductiv ca multimea polinoamelor de grad eel mult n
este numarabila. Orice element din An+1 este de forma:
P(x)+an+
1
, PEAn. an+1 EO. Deci, fiecarui element din An+1
i se poate pune in corespondenta (P(x), an+1)
e) F:NxN -+N, f(p,q) = este o bijectie.
24. Sa se arate ca orice multime de intervale disjuncte doua
cate doua este numarabila.
\lndicatie I
Fiecarui interval (a,b) ii corespunde un numar rational
[na]+
1
, unde n = [-
1
-]+1.
n b-a
25. Sa se arate ca urmatoarele multimi sunt nenumarabile
(de puterea continuului):
a) orice interval marginit [a,b), (a,b), [a,b];
b) multimea numerelor irationale;
c) multimea numerelor transcendente (complementara in R
a multimii numerelor algebrice);
d) multimea l?irurilor ai caror termeni parcurg o multime
nenumarabila.
lndica(ie Se folosel?te exercitiul rezolvat nr. 11.
a) A= (a,b), X= {a}, [a,b) = AuX -A= (a,b) :=:- card[a,b] =
card R
b) R = luO -1 :=:-card! = cardR, I = R\0;
X, X El-aQ
c) Analog, c sau: f: 1-+ (0, 1], f(x) = 2x,
a
xEaQ,xs-
2
a
2x- 1 , x E aQ, - < x s a
2
este bijectie unde aO = {ar, rEO};
d) se folosel?te procedeul diagonal allui Cantor.
36
26. Fie X o multir
't = {X,0,A,B}, unde A!?
ale lui X. Ce conditii trebl
fie o topologie pe X?
\ lndicatie \
A !iii B determina
ordonata in raport cu rei
27. Sa se gase
multimi, pe multimea X'
28. Fie A o subn
Aratati ca 't = {X,0,A} E
29. Fie N multin
din 0 !iii din toate multir
a) Sa se verificl
b) Sa se indic
numarul K;
c) Sa se gasea
d) Determinati:
i) punc1
ii) interi1
{2,4,1
iii) toate
30. Fie X un
a) interiorul 1
multimilor c
mai mare r
b) aderenta
multimilor
multimein
c) A= AuA'
din An+1
o bijectie.
(X
2
<xsa
doua
26. Fie X o mul(ime care are eel pu(in doua elemente ~ i
t = {X,0,A,8}, unde A ~ i 8 sunt doua submultimi proprii distincte
ale lui X. Ce condi(ii trebuie sa indeplineasca A ~ i 8 pentru ca , sa
fie o topologie pe X?
/Jndicatie I
A ~ i 8 determina o parti(ie pe X sau ' este o mul(ime total
ordonata in raport cu rela(ia de incluziune.
27. Sa se gaseasca toate topologiile formate din patru
mul(imi, pe mul(imea X= {a,b,c}.
28. Fie A o submul(ime proprie a unei mul(imi oarecare, X.
Aratati ca' = {X,0,A} este o topologie pe X.
29. Fie N multimea numerelor naturale ~ i t familia formata
din 0 ~ i din toate multimile En= {n, n4-1, ... }, nEN.
a) Sa se verifice ca, este o topologie pe N;
b) Sa se indice toate multimile deschise care con(in
numarul K;
c) Sa se gaseasca familia Ya multimilor inchise in (N, -r);
d) Determinati:
i) punctele de acumulare ale multimii {2,8, 13,50};
ii) interioarele ~ i inch ide rile mul(imilor { 2,8,50} ~ ~
{2,4,6, ... };
iii) toate mul(imile EcN, astfel incat E' = N
30. Fie X un spa(iu topologic. Sa se arate ca:
a) interiorul unei mul(imi AcX este reuniunea tuturor
multimilor deschise care sunt inclusa in A (lntA este cea
mai mare mul(ime deschisa inchisa in A);
b) aderenta unei mul(imi AcX este intersectia tuturor
mul(imilor inchise care includ A (A este cea mai mica
multime inchisa care include pe A);
c) A = AuA', oricare ar fi AcX.
37
31. Determinati:
a) familia multimilor lnchise;
b) un sistem fundamental de vecinatati
In topologia cofinita (vezi exercitiul rezolvat nr. 18).
32. Fie (X, 't) un spatiu topologic .<Tfamilia multimilor
incluse In (X, 't):
a) Sa se arate ca .<Tverifica urmatoarele conditii:
(F1) X,0EF
(F
2
) Daca F
1
,F
2
EF, atunci F1uF2 EY
(F
3
) Oaca F; EY. pentru i apartinand unei familii oarecare de
indici I, atunci nF, .EY

0 familie de submultimi ale unei multimi X care verifica
conditiile (F
1
), (F
2
) (F
3
) se cotopologie pe X.
b) Sa se arate ca daca .<Teste o cotopologie pe X,
atunci exista o topologie unica ' pe X pentru care .<Tsa fie clasa
multimilor lnchise (exista o bijectie lntre multimea cotopologiilor
multimea topologiilor pe o multime X);
c) Fie R multimea numerelor reale cu ordinea uzuala.
Sa se stabileasca daca clasa intervalelor lnchise este o
cotopologie pe R.
d) Fie X o multime arbitrara .<7= {X, 0,A} o familie de
submultimi ale lui X, In care A este o submultime proprie a lui X.
Sa se precizeze daca F este o cotopologie pe X.
33. Fie R multimea numerelor reale ' familia de parti ale
lui R formata din 0 acele multimi OcR pentru care XED implica
-X ED:
a) Sa se verifice ca , este o topologie pe R;
b) Sa se arate ca In aceasta topologie o multime este
lnchisa daca numai daca este deschisa, dar , este
diferita de topologia discreta de topologia grosiera pe R.
0 -
34. Sa se determine E, ext E, frE, E, E' lzE In R, unde:
i) E = {X1,X2, .. , Xn} iii) E = 0
ii) E = [1 ,2] u(3,4) u{5} iv) E
38
R. i) 0, R'\E, E, I
[-oc, 1) u(2,3) u(
[1,2] u(3,4), {5}; iii) C
E=Q, extE = R\{0,1,
= frE, lzE =E.
35. Sa se c
intervalului:
0
0 0
a) A=A; b) A
' '
AUB.
Sa se dea un e
\lndicatie \
'
XEB
0
b) XE A => 3 I
0

C) XE AnB=>3
d)A = (0,1]; B
multimi;
0
0
nA,cnA,;
' '
36. Sa se
aderentei:
a) A.=X;b}
d) An'Bc'Ar
37. Sa se
frontierei:
a) frA =A r
c) fr(AuB) <;;;
18).
oarecare de
l p o l o ! ~ i e pe X,
fie clasa
Plop.ologiilor ~ i
de pafti ale
xeD implica
R. i) 0, RIE, E, E0,E; ii) (1,2) u(3,4),
(-ex:, 1) u(2,3) u(4,5) u(5, ex:), {1 ,2,3,4,5}, [1 ,2] u(3,4] u{5},
[1,2] u(3,4}, {5}; iii) Q = extQ = lzQ = 0, Q=Q' =frQ = R; iv)
0 - ]] ]] '
E=Q, extE = R\{0,1,-, ... ,- .. }, frE = {0,1,-, ... ,-, ... }, E = {0},
2 n 2 n
E= frE, lzE =E.
35. Sa se demonstreze urmatoarele proprietati ale
intervalului:
0 0 0
01) oo ,............._oo ,....---A--....
0 0
a) A=A; b) AcB ::::> AcB; c) AnB= AnB; d) AUB:::l
Au B.
0
XEB
Sa se dea un exemplu In care incluziunea d) este stricta.
/rndicatie J
0
b) xe A ::::> 3 Ue-r, astfel Inca!, xeUcA Acs > XeUcB ::::>
0
~
c) xe AnB::::>3Ue1:,UcAnB::::>U cA, U cB
d)A = (0, 1]; 8=[1 ,2], adevarata pentru familia oarecare de
multimi;
~ 0 ( 1 ])
(lA. c(IA; Ak = -k.k .
36. Sa se demonstreze urmatoarele proprietati ale
aderen(ei:
a) A= A; b)AcB ::::> AcB; c) AuB= AuB;
d) AnBcAnB; e) QA. cQA., An= [>J. neN*
37. Sa se demonstreze urmatoarele proprietati ale
frontierei:
- ~ - 0 0
a) frA = A nC A; b) frA = AlA = lzA u(A' I A);
"
-
c) fr(AuB) <;; frA ufrB; d) frfrA = frA \ frA; e) frA = frA;
39
f) fr(AnB) <;;: frA ufrB,
A= Qu[O, 1] u[3,4], B = [2,3]
0 0 - -
g) A = A-frA; fr A <;;: frA; h) A = AufrA, fr A <;;:frA.
38. Sa se demonstreze ca derivata unei multimi are
proprietatile:
a) AcB =>A' cB'; b) (A')' c A'; c) (AuB)' =A' uB';
d) (AnB)' c A' nB'
Observatie. Pentru familii infinite de multimi are loc doar:
c') (l)A.}:ol)A.' ,
39. Fie multimea AcR. Sa se arate ca:
a) daca cardA s x o. atunci (CA)' = R ;
b) daca cardA = x o !ii A este inchisa, atunci lzA ;t0
40. Sa .se demonstreze ca urmatoarele propozitii sunt
echivalente: (P
1
) ffE.<T,(P
2
) F = F; (P3) F:::>F'; (P4) F:ofrF
41. Fie X= {a,b,c,d} o multime fermata din patru elemente:
a) Aratati ca-r= {X,0, {b}, {a,b}, {b,c}. {a,b,c}} este o
topologie pe X;
b) Sa se gaseasca clasa multimilor inchise in spatiul
topologic (X, -r);
c) Sa se gaseasca sistemele de vecinatati ale punctelor b
din spatiul topologic (X, -r).
42. Fie (X, -r) un spatiu topologic cu baza numarabila q o
familie de deschise dispuncte doua cate doua. Sa se arate ca q
este numarabila.
llndicatie I
f: q f(G) = min{n!Bn cG}, GE y este o functie bijectiva
=> B = {Bnl nEN} baza numarabila.
40
43. Fie X un
a) Sa
b) Sa
lndica(ie a) :
b) Se arabi
deschisa ilx care il
44. Dreapta
(orice pereche
llndicatie I
X;ty, 0 <r<
UnV=0
sunt
spatiul
bijectiva
43. Fie X un spatiu topologic !?i A,B doua submul\imi ale lui X:
- 0
a) Sa se a rate ca daca AuB = X, atunci Au B = X
- 0
b) Sa se arate ca daca AnB = 0, atunci AnB = 0
- 0
lndicafie a) Se arata ca x ., A implica x E B ;
b) Se arata ca daca XE A, atunci pentru orice multime
deschisa L'l.x care II contine pe x, avem !l.x r:t:. B.
44. Dreapta reala este spatiu topologic separat (Hausdorff):
(orice pereche de puncte distincte au vecinatati disjuncte).
llndicatie I
x., y, 0 < r < lx-yl, U = (x-r, x+r), V = (y-r, y+r),
2
U nV= 0
41
COMPLEMENTE DE TEORIA $1RURILOR
$1 SERIILOR NUMERICE
.
Autor: lector.dr. LIANA MANU-IOSIFECU
8
COMPLEMENTE DE TEORIA
2 A SERIILOR NUMERICE

A. Probleme rezolvate
n' + 1
1. Sa se verifice ca cu termenul general Xn = _ _c_
4n' -1
are lim ita _1_. Sa se determine rangul incepand de Ia care toti
4
termenii difera de _1_ cu mai putin de -
1
-.
4 . 1000
I Solu1ie I
Fie E>O arbitrar. lnegalitatea 1 n' +
1
_ _1_ 1 < E este satisfacuta
4n' -1 4
daca n> Luand N(E) = ]+1, rezulta ca daca
l
n::::N(E), IXn--1 <E.
4
Pentru E = 10-
3
se obtine N(E) = [Jsoo4 ]+1= 18, deci
termenii al caror rang este eel putin 18 difera de cu mai putin de
1
1000
2. Sa se calculeze a) b) lim _t:t_.
n-+"" 2n n->oc 2"
45
I Solutie I
V
vt v l" n! . 1" n 0
om ara a ca tm-= oc, 1ar tm-=
!l-4"' 2" n--.o'<' 2"
a) lntr-adevar
= ... ; =%Gr deci oricare
ar fi a>O, exista un rang N(a) astfel incat pentru orice n;>: N(a) sa
avem an> a, adica >
9
a ::::> n log
9
a ::::>
92 2 2 2 2
log9a -log2 N( ) [log9a -log2]
1
n> ::::> a = +
1og3 -1og2 log3 -log2
b) n
2" (1+1)"
<
1
+n+ n(n-1) + n(n-:-1)(n-2) + ... +
1
2 3!
2n
<--'---
n(n -1)
-
2
- <'E pentru orice n>N(c), cu N(c) = [-
2
-+-
8
]+ 1
n -1 E
3. Sa se studieze convergenta irurilor:
1 1 1
a) Xn = 1+-+-+ ... +-
2
2
3" ll
2
'
b) Yn = ao+a1q+ ... +anq"
I Solutie I
Vom arata ca irurile sunt fundamentale (Yn pentru lql<1),
demonstrand ca, oricare ar fi > 0, exista un rang N(E), astfel incat
pentru orice n ;>: N(E) i orice pEN sa avem IXn+p-Xnl <E.
a) IXn+p-Xnl = ;, + ... + +

+ ... + C + ;, + ... +
1 1
+ ... +
(n+1)' (n+p)'
F I
. d . I 1 1 1 b'
o os1n maJora rea - < = ----, o tmem
n' n(n+1) n+1 n '
46
1 I
1
IXn+p-Xnl < ---+---'
n n+1 n
1 1 1
=----<-
n n+p n
este strict crescator, r
b) Procedand analog
$ lql"+
1
(lan+11 + lan+2llq
lqlp-
1
) = sup{lanl}lql"+
1
Prin urmare, pentru 1
n;>:N() sa avem sup{
1-
pEN in;>: N(s).
4. Fie Xn = (t-
(Xn)n i (Sn)n sunt COl
numar irational.
I Solutie I
Aplicand formt
(
1)" 1
Xn= 1+- = 1+1+-
n 2!
n
intrucat: Xn :> 1 +1 +.!.
2!
aratat mai sus ca Xn
m>n, avem:
deci oricare
orice N(a) sa
9a

2
;,+ .. +n\)/=
I I I I I
IXn+p-Xnl < ----+-----+ ... +--"---
0 n+l n+l n+2 n+p-2 n+p-1 n+p
I I 1 I [I]
=----<-<E pentru n>-. Deci N(E) = -; +1; !}irul
n n+p n E "
este strict crescator, marginit superior de 2.
b) Procedand analog IXn+p-Xnl = lan+1llql"+
1
+ ... + lan+pllq"+PI :S
'> lql"+
1
(1an+11 + lan+2llql + ... + lan+pllqiP-
1
) :S sup {lanl}lqln+
1
(1+1ql + ... +
lqiP-1) = sup{lanl}lq1"+1 Hql' < sup{la,l}lql"'.
Hql 1-lql
Prin urmare, pentru orice E >0,. exista N(E)EN astfel incat pentru
sa avem sup{la,l} lql"' < E , de unde IXn+p-Xnl< E pentru orice
Hql
pEN N(E).
4. Fie Xn = (1 + _!_)" !}i Sn = 1 +_I_+ __I_+ ... + __I_. Sa se a rate ca
n 1! 2! n!
(xn)n !}i (sn)n sunt convergente !}i au aceea!}i limita, care este un
numar ira(ional.
I Solutie I
Aplicand formula binomului lui Newton, rezulta:
Xn= (
1
+_!_)" =
1
+
1
+ __1_ n(n -1) +_I_ n(n -l)(n- 2) + ... =
1
+
1
+__I_ (l-_1_) +
n 2! 11
1
3! n
1
, 2! n
+ ;! (1- !) (1- + ...
k
Cum 0<1-- s 1, kEN, !}irul (xn)n este crescator !}i marginit,
n
intrucat: Xn:::; 1+1+__1_+_1_+ ... + __!_., 3, deci convergent Ia eER. Am
2! 3! n!
aratat mai sus ca Xn s sn. Pe de alta parte, fixand nEN !}i luand
m>n, avem:
47
1 1 ( 1 ) 1 ( 1 ) ( 2 J ( m- 1)
Xm= 1+-+- 1-- + ... +- 1-- 1-- ... 1--- + ... >
1! 2! m m! m m m
>1+-+- 1-- + ... +- 1-- ... 1--- , de unde:
1 I ( l ) 1 ( I ) ( n - l)
1! 2! m n! m m
e=
I I ( 1 ) 1 ( I) ( n -I)
1imx ;o>lim(1+-+- 1-- + ... +- 1-- ... 1--- )=sm,
m-->"' m m.->> 1! 2! m n! m m
deci Xn sn adica lim s = e.
m.->< m
Pentru a arata ca e este irational (transcendent: nu exista
PEO[x] astfellncat P(x) = 0). Fie m > n. Atunci,
I I
0 < Sm - Sn = + +- =
(n+l)! m!
_ I ( 1 1 1 )<_!_ -_!__!_
- n! n+l + (n+1)(n+2) + ... + (n+1)(n+2) ... m - n!
1
__ _1__ n! n
n+ 1
Trecand Ia limita In raport cu m, se deduce ca:
0 < e - _!_ (1 ). Sa presupunem ca e este numar rational, ..
n! n q
undep,qEN*. Cum XnE(2,3), avem eE(2,3) deci q>1. Tn caz
contrar, e = p E N. Din (1), pentru n = q, se obtine 0<..-
q
(1+_!_+ ... +_1_+ ... +_!_) -
1
-, de unde 0< p(q-1)! -(q! +3.+ ... +_9_)
I! 2! n! q!q 1! q!
_!_ < 1, ceea ce este o contradictie caci In membrul stang avem
q
un numar lntreg.
5. Determinati limitele extreme ale
a) Xn = ' _ ; b) Q numerelor rationale)
{
a" dKin=2k . .
b", daca n = 2k +I
48
I Solutie I
a) Daca O<a<l
Daca a= 1<b,
Daca a< b=1,
Daca 1 <a<b,
b)L(Q)=R,
contine o infinitate
lim Q = -ro . Multimea
H<
chiar sa cantina stric
6. a) Grice
b) Un d
daca multimea punc1
I Solutie I
a) Daca x =
n ;::: N(E) avem x-E <
\Xn\ SUp{\X1\ ... \XN(t)
superior.
Observatie.
(adica termenii
lncepand de Ia un a
X-E >0, daca X>0, rE
rezulta imediat afirrr
b)"=>" Cum
\Xn\ M V nEN, dec
M. Prin urmare L(x,:
Cum 1imx,=
arbitrar, fixat, exista
m-1)
----;-- + ... >
unde:
(
n-1)
1---;;;- )= Sm,
n! n
rational, P
q
obtine 0< 1'_-
q
! +1+ ... +_9_)
1! q!
slang avem
rationale)
I Solutie I
a) Daci\ 0<a<b<1, atunci lim a" =limb" = 0 lim x" = 0

Daca a = 1<b, atunci lim x = 1, lirnx = oo
-;;:::;:;- " n ._...., n
Dacaa<b=1,atunci limx =O,Iimx =I
-;;:::;:;- n 0 __,."" n
Daca 1 < a<b, atunci lim x = oo.
n
b) L(Q) = R, caci oricare ar fi x"' R, orice vecinatate a sa
contine o infinitate de puncte rationale. Deci IL':,!Q = oo
lim Q = -J . Muljimea punctelor lim ita ale unui poate fi infinita
...
chiar sa conjina strict multimea termenilor sai.
6. a) Orice convergent in R este margin it.
b) Un de numere reale este marginit daca numai
daca muljimea punctelor limita este marginita.
I Solutie I
a) Daca x = x,, atunci exista N(&)E N astfel in cat pentru
n;:: N(&) avem x-& < Xn < x +& (2) de unde, pentru orice nEN avem:
IXnl $ sup{lx11 ... 1xN(o)-1 1. &+x}. Deci muljimea {IXnl} este marginita
superior.
Observatie. Daca x ;e 0, atunci XXn >0 pentru n ;:: N(&),
(adica termenii sunt numere de semn cu limita
incepand de Ia un anumit rang). intr-adevar, luand & >0 astfel incat
x-s >0, daca x>O, respectiv x+& <0, daca x<O, din inegalitatea (2)
rezulta imediat afirmajia.
b) ":=;." Cum (xn)n este marginit, exista MER: astfel incat
IXnl $ M V n EN, deci daca exista un x ,, x, rezulta 1x ,, 1 $
M. Prin urmare L(xn) este marginita.
Cum limx = minl(xrt) s maxl(xn) = limx , pentru orice & >0
" n->oo n
arbitrar, fixat, exista un rang N(&)EN astfel incat pentru orice nEN,
49
ri?: N(E) sa avem -M-E:<=: Xn :<=: M+E, adica min{xa,x1, XN(l -M-E} :<=:
Xn Xa,X1, .. , XN<l M+E} deci {xn}n este marginit.
7. Sa se demonstreze egalitatea:
limx =infsup{Xn+1, Xn+z, ... }.
n--->"' n neN
I Solutie I
Notam L=limx,, Ln = sup{Xn+1, Xn+z, ... } La= inf{Ln}
n---+oo R<"N
Daca L = oc, cum LEL(xn). exista Xkn --+oc. Deci, pentru orice
nEN avem Ln = oc, de unde La= L = oc.
Daca L < oc, atunci pentru orice E >0 se poate gasi N(E)EN
astfel incat din n ?: N(E) sa rezulte Xn :<=: L +E. Dar E >0 fiind arbitrar,
se obtine inegalitatea Las L. Pede alta parte, oricare ar fi N(E)EN
fixat, pentru n ?: N(E) avem Xn s LN<l de unde rezulta L = l}','lx .. s
LN<l deci L s inf{Ln} =La. L =La.
8. Sa se demonstreze inegalitatile:
lim x,,, :<=: lim;{;;:: s lim;{;;:: s lim x,,, , Xn>O
R->> Xn R-->o<l R_,.'Xl R--+' Xn
I Solutie I
Vom demonstra inegalitatea:
lim;{;;:::<=: lim X,,, . Fie a = lim x.,, .
,_,.oo n-->oo X R-->'-" X
" "
Daca a = oc, inegalitatea este evidenta. Daca a < oc, pentru
orice E >0, exista eel mult un numar fin it de termeni mai mari decat
a+E, deci exista n'EN astfel incat, daca n ?:n', sa avem x,, < a+E.
x,
De aici se obtine Xn +k+1 < (a+E)Xn +k, pentru orice kEN atunci
pentru orice pEN, vom avea Xn +p s (a+E)P Xn Prin urmare Xn
(a+r.)n-n'xn, oricare arfi n ?:n'. Rezulta: {;: s:.Jx,.(a+E)"(a+E), de
unde lLriJ{;: < a+E. Cum E >0 este arbitrar, se obtine: 1}'!!;{;: = a.
50
Observatie. Din
daca exista I = lim x,,,
n-->oo X

este egala cu I.
9. Fie (Xn)n un
limy = oo . Sa se arate
,_ "
( .:s_) este conv
Y. "
Cesaro).
I Solutie I
(
X -X)
Daca " "
Ya+I-Ya
N(E) = n' EN astfel tnc
de unde (Yn)n fiind stric
E
Xn < (x+- )( Yn+1- Yn).
3
Scriind aceste
tnsumand, obtinem: (
relatie pe care tmparti
)(1- Y,) +
3 Y.
Cum n' este fix
Astfel, x-E <
rezulta ca: lir
-;;:;:;;- Y n n-+
-M-E} $
{ln}.
orice
N(E)EN
arbitrar,
fi N(E)E N
pentru
deetH
Observafie. Din de inegaliU\ti de mai sus, rezulta ca
daca exista I = lim x,., , fin ita sau infinita, atunci !i!Il F. exista
n--+> xn
este egala cu I.
9. Fie (xn)n un arbitrar (Yn)n un strict crescator cu
l,i!!! y. = oo . Sa se a rate ca: daca ( x,., -x,J este convergent, atunci
Yn+l -yo n
( este convergent are lim ita. (teorema Stolz-
Y, "
Cesaro).
I Solufie I
Daca ( x,., -x,) XER, atunci, pentru orice E >0, va exista
Yn+l -yo n
N(s) = n' EN astfel in cat daca n ::: n', atunci x - .:. < x,, - x, < x +.:.,
3 Y,., -y, 3
de unde (Yn)n fiind strict crescator, obtinem (x- (Yn+1 - Yn) < Xn+1 -
E
Xn < (x+- )( Yn+1- Yn).
3
Scriind aceste inegalitati pentru n', n'+1, ... n apoi
insumand, obtinem: (Yn - Yn) < Xn - Xn < )(yn - Yn),
relatie pe care impartind-o Ia Yn >0, vom obtine:
(x-.:. >(1- Y,) + < (x+.:. )(1 +
3 Y, Y, Y, 3 Y, Y,
Cum n' este fixat, din Yn rezulta ca
Y" x"
x -x
Astfel, X-E < lim-" s; lim-" < X+E. Cum E este arbitrar,
n=;:;- Y n ' n->oo Y n
51
Fie
X -X
lim n+[ II oo. Atunci, pentru orice M>O va exista n' EN
n-->"' Y n+l - Y n
astfel ca din n sa rezulte Xn+1-Xn >M(Yn+1-Yn). Facand
insumarea ca mai sus, vom Xn -Xn >M(Yn - Yn-l sau
+M(l-2::.,:.) ceea ce ca: M.
Y, Y, Y, Y,
Cum M>O este arbitrar, rezulta ca oc. Cazul limitei -
n--+"' Y n
oc se trateaza analog.
Observatie. Reciprocele teoremei sunt false.
1. Daca (Xn)n, (Yn)n sunt de numere reale pentru care
exista lim x, - x,_, = x E R , atunci (Yn)n nu este
n-+'"' Y n n--+" Y n - Y n-1
neaparat crescator strict nemarginit (xn = Yn = (-1)").
2. Daca (Xn)n, (Yn)n sunt de numere reale,
O<Y1< ... <Yn ... limy = oc, iar x E R, atunci lim x,- x,_, x,
' n-+oo n l\--+' Y n __,.,., Y n - Y n-1
doar daca liEJ ...r,_ = u E R+ \{ 1}.
Y,+l
Tntr-adevar, daca Xn = (-1)", iar Yn = n, 'v'nEN*, atunci
dar nu exista lim x, -x,_,. x, -x, '(1- Y,,J +
n-->"'-Yn n-->"'yn-Yn-1 Yn Yn-Ynl Yn
x,_, . Y,_, , trecand Ia limita dupa n,
Y .. -1 Y ..
X -X
(1-u)x = (1-u) ljEJ " "' .
Y n - Y n-l
10. Fie (xn)n>1 un de numere pozitive. Atunci
I= limn(
1
+ x,_, - 1) 1.
""' X"
52
I Solutie I
Presupunem p1
(
I+ X )
n xn n+l 1 <
(n+1)Xn- n Xn+1 > r
Sumand de Ia
XN XN XN+t
->---
N N N+
Seria I-
1
-
JI"-l N+p
deci I
Observatie.
- (l+x )
limn ,., -I <1+1
II-+"' x"
11. Sa se con
incat L(xn)
cu[0,1].
I Solutie I
Fie An= {XE[O
elemente.
Sa aratam c;
din enw
Tntr-adevar, fi
Deoarece bk- ak =
va exista n'EN
Faeand
Yn) sau
oc. Cazul limitei -
x -x
lim n n-1
B_.,..., Yn- Y n--1
X,
'<fnEN*, atunei
'1-y"_')+
Y.-t Yn
Atunei
I Solutie I
Presupunem prin absurd ea I <1. Atunei :3 N:?:1 astfellneat
n(
1
+x: -1) < 1, Vn:?:N <:::> n(1 +Xn+1 - Xn) < Xn, 'i n:?:N
=> (n+1)Xn- n Xn+1 > n, (V)n:?: N <:::> ~ - x,., >-
1
-, 'in:?: N
n n+l n+l
Sumand de IaN Ia N+P (p :?:1) obtinem:
X,> X,_ X""' >-'-+-'-+ ... +-'- (3).
N N N+p N+l N+2 N+p
Seria L:-
1
- fiind divergenta, relatia (3) este absurda,
p:-:t N +p
deci I :?:1.
Observatie. 'i E >0 exista (Xn)n :<; R: astfel In eat
lim n(
1
+ x,., 1) <1 +E.
..... .., X
"
11. Sa se eonstruiasea un ir (Xn)n>1 de numere reale astfel
!neat multimea L(xn)n>1 a tuturor punetelor sale limita sa fie egala
cu [0, 1].
I Solutie I
Fie An= {XE[O, 1]; x = ; , 0 :<; m :<; 2n}. Evident, An are 2n+1
elemente.
sa aratam ca irUI X
1
,X
2
,XJ'X
4
,X
3
,X
6
,X
7
,XpX
9
,X
10
, . ,X
17
, .. are
~ ~
AI A2 A3
proprietatile din enunt.
Tntr-adevar, fie XE[O, 1] fixat,
I . b
ak = x- -"1 k = x+
2k+l y 2kl
Deoareee bk - ak = -
1
"-i diferenta dintre doi termeni eonseeutivi ai
2" y '
53
Ak este ;, , rezulta ca [ak,bk) nA ;t0; fie x ,, E[ak,bk] nAk
fixat. Evident avem n1 <n2<- .. < nk<- .. "'i IX -XI< bk - ak = -
1
, 'v'k 2':1,
y "k 2k
deci lim x = x.
k->o> "k
12. Sa se stabileasca natura seriei armonice generalizate:
I I I
1 +-+-+ ... +-+ ... ,a ER.
2
1
3a n'
I Solutie I
Daca a ::;; 0, irul termenilor seriei nu converge catre 0, deci
seria este divergenta. Daca a>O, termenii acestei serii formeaza
un ir descrescator de numere pozitive. Din criteriul de
condensare .al lui Cauchy, pentru a>O, seria data este de aceeai
natura cu seria geometrica _3.':._ =
1
. Cum aceasta serie
. ')"
are raia -
1
-, rezulta ca ea este convergenta pentru a > 1 "'i
\ 2 o-> Y
divergenta pentru a :s:1.
Observatie. Pentru a = 1 obtinem seria divergenta
1 + _!:._+ ... +_!:._+ ... , care are proprietatea ca orice termen al ei,
2 3 n
incepand cu al doilea, este medie armonica intre termenii alaturati
(se numete serie armonica). Divergenta ei se poate demonstra
rapid, prin reducere Ia absurd, astfel: presupunand ca seria L _!:._
n;,\ n
este convergenta, fie s suma ei. Atunci avem:
s = I + _!:._ + (_!:._ + _!:._) + (_!:._ + _!:._) + (_!:._ + _!:._) + ... > _!:._ + I + (_!__ + _!:._) + (_!:._ + _!:._) + (_!:._ + _!:._) +
234 56 78 2 44 66 88
+ ... .. )
adica s 2>: _!_+s, absurd.
2
54
13. Sa se stabile
a) I(.Jn+a+l

c) I-n-;

I
d)
l I I
e) 1+---+-+
2 3 4
I Solutie \
a) Notand an =
suma pa!iiala de ran
lims =..Ja -.Ja+l+lin
n--+oc n n-+<
convergenta i are su
b) Suma pa
" k +I "
2:1n -= 2:lln(k+l
k"'l k k<'l
ca seria data este d
acestei serii converg
Se confirma inca o d
convearga catre 0 n1
unei serii.
c) Din criteriul
convergenta a unei l
Ia zero. Cum
n-->"' n + 1
d) Termenul g
I
16n'-8n-3
Amplificand p
fie x,, E[ak,bk] nAk
- ak = _!__ 'v'k >1
2k I -
generalizate:
I)VerQe catre 0, deci
serii formeaza
Din criteriul de
este de
aceasta serie
termen al ei
'
ca seria I_!_
n;,l n
13. Sa se stabileasca natura urmatoarelor serii:
a) a>O;

c) f,-n-;
n + 1
. l
d) -Sn-3;
!1111 I 1 I
e) 1+---+-+---+ ... +--+----+ ...
2 3 4 5 6 3n- 2 3n- 1 3n
\ Solutie \
a) Notand an = '-'n +a- '-'n +a -1 , se demonstreaza ca
suma partiala de rang n este sn = an+1 - a1. De aici, rezulta ca
lims. = -v'a- '-'a+ 1 +lim'-'
1
seria data este
U-->"' n.-.>"1> n+a+l- n+a
convergenta are suma -v'a- '-'a+ 1 .
b) Suma partiala de rang n a seriei este sn =
" k +I
2)n --= I[ln(k+l)-lnk}=ln(n+l). Deoarece \.iEJs. =oo, rezulta
k k-l
ca seria data este divergenta (avand suma +oc). $irul termenilor
acestei serii converge catre 0, cu toate ca seria este divergenta.
Se confirma Inca o data ca cerinta ca termenilor unei serii sa
convearga catre 0 nu este condi\ie suficienta pentru convergen\a
unei serii.
c) Din criteriul lui Cauchy rezulta ca o condi\ie necesara de
convergen\a a unei serii este ca termenilor sai sa convearga
Ia zero. Cum lim _11_ = 1, rezulta ca seria diverge.
_., n +I
d) Termenul general al seriei se descompune astfel:
1 (4)
16n' -Sn-3 4n-3 4n+l
Amplificand pe rand (4) cu 4n-3, respectiv 4n+1, ob\inem:
55
1 4n- 3 . 1 4n + 1
--=A+ B--. respect1v ---=B+A--
4n + 1 4n + 1 4n- 3 4n- 3
Pentru n = obtinem A= _1_ iar pentru n = - _1_ se obt.ine
4' . 4' 4
B=- _1_ , deci
1
_1_(-
1
---
1
-) . Astfel Sn = _1_ (1 - -
1
-) ,
4 16n' -8n-3 4 4n-3 4n+1 4 4n+1
deci seria converge, iar suma acesteia este _1_.
4
e) Vom stabili divergenta seriei cu ajutorul criteriului lui
Cauchy; avem: 1Un+1+Un+z+ ... + Un+pl=
1 1 1 1 1 1 1
= --+-----+--+-----+ ... + +
3n+l 3n+2 3n+3 3n+4 3n+5 3n+6 3n+3p-2
1 1 1 1 p
+ + >--+--+ ... + >
3n+3p-l 3n+3p 3n+3 3n+6 3n+3p 3(n+p)
Luand p =n, obtinem 1Un+1+Un+z+ ... + u znl > Prin urmare,
exista e >0 (e = _1_) astfel incat oricare ar fi mEN, exista n >m i p =
6
n astfel ca lun+1+ ... +un+pl>&, deci seria obtinuta este divergenta,
nesatisfacand condiatia din criteriul general allui Cauchy.
14. Fie "L:U. o serie cu termeni oarecare i (u1+u
2
+ ... +u,,)
+ (u .,., + ... +u ., )+ ... +(u .,_,_, + ... +u ,.)+ ... seria obtinuta din aceasta
printr-o grupare oarecare a termenilor, tara a le schimba ordinea,
In aa fellncat In fiecare grupa sa intre termeni de acelai semn.
Sa se arate ca daca aceasta serie este convergenta, atunci
converge i seria initiala i are aceeai suma.
I Solutie I
Fie S sumele partiale ale seriei initiale i S sumele
partiale ale seriei convergente asociate, cu suma S". Cum
56
V!11 S = S", pent1
'
> - I c
nk_n ,, , sa avem ,
Din criteriul
- 1 < e
!<U k + p
daca nk nk+1,
. Daca n> n,,
'
pentru care -S'
= e. De aici
Observatie
lui Leibniz pentru :
termeni. Pentru s1
trei termeni poziti'
rezulta ca seria e
poate demonstra
ale seriei
""
15. a) Sas
I I I
1--+---
2 3 4
a) Sa se!
+ _]_In
2 2
I Solutie I
Seria arn
convergenta (Lei
I (
1
__ 1 )
4 4n +I '
criteriului lui
... +u "')
semn.
at unci
sumele
= S", pentru orice E >0, exista n,, EN astfel Inca!, daca
k-->., k
> " "
nk_n ,, , sa avem I S n, -S I <
2
.
Din criteriul de .convergenta general al lui Cauchy, rezulta
- I < E 'v'pEN, iar din modul de grupare al termenilor,
kO k + p
k k +I 1:: +I
"
S
0
. Daca n> n, , va exista k astfel Inca! nk<n:o;nk+1, nk <! n ,
. -
'
" II' E"" EE
pentru care ISn -S"I $/Sn -S"I +I sn -S n I< -+ISn - s n I< -+-
k k 2 k k+i 2 2
= E. De aici rezulta concluzia.
Observatie. De aici rezulta posibilitatea aplicarii criteriului
lui Leibniz pentru serii In care alterneaza semnele unor grupe de
termeni. Pentru seria Ic-liflln(a+n), a>O, alterneaza grupe de
n
!rei tenmeni pozitivi l?i !rei negativi. Cum In ( + 1) ---+0 descrescator,
rezulta ca seria este convergenta; acelal?i lucru (convergenta) se
poate demonstra aplicand direct criteriul lui Abel (sumele partiale
ale seriei Ic -l)[f] sunt marginite l?i In (a+") ---+0) .
11;..1 n
15. a) Sa se gaseasca suma seriei armonice alternate:
I I I I I
1--+---+ ... ...
2 3 4 2n -I 2n
a) Sa se gaseasca o comutata a acestei serii cu suma ln2
I p
+ -ln-.
2 2
I Solutie I
Seria armonica alternata Un = I (-I)"-' este evident
n
convergenta (Leibniz).
57
Noland Sn = 1+ _!_+_!_+ ... +__!_,evident Un = S2n- z(_!_s) = S2n-
2 3 n 2 "
. 111 1 I I 1 I
sn. ad1ca 1--+---+ ... +----=--+--+ ... +- (identitatea
2 3 4 2n + 1 2n n + 1 n + 2 2n
lui Catalan).
( I)"' ' d
Astfel "L---= J_2_=ln2.
Ilk! n ol+x
Observatie. Suma seriei armonice alternate poate fi
calculata cu ajutorul constantei lui Euler (c = a,577216 ... ),
demonstrand ca Sn = c+ln n +En, En (5).
Cum (1+_!_)' <e< (I+_!_)''', pEN, rezulta -
1
-< ln(l+_l_) < _!_
p p p+1 p p
(logaritmul fiind functie crescatoare).
Sumand inegalitatile obtinute pentru p = 1,2, ... ,n-1,
obtinem:
1 I . 1 1 1
Inn< 1+ -+ ... +--, dec1 a< -<1+ -+ ... +--Inn.
.2 n-1 n 2 n
$irul Yn = 1 + _!_ + ... + _1_ -In n este convergent (minora! de c i
2 n
descrescator: Yn+1-Yn = -
1
--ln(n+1)+lnn=-
1
--ln(l+_!_)<a),
n+l n+1 n
Yn=C, iar En = Yn-C
Cum Un= S2n -Sn. se obtine lim Un = lim (ln2n - In n +E2n -En)
= ln2.
1\--J.oo n-+oo
fnlocuind in (5) pe n cu 2n, obtinem: S2n = c + ln2n +t::2n
fnmultind (5) cu _!_, rezulta: _1:_+_1:_+ ... +-
1

' 2 2 4 2n 2 2
Scazand relatiile de mai sus obtinem:
1 I 1 c ,-;-- &"
1+-+-+ ... +& --.
3 5 2n -1 2 '" 2
b) Comutand termenii seriei armonice alternate astfel ca
dupa un grup de p termeni pozitivi, in ordine descrescatoare sa
urmeze un grup de q termeni negativi i descrescatori in valoare
absoluta, obtinem seria:
58
1 I I I
1+-+ ... +------
3 2p -1 2 4
1
... --+ ...
2nq
Conform exe
obtinuta mai sus es
i numai daca seria:
(
1+_1:_+ ... +-1 )-(!:.
3 2p-l 2
(
I !
- 2nq -2q +2 + ... + 2t
este convergenta
partiale ale acestei
I I
0"2n = 1+-+ ... +--
3 2p-l
0"2n = + In .J4;;p + 1
2
&
lln= E2n-2, de und1
Deoarece a
rezulta lim cr2n-1 =

catre ln2 +_!_In
2 q
lim ita).
Aplicatie:
alternate cu suma
16. a) Sa '
pentru care irul te
b) Exista
condi\iile criteriului
1
(identitatea
1 I I I I
1+-+ ... +-------... ---+ ... + + ... +-_c_-
3 2p-J 2 4 2q 2pn-2p+l 2pn-J 2nq-2q+2
1
... --+ ...
2nq
Conform exercitiului anterior (problema rezolvata 14) seria
obtinuta mai sus este convergenta i are sum a ln2 + daca
i numai daca seria:
(
1+.!.+ ... +-1 J -(_1_+_1_+ ... +-1 )+ ... +( I + ... + 1 J _
3 2p -1 2 4 2q 2pn- 2p +I 2pn -I
-Cnq -
1
2q +2 + ... + ...
este convergenta i are aceeai suma. Noland (crn)n irul sumelor
partiale ale acestei serii, avem:
I 1 1 I( 1 I 1) .
cr2n = 1+-+ ... +--+ ... + - 1+-+ ... +-+ ... +- , obtmem
3 2p -1 2np -1 2 2 q nq '
cr2n + 1"J _ _l_(c+ In n2+ E ) = ln2+_1_ln!'.+ 1'1
2 "'2 . "' 2 2 ""2'
unde
l'Jn= E2n_.'S,_, de unde lim cr2n =In 2 + _1_1n!'..
2 H" 2 q
Deoarece cr2n-1 = cr2n + (
1
+
1
+ ... +-
1
-) ,
2nq- 2q + 2 2nq- 2q + 4 2nq
rezulta cr2n-1 = 1),! cr2n . Prin urmare, irul (crn)n este convergent
catre ln2 +_1_1n!'. (subirurile de paritati diferite avand acceai
2 q
lim ita).
Aplicatie: Sa se gaseasca o comutata a seriei armonice
alternate cu suma In
2
.
2
16. a) Sa se dea exemplu de serie alternata divergenta
pentru care irul termenilor converge Ia zero;
b) Exista serii alternate convergente care nu satisfac
conditiile criteriului lui Leibniz?
59
I Solu1ie \

a) Fie . Cum lim Uzn = 0 = lim Uzn+1, rezulta
nsl n O-+"' n .. ""
ca irul modulelor termenilor acestei serii converge Ia 0, dar irul
I rt
. I t d. - . 1 3 1 1 3
sume or pa 1a e es e tvergent, cac1: Szn =- ...
. 2 3 2n 1 2n
= +..!._+ ... +..!._) +..!._+ ... +-
1
-) = +Inn+ E ) ln.J4n" 11
2 2 n 3 2n 1 2 " 2 "
Aadar, conditia ca irul modulelor termenilor unei serii alternata
sa convearga Ia zero nu este suficienta pentru convergenta seriei,
ci devine astfel numai impreuna cu conditia ca acest ir sa fie
descrescator (C.Leibniz).
b) Cu toate acestea, conditiile criteriului lui Leibniz sunt
numai suficiente, nu i necesare pentru convergenta seriilor
alternate. Vom stabili existenta seriilor alternate convergente
cu !Jn 2': 0, pentru care irul (un)n nu este descrescator.
n=!
Notand Vn
=
{
u . , , daca n = 2k
u "'' , dacii n = 2k + 1 '
unde
satisface criteriul, seria I; v" este evident alternata, dar irul (lvn\)n
n=l
tinde Ia zero, insa nu descrescator. Noland cu (sn)n irul sumelor .
partiale al seriei respectiv cu (crn)n pe eel al I;v .. i
n=l
. {s , pentru n = 2k
luand so = 0, avem crn = " , de unde rezulta ca
s._, , pentru n = 2k + 1
I}':: crn = l}EJ Sn, deci seria I; v, este convergenta.
n=l
60
17. Sa se studie:
a) .
n;;,l n
b)
""1 n(n ...
c)
nl!l n-
lsolu1ie I
a) Pentru orice
2iai a
n> -, tg - are serr
n n
seriei date, obtinem c
initiala, care satisface
irul (ltg \)n este d
n
(convergenta nefiind
b) Seria este '
Pentru calculul sum<
. I I
s1mpe =
n(n + l)(n + 2)
La tel ca Ia d'
succesiv cu n, n + 1, 1
n = - 2 A= ..!._ = C "i B
2 y
Astfel, --
n(n+l
sumelor partiale ale a
cr ... +-
1
-
'" 2 2 2n -I
I 1 ]
=
2n+l 2n+2
lim u2n+1, rezulta
-
Ia 0, dar
1 3
---+-
2n-l 2n
L(-l)"''u,
...
sumelor
rezulta ca
17. Sa se studieze convergenta seriilor alternate:
a) aER;
nd n
b) L(-1)"'' I ;
.,, n(n + 1)(n + 2)
c) I;C-l)"'' 2" sin'" x.
n;>:\ n + 1
I Solutie I
a) Pentru orice n natural satisfacand rela\ia < , adica
n>
2
1al, tg are semnul lui a. inlaturand primii [
2
1al] termeni ai
x n n
seriei date, obtinem o serie alternata, de natura cu cea
ini\iala, care satisface criteriul lui Leibniz, 1ntrucat lim tg =0, iar
n-ooo n
irul (ltg l)n este descrescator. Seria este deci convergenta
n
(convergenta nefiind absoluta) .
b) Seria este convergenta satisfacand criteriul lui Leibniz.
Pentru calculul sumei, se descompunerea In fractii
. I I A B C \.1 N*
Slmpe vnE .
n(n+l)(n+2) n n+l n+2
La fel ca Ia determinarea coeficientilor In (4), amplificand
succesiv cu n, n + 1, n + 2, obtinem, pentru n = 0, n= - 1, respectiv
n = - 2, A= .!_ = C B = - 1.
2
Astfel,
1
.!_(.!_--
2
-+-
1
-). Fie (crn)n
n(n+l)(n+2) 2 n n+1 n+2
sumelor partiale ale acestei serii. Atunci:
a, ... +-1 ___ 1 +(-.!_+.!__.!_+ ... +-!-) +.!__.!_+.
" 2 2 2n- I 2n 2 3 4 2n +I 3 4
+-1 ___ -2-1+.!_+_3 ___ 1_)
2n +I 2n + 2 2 '" 2 2n + 1 2n + 2 '
61
unde (sn)n desemneaza irul sumelor par(iale ale seriei armonice
alternate.
Astfel, lim cr2n = 2Iim S2n - i, cum seria data este
4
convergenta, rezulta ca irul (crn)n are aceeai limita, deci suma
seriei este 21n2
4
c) Aplicand corolarul criteriului radacini, obtinem:
I
. ViuJI_
1
. 2" sin'" x
1
. 2sin' x zsn2x
lffi"U-lffi" -un I,
II-+"'> X D--+00 n + 1 11-T"' rn+l
adica absolut
convergenta seriei pentru 2sin
2
x < 1 =>
cos2x > 0 =>
XE 1t).
hZ 4 4
Pentru cos 2x <0, seria este divergenta, iar pentru x=kn
4
seria devine "(-Iy, care, in baza criteriului lui Leibniz este
L... n+l
convergenta. Seria modulelor este insa divergenta (seria armonica
din care lipsete primul termen). Aadar, pentru x = kn seria
4
este semiconvergenta.
Observatie. Ca i criteriul lui Leibniz, criteriul lui Abel da
conditii suficiente (nu i necesare) de convergenta a seriilor.
. !Ill I I
Exemplu: y--2-+5-"-4--... - (
2
n)' + (
2
n+l)' converge pentru orice
a>O (dei irul (an)n nu este monoton).
18. a) 0 serie cu termeni pozitivi L;x" este divergenta =>
""
3 (an)n. un ir strict crescator, divergent, de numere pozitive astfel
incat Xn = aH, -I , 'd n ::0: 1.
a"
62
b) 0 serie c:
3 (an)n. un ir strict
incat Xn = a"" -a, .
anan+l
I Solutie I
b)"=>" Daca
ordin n, rn = L;x, e:
il:ll
'<I n ::o: 1. Evident an :
"<=" Daca o
'<I n::o: 1, atunci averr
urmare, seria L;x.
""
a) "=>" PresL
ir definit astfel: a1
avem: an+1 = (1+)
'<I n ::0: 1 , deci an+1 =
Din inegalit<
deducem an+1 ::0: (11
"<=" Fie (an)r
pozitive, astfel incal
seriei armonice
seria data este
limita, deci suma
adica
obtinem:
absolut
cos2x > 0 =>
= krr _::, seria
. 4
lui Abel da
a seriilor.
divergenta =>
pozitive astfel
b) 0 serie cu termenii pozitivi L>" este convergenta ::>
""
3 (an)n, un strict crescator, divergent, de numere pozitive astfel
incat Xn = a".' -a" .
ananfl
/ Solutie /
b) "::::}" Daca .L;x" este convergenta, atunci restul seriei de

ordin n, rn = .L;x, este descrescator !?i convergent Ia 0. Fie an= _!_,

\f n 1. Evident an /lac -
1
--
1
-= rn- rn+1 = Xn, V 1.
an all+!
":::" Daca 0 < an < an+1 V nEN* an /lac, iar Xn =
" "(I I) I I I
.L;x,=.L; --- =----+-(n-Ho).Prin
al ai+t a, an+! a,
urmare, seria .L;x" este convergenta.
a) "::::}" Presupunem seria .L;x" divergenta fie (an)n,
1
un
""
definit astfel: a1 >0 arbitrar an+
1
= (1 +xn)an, V n 2. Atunci
avem: an+1 = (1 +xn)(1+xn-1)an-1 = ". = (1 +xn)(1 +xn-1) ... (1+x1)a1
V n 1, deci an+1 = (1+xn)an >an, V n 1.
Din inegalitatea lui Bernoulli (:t,o+a,):::: I+ :t,a, Ita,> 1)
deducem an+1 (1 + t,:x.)a,, deci an+1 = ac.
":::" Fie (an)n>1 un strict crescator divergent de numere
pozitive, astfel Tncat, Xn = a".' -I V n 1. Atunci V 1 n < m avem:
a"
Ill m a -a_ 1 m a
.L;x, = L ,., '>-.L;(a,., -a.)=i--"
1=11 111 ai am+! l=ll an!
63
Cum am 71oc, pentru n fixat, exista m > n astfel incat
< , deci ! x, >_I_ conform criteriului general al lui Cauchy,
am+l "-' 2
seria Lx,. nu este convergenta.
,.,
19. Folosind criteriile de comparatie pentru serii cu termeni
pozitivi, stabiliti natura urmatoarelor serii:
. I
a) L( )"";
,.., In Inn
d) "-
1
-,a>-1;

r>. ' I
\,9L ,a>O;
, ,, n(l + a+ ... +a")
X
f) an = 1- cos-, n ;:>: 1.
II
I Solutie I
a) Cum (In In n)ln n = eln n lnlnln n = (eln n)lnlnln n = nlnlnln n lnlnln
' 2 2
e'' = lnln e'' = lne = 2; pentru n > e'' avem n lnlnln n > n
2
deci
I I 1 I -, b . I . .t . d .
--- n n = --<- . n aza pnmu Ul en enu e comparat1e
(lnlnn) n'"""" n' ' '
deducem ca seria data este convergenta, caci seria cu care o
comparam este seria armonica generalizata cu a = 2> 1.
b) Tntrucat "./Inn <Vn, V nEN*, rezulta: > Vn ;:>: 2.
\Jlnn Vn
Tnsa =1, primul criteriu de comparatie conducand Ia
64
divergenta seriei date, '
sus, divergenta.
c) Deoarece
criteriul lui d'Aiambel
convergenta, iar daca
avem u,., = 2- 'ife > 2
u,
orice n ;:>: 2 (deoarece
Tn baza celui
divergenta seriei (seri
d) Daca a > '
seria geometrica de
primului criteriu de cc
Daca -1 <a
a = 1, seria fiind cea
pentru 1a1 <1, compa
conduce Ia rezultat1
1
,. n
a +n =-- lim
1 an+n R--><X>
n
e) Pentru a>
data este converge
seria de compara1
convergenta.
!neat divergenta seriei date, seria L fiind, conform celor aratate mai
vn
termeni
lnlnln
deci
Ia
sus, divergenta.
c) Deoarece
criteriul lui d'Aiambert deducem ca daca a < 1, seria este
convergenta, iar daca a < 1 este divergenta. Pentru cazul a = 1,
I
( )
"''
avem :: = 2- :Je > 2- 1 +
= 2 1 I n-1 n
-
pentru
n-1
(
!)"''
orice n <! 2 (deoarece e < I+-;;- , 'It nEN).
fn baza celui de-al doilea criteriu de comparatie, rezulta
divergenta seriei (seria cu care am comparat fiind cea armonica).
d) Daca a> 1, -
1
-<-
1
, VnEN. Cum este
a"+n an
seria geometrica de ratie .!. <1, convergenta, rezulta, In baza
a
primului criteriu de comparatie, convergenta seriei In acest caz.
Daca -1 < a s 1, seria este divergenta; In cazul In care
a = 1, seria fiind cea armonica din care primul termen, iar
pentru Jal <1, compararea cu aceasta serie (criteriul al treilea) va
conduce Ia rezultatul anuntat. Astfel, Jal <1 :::::> Jal" = 0 :::::>
I
= _n_ lim _n_ = lim -
1
- = 1, deci concluzia.
1 a" +n n-->oo n+a" n-->o<> a"
1+--
n
n
e) Pentru a >1, avem
1
<-
1
, VnEN. Astfel, seria
n(l+a+ ... a") a"
data este convergenta (In baza primului criteriu de comparatie},
seria de comparatie fiind cea geometrica de ratie
1
deci
a
convergenta.
65
Pentru a = 1, seria data devine
" I
2: .
""' n(n +I)
Cum
1
rezulta i in acest caz convergenta.
n(n+l) n
Pentru a <1, vom aplica criteriul al treilea de comparatie,
folosind ca serie de comparatie pe cea armonica, aceasta fiind
divergenta. Avem lim
1
- lim
1
lim
1
- a =
n..;..., n(l+a+ ... +a") n->"' l+a+ ... +a" n-) 1-an+l

I
n
=1-a; rezulta ca seria data este divergenta.
X . 2 X ( X)'
sm - -
f) C
n 20
2n x' b I .
um lim -..,..-"-- = 2lim ---=- m aza ce Ul
:, H (;)' :, 2 O
de-al treilea criteriu de comparatie, seria este convergenta.
20. Sa se arate ca exista atat serii divergente, cat i serii
convergente L u,
tf
-u
cu termeni pozitivi, as el incat lim--"'.!.>1 i
u.
1 Solutie I
S
. I I '
1
2" t d . t -
ena -+2+-+2 + ... +-+ + ... es e e aceeal na ura cu
2 2' 2"
IH-+ 2") . Avem u, .. , =
2
,
1
,,, i = 2'" , deci lim u,., = ex: >1 i
O'>[ 2 u2n u2n-J un
lim u ""'- = 0 < 1.
u,
Cum -
1
+2" > 2", 'linEN, seria considerata este divergenta.
2"
66
S
. 1 I
ena -+-+
2 3
Cum
2 3 .
criteriu de comp;
convergenta (seria '
deci convergenta).
Pentru aceal
-u .
lim--"'.!. = ex: > 1 I
0-><>0 un
divergente,
n;.l 2
proprietatile din er
avand termenul ge
partiale, avand tem
Sn = (.!.+_!_) +(J..+-
2 3 2' :
1-(.!.)" 1-(_!_'
I 2 I 3-
+---
2 1-_1_ 3
2 3
convergent.
Suma seriei
Observatie.
asupra convergent
. . 1 Jf#v
sene1: - - -
2' 3' 2
2
'
ilustrand faptul ca
Cauchy este mai be
I
L . Cum
,., n(n+l)
de comparatie,
;!10n1ica, aceasta fiind
---= lim 1-a =
n-->"' l-an+l
. '
Tn baza celui
2
-u
lim----"-'-> 1 i
u,
aceeai natura cu
lim u.,, = oc >1 i
....... UD
este divergenta.
S
.llll ]] td
ena -+-+-+-+ ... +-+-+ ... es e e aceeal na ura cu
2 3 2' 3' 2" 3"
"( 1 1 ) C I I I l l N f I.
L... -+-. um -+-<-+-=-,'linE ; conorm pnmuu1
n?l 2n 3n 21\ 3n 21\ 2n 2n-l
criteriu de comparatie, rezulta ca seria considerata este
convergenta (seria de comparatie fiind cea geometrica de ratie
deci convergenta).
P
. u I ( 3)" .
entru aceas a sene avem ----""'- =- - I
u,. 2 2
- u 1 . u 0 1 p . . tt ..
lim-"-"-- = oc > I lim-"-"--= < . nn urmare, ex1s a a a seru
n-+oo U n-+< U
. "
divergente, cat i serii convergente cu
n21 2 n;>l 2 3
proprietatile din enunt. Pentru seria convergenta de mai sus,
avand termenul general Un = -
1
+_!_, rezulta ca irul sumelor
2" 3"
partiale, avand termenul general:
s = (_!_+.!_)+(-1 +_!_)+ ... +(-! +_!_) =_!_+_!-+ ... +-! +_!_+_!_+ ... +_!_=
n 2 3 2' 3' 2" 3" 2 2' 2' 3 3' 3"
I
2
1
_ _!_ +3
1
_ este
2 3
convergent.
Suma seriei este: lim Sn =
n-->oo 2
Observatie. Criteriul lui D'Aiambert nu precizeaza nimic
asupra convergentei acestei serii, dar irul (F.), corespunzator
-1. F, I 1 I sene1: - -'- - ... '- '- are 1m u =-<1, exempu
2' 3' 22 ' 32 ' ' zn ' 3n n--><><' n .J2
ilustrand faptul ca (vezi problema rezolvata nr.8), criteriul lui
Cauchy este mai bogat In informatii decat eel allui D'Aiambert.
67
21. Studiati natura seriilor:
a) a>O
.i..J ' I I
11"1 n.
b) L(n:
1
r' -a", a>O;
c) I;a'"" , a>O.
""'
I Solutie I
a) I
. u ' I' a"''(n +I)"" n! I' (1 1)'- A tf I
tm -"'-= tm = tma +- - ae. s e'
IH> un ll-H) (n + 1)! allnll n-+oo n
conform criteriului lui D'Aiambert, seria converge pentru a< _!_
e
diverge pentru a >_I_. Pentru a= _!_, avem:
e e
(
])" '( ])"
1+- 1+--
u,,, = n > n = _
1
_ = _n_ = n + 1 de unde, in baza
(
I+ '"' I + 1_ n +I 1_ '
u
e
0
n n
celui de-al doilea criteriu al comparatiei, rezulta divergenta.
b) l,it;l.Ju': a( ae. Conform criteriului lui Cauchy,
rezulta ca seria data este convergenta pentru a<_!_ divergenta
e
pentru a > _!_ . Pentru a = _!_ , seria devine 't(l + .!..) '
e e "'""[ ne
(
I)"''
lnegalitatea utilizata Ia punctul a), e< I+; VnEN
(
1)"'
, I+--
.
1
. " ( I\" 1 n
rmp rca: 1+;) ;.-> (
1
)""
I+-
n
1 ( I) ' 1
(
, de unde lim 1+-
I) - n e"
I+-
n
68
lim--'--
I D .
-. ecr
H ( 1 )"
1+-
n
e
I
I .
cazu a = - , sena re
e
c)
n-+> U
"''
Ina-lim In
1
= lr
H ( I)'
1+-
n
adidi a < _!_ , seria
e
divergenta.
I
Pentru a= -
e
armonica, deci diver
Observatie.
raportului nu poate 1
22. Sa se stu
a) 1 +a+ at
b) Un = 2 ,_,,._,
c) 1+ I (2n-
- (2r
I Solutie I
Vom aplica
considerate.
rapoart
ae. Astfel,
Tn baza
divergenta
lim
1
= _1_. Deci l?irul termenilor seriei nu converge Ia zero Tn
e
.
n
cazul a = _1_ , seria rezultand a fi divergenta.
e
(
u ) ( tn...!!... ) a rn,:
1
-1 ( n ) "
c) limn -" -1 =limn a"" -I = lim--ln - =
n-+"' u "-+"" , __'"'" 0 n + 1
..... 1 In --
n+l
Ina-lim In
1
( ')"
I
Ina . = - Ina. Prin urmare, daca - In a > 1,
e

n
d
. - I
a tea a <
e
I
seria data este convergenta, iar daca a este
e
divergenta.
Pentru a = a
1
" " = ( _1_ )
1
" " = ..!.. , seria este tocmai seria
c
armonica, deci divergenta.
e n
Observatie. Asupra seriei de Ia punctul c), criteriul
raportului nu poate preciza nimic, Tntrucat lim u.,., = 1.
,__,., U n
22. Sa se studieze natura seriilor:
a) 1 +a+ ab + a
2
b + a
2
b
2
+ ... + a"b"-
1
+ a"b" + ... (0< a<b);
b) Un = 2<->"-"
C)
1
1+ L.
"0' (2n)!! 2n+l
I Solutie I
Vom aplica criteriul raportului pentru fiecare din seriile
considerate.
$irul rapoartelor ( :"' J va fi:
" "
69
)
{
a , n = 2k + 1 . b) {_1_ n = 2k . ) . u . (2n- 1)'
a ' 8 , ' c hm--""- = hm __c.__ _ _!__
b , n = 2k
2
n =
2
k +
1
>H u. " 2n(2n +I)
,
1
Astfel:
a) Cum lim u""-= b lim (u) = a, daca a>1, seria este
n-+oc un n-ul un n
divergenta, iar daca b < 1 obtinem convergenta. Criteriul nu da nici
un raspuns In cazul a < 1 < b.
Utilizand criteriul radacinii, ,.;-;;: = n =
2
k
II lll\lallbll 'n =2k+l
are {;;:=...lab . Atunci, daca ab <1 seria este convergenta, iar
daca ab >1 seria este divergenta. Daca ab = 1, adica b = _1_, se
a
obtine seria divergenta 1 + a + 1 + a + 1 + a ...
b) Multimea punctelor limita a rapoartelor este
L( :j = H+ deci I f1 L .,l1. Din nou, criteriul lui D'Aiambert
nu da nici o informatie asupra naturii seriei. Tn schimb, criteriul
radical al lui Cauchy convergenta seriei, lntrucat
= 2'_,. <1; se poate folosi criteriul I al comparatiei.
c) Nici In acest caz, criteriul lui D'Aiambert nu precizeaza
nimic asupra naturii seriei (I = 1). Tn schimb, convergenta se
cu ajutorul criteriului Raabe- Duhamel:
aER;
n(2n(2n+1)
1
) =n 6n-l
(2n-l)' 2n-l 2
23. a) Cerceta\i natura seriei t[a(a+ n) ... (a+nr-r)]", a,b,r >0,
. , b(b+r) ... (b+nr-r)
b) Studiati convergenta absoluta semiconvergenta seriei
ta(a-l) ... (a-n+l) aER\N.
n!
70
\ Solutie \
Seria a) este
vom aplica criteriullu
a)
u ...
a calcula aceasta lim
(
r+bx)"
1
f(x) = r+ax In
X
. . (r+bx)
hmf(x)=hma --
x_,.o x-.1} r +ax
I
. (r+bx)'-
= tm--
x .... o r+ax
Rezulta ca: lirr
seriei daca r < a(b-a
b) Avem
lum+l
deci pentru r
ln-al
penl
convergenta.
Pentru a= 0,
absoluta are loc Va.
Daca a.<O,
unde an = (-aX!-
alternata.
a,
1
a> 1, seria este
nuda nici
,
I
adica b = -, se
a
este
I Solutie I
Seria a) este o serie cu termeni pozitivi. Pentru ambele,
vom aplica criteriullui Raabe- Duhamel. Obtinem:
a) Jimnl(nr +b)" -I Pentru
0--HO u n-H'l nr + a n-HO 1
O+i -
n
a calcula aceasta limita, vom determina limita functiei f:R:
f(x)
__ ( 71
in origine, cu ajutorul regulii lui L'Hospital. Astfel,
X
I
. f( ) _
1
. (r + bx)"' b(r +ax)- a(r + bx)
1m x- tma --
HO X->0 r+ax (r+ax)2
lim(r + bx)"' ar(b- a)'= a (b _a)
H" r +ax (r +ax) r
Rezulta ca: lim j .!.) =a (b-a), deci convergenta
n_...., un-1 n r
seriei daca r < a(b-a) l}i divergenta daca a(b-a) > r.
b) Avem la(a-l) ... (a-n+l)l (n+l)! =
lu,..J n! Ja(a-l) ... (a-n+l)(a-n)l
_E_:_!_, deci pentru n >a obtinem: lim n(.J.t!.J_-1) =lim n(a +I)- a+1.
Jn-al lu,.,l n-a
Al}adar, pentru a > 0 (a+1>1), seria data este absolut
convergenta.
Pentru a = 0, toti termenii seriei sunt nuli, deci convergenta
absoluta are loc Va :::0: 0.
Daca a<O, a(a-l) ... (a-n+l) =(-!)" (-a)(l-a) ... (n-a-1)'
n! n!
unde an = (-a)(J-a) ... (n-a-l) >0 deci seria data este o serie
n! ,
alternata. Cum a,., l}irul (an)n este l}ir crescator de numere
a, n +I
71
pozitive, daca a :o:: -1, deci are lim ita nenula, iar seria este
divergenta. Pentru a E (-1 ,0), seria satisface criteriul lui Leibniz,
fiind, prin urmare, semiconvergenta (seria modulelor fiind
divergenta).
B. Probleme propuse
1. Determinati. pentru orice >0, rangul cu proprietatea
ca, pentru orice n ::>: avem, IXn-01 < daca:
( I)"''
a) Xn = ----;
n
b) Xn = -
2
-;
n'+l
23" +(-3)"
C) Xn = ---'-'-"----
4"
R. a) [B+l b) [

]+ 1; o) [::!]+ 1
2. Folosind definitia limitei unui aratati ca:
) I
. 4n+l 4. b)
1
. n'+l
2
. )
1
. n'+2
a tm--;;t; I c Im--=00.
"_,..., Sn- 1 5 n ..... "' n
2
-1 "--."' n + 1
3. Sa se arate, plecand de Ia definitie, ca: = 1.
4. Folosind criteriul lui Cauchy, demonstrati convergenta


a) Xn = L...-,-;
2
b)
cosx cos2x cosnx
Yn = --+--+ ... +--
3 3' 3" '
cos!! cos2! cosn!
C) Zn = --+--+ ... + .
12 23 n(n+l)
72
5. Fie {Xn)n>l
(x
2
n) n>l,
are limita.
6. Fie {Xn)n Ul
este cuprins lntre Xn
llndicatie I
Fie I
'
intervc
iar nln = {x}.
7. Fie (an).,
1
Atunci lim a exista
n-->> n
'if m,n ::>: n
0
.
llndicatie
" , Ia!
=> z:=-, se
2
este fundamental.
8. Fie {Xn)n>
1
L
Xn+m $ Xn +X 1-J n I m, v ,
. fx
Ill-
n;:,l n
9. Daca lim x,:
lndicaveJ
Se poate demc
seria este
lui Leibniz,
fiind
cu proprietatea
=I.
convergenta
5. Fie (xn)n>
1
un !?ir de numere reale cu proprietatea ca
SUb!?irurile (Xzn) n>1' (X2n+1) n>1 !?i (X3n) n>1 au limita. Aratati ca (Xn)n
are limita.
6. Fie (Xn)n un !?ir cu proprietatile 1 lim (xn-Xn-1) = 0; 2 Xn+1
. "-"'"'
este cuprins intre Xn !?i Xn-1. V n>1. Atunci (xn)nz1 este convergent.
\lndicatie \
Fie I,, intervalul de capete Xn-1 !?i Xn l1 :::> lz => ... :::> In ... ,
iar nln = {x}.
7. Fie (an)n>1 cR astfel incat
Atunci exista 3 8 >0, 3no
'if m,n :o:: no.
b = lima' exista !?i bER*.
n--+> n
:0::1 astfel incat lan+aml >8,
lndicatie
E:= 1;1 , se folose!?te definitia; "<=": demonstram ca (an)n
este fundamental.
8. Fie (xn)n>
1
un !?ir de numere reale care satisface conditia
Xn+m :::; Xn +xm, V n,m >1. Atunci exista !?i este egala cu
n--+"' n
. fx
m -'
n
9. Daca lim x, = x, atunci lXI :::; sup IXnl
n -H:> n>l
\lndicatie \
Se poate demonstra prin reducere Ia absurd.
73
10. Fie (xn)n<1 un de numere pozitive. Atunci
-
1
.-(J +X"'')">
1m _e.
n-HO xn
\lndicatie
Problema rezolvata nr.10.
11. Fie (Xn)n<1 un de numere reale cu proprietatea ca
exista 1"-1 < 1, I. E R astfel incat: ljE,!(xH,- A.x") =0. Atunci (Xn)n este
convergent are limita 0.
12. Fie (xn)n<1 un convergent cu elemente din R*. Atunci
(Xn)n>1 are lim ita nenula <=> inf IXnl >0.
""
13. Demonstrati convergenta
a) ... ;
b) X1 = Xn +1 = F-e, a E(0,1);
" I
c) Yn =
d)
Inn!
Zn= --.
nlnn
R: a) an+
1
= , lim a =
1
+ .J5 ; b) de paritati
n n->oo n 2 '
d
t 't . I' 't" .J5 -I ) [ 47]
1 en e au 1m1 a, --; c YnE 1,-
2 36
crescator; d) se aplica Stolz-Cesaro; lim z =1.
n_,.oc n
14. Determinati limitele extreme ale
este moncton
. n7t n 1{-l)" -
a) Xn=sm
3
;b)xn=-
11
-;c)xn=1+2(-1)"
1
+3(-1)';
d) Xn = _!_n(
1
l" +sin nrr; e) Xn = 1+ (-!)" + (-1)"-
11
-;
n 2 2 2n +I
74
f) F = (]+.!._'
111
. nrr
sm-.
2
R: a) -
e+1;g)-1,1;t
15. a) F
de numere re<
b) Fie J
acumulare in
L(xn) =A.
R: a) J
I .
az+-, ... ,
2 .
b) A=
I I
a1+- a2+-,.
n n
16. Fie
exista XnEA \
Obsen
XE R este pu
al sau 1
17. Sp1
exista MER:
'v'nEN*\{1}:
a) Sa
esll
b) sa
vari
Atunci
proprietatea ca
Atunci (xn)n este
din R*. Atunci
este monoton
n(n-1)
,..-.,,."1)
+I'
f) .Jx> (1+
0
1
) (-1)" +sin
4
11
; g) Xn =

h) Xn =
0
o'+1 3 o+1
. 01!
Sin-.
2
R: a) - .fj ,-1
3
; b) O,oc; c) -4,6; d) -1,1; e) __ !).; f) --e-.!.,
2 2 2 2 2
1
e+1; g) -1,1; h)--, 1.
2
15. a) Fie A c;; R o multime finita. Atunci exista un (xn)n>1
de numere reale, astfel incat: L(xn) = A.
b) Fie A c;; R o multime numarabila care nu are puncte de
acumulare in R . Atunci exista un ir (xn)n>1 din R astfel incat
L(Xn) =A.
1
R: a) A = { a1 ,a2, ... , ak} este a1 ,a2, ... , ak, a1 +
2
,
+ I 1 1 1
a2 -
2
, ... , ak+-, a1 +-, ... ak+-, ...
2 o n
16. Fie A c R. Sa se arate ca XEA' daca numai daca
exista XnEA \ {x} Xn --+ x.
Observatie. Analog, se demonstreaza afirmatia: un punct
xE R este punct limita al unui daca numai daca exista un
al sau convergent in R Ia x.
17. Spunem ca (xn)nEN" este cu variatie marginita daca
exista MER: astfel incat Jx2- X1l + Jx3- X2l + ... + Jxn- Xn-11 < M
l;lnEN*\ {1}:
a) Sa se demonstreze ca orice cu variatie marginita
este convergent;
b) Sa se construiasca un convergent care sa nu fie cu
variatie marginita.
75
lndicatie
a) Yn = fix, - x,.,l convergent=> (xn)n fundamental

R. b) Xn =
1
-(-
1
)"
2n
18. Fie (xn)n un de numere reale.
a) Dad! x* = Xn X = lim Xn, atunci ::; X ::; x* ::;
sup X; , V k :?. 1 ;

b) Multimea L(xn) a tuturor punctelor limita ale dat
este inchisa.
19. Fie o serie de numere reale cu proprietatea ca
""
exista un convergent de numere reale (f(n)n,1 p :::-.1 astfel
incat an = f(n+p)- f(n), Vn:::-.1.
Atunci, seria este convergenta are suma s=
nd
p l}E2 f(n) - f f(k). Aplicatii: Sa se calculeze sumele:

" I " 1 " n+k
a) ..L. , , ; b) ..L. ; c) ..L. , , , ;
"" n -k ,, n(n+p) ,, n (n+1) ... (n+2k)
d) L(p+n)(p+n-l) ... (p+2)(p+l)' q-p>1; e) L_l_' kEN*\ {1};
,, (q + l)(q + 2) ... (q + n) ,, c: ..
f)L_l_. )L(2n-l)!!.h)Ln'+n-l.i)L n .
,, c:,, 'g ,, (2n+2)!!' ,, (n+l)! ' ,, (2n+l)!!
\lndicatie \
Se va calcula suma paftiala Sn. s = 1}!!! Sn.
76
Se aplica
1 1
b)--; c)---
pn 4k n'
R.
a)
3
b) _!_ _!_ c
4k'' .L.
p '"' k .
") 1

2
20. Sa se st
R
. - I
1-a
21. Sa se c
parametrului aER
_ 1 I
... n
2 n
R. $irul (x
pentru a ::; 1 ; (Yn)n
limita sa se
22. Sa sea
sa se calculeze s
_tCn-m+l)(n-mt
n!
R. 2e, 5e, (n
23. Fie l;u.

(an)n un cresca
; x*
ale dat
proprietatea ca
i p <:1 astfel
are suma s=
2k)' '
Se aplica exercitiul teoretic pentru f(n) = : a) ---,
2k n-k
b) __ 1_. c) _I . d)- I q! (p+n)!
pn' 4k n'(n+l)' ... (n+2k-l)'' q-p-1 p!(q+n-1)!
R.
a) 3 . b) I I c) I . d) p + I . e) I . t) 2; g) _21 ,
4k'' pf;-k 4k((2k)!)'' q-p-1' k-1'
") I
I-.
2
h)
1
'
o->
20. Sa se studieze natura seriei L a , a,bER.
<> (l+a"-'b)(l+a"b)
--, lal<l
R S= 1-al+b
\
I I
I I I I
-------, lal>l
1-a l+b 1-a b
21. Sa se discute convergenta ijirului dupa valorile
parametrului aE R, unde Xn = 1 +
2
1
+
3
1
+ ... +
0
1
iji a ijirului
I I
Yn=1+-+ ... +--ln n.
2 n
R. $irul este convergent pentru a> 1 iji divergent
pentru a 1; este strict descrescator iji minorat de 0, iar
limita sa se numeijte constanta lui Euler.
22. Sa se arate ca seria L (pEN) este convergenta i
n!
sa se calculeze suma sa in cazul p = 2 iji p = 3. Calculati
( n - m + I)( n - m + 2 ) ... ( n -I )n'
i...J n!
R. 2e, 5e, (m+1)e.
23. Fie L u" o serie cu termeni pozitivi i descrescatori, iar

(an)n un ir crescator divergent de numere naturale, astfel incat
77
l?irul de termen general a,., -a, sa fie marginit. Sa se a rate ca
a -a
n n-l
seriile l?i -aJu,, sunt de aceeal?i natura (criteriul de
n=! n=l
condensare allui Cauchy).
Observatie. Tntrucat l?irul an = 2" satisface in mod evident
proprietatile impuse, de cele mai multe ori aceasta va fi alegerea.
24. Stabiliti natura seriei geometrice a + aq + aq
2
+ ... +aq"
1
+ a,qER, precum l?i cea a seriilor Ina"' l?i L unde aER.
n'<:l n:<:l a
R. lql <1 => limsn = Iaq" =-a-. lql ::::1 =:. l?irul termenilor
n-.-+< n<:O 1-q
seriei nu tinde Ia 0; L na ,_, =
1
daca lal<1; pentru Ia I > 1
(1-a)''
"''
'i;"' n a
(1-a)'.
Observatie. Daca q = -1, suma partiala de rang n a seriei
geometnce este Sn = , ec1 l?lru Sn n a sumelor
. {0 , daca n = 2k d . . I ( ) I
a , daca n = 2k + l
partiale este marginit, dar nu are limita (conditia de marginire a
l?irului sumelor paftiale ale unei serii nu este o conditie suficienta
pentru convergenta seriei).
25. Daca in seria armonica L eliminam toti termenii care
n<:l n
contin o cifra para, seria rezultata este convergenta l?i are suma
s < 7.
llndicatie I
Grupand convenabil termenii, se obtine:
25 125 325 25 l
S<2+ ... <2+---=7
ll Ill llll 10
1
_ _s:_
10
78
26. Sa se st
a) 2:2"+(:
5
l l l (-1
d)---+-+ ... +-
3 9 27 3
...+------1
(2n -l)(2n + IX2r
R. a)2.;b)
5
-
6
21. Utiliza1
Cauchy), stabili\i c
a) L cos.nx
n<:l 3
Observatie
28. Fie (Xn
astfel incat lim xP
marginit p >1.
29. Fie (Xn)
l,i!'} nP(xn+1 - Xn) = a
30. Daca s
(Xn)nz1 este conve1
llndicatie I
(-1)"
n
Sa se arate ca
(criteriul de
in mod evident
fi alegerea.
+ aq2 + ... +aqn-1
n
, unde aER.
termenilor
; pentru [al >1
l
rang n a seriei
26. Sa se stabileasca suma seriilor:
a) L2"+(-l)"";b)'t2n-1;c)'t 1 ;
"" 5" ,., ( -13)" ,., (n + -./2)(n + -fi + 1)
I 1 I (-1)"'' . I 1 I . 1 I
d)---+-+ ... +---+ ... , e)-+-+ ... + + ... ,f)--+--+
3 9 27 3" 14 25 n(n+3) 1-35 357
I I 2n - I ( -1 y-
+ + g)a = h)b =-- i)c =--
... (2n -1)(2n + 1)(2n + 3) ... , " n(2n + 1)' " 2" ' " 2"-'
5 5-313 I I II I . 2
R a)-;b)
6 2 l+-v2 4 18 12 3
27. Utilizand criteriul general de convergenta (al lui
Cauchy), stabiliti convergenta seriilor:
a) L cos,nx; b) L cos?a .
n;>,l 3 n;,.J n
Observatie. Se poate folosi un criteriu de comparatie.
28. Fie un l;lir strict crescator de numere pozitive
astfel incat lim xP(xn+1 - Xn) = a > 0, cu p >0. Atunci (xn)n este

marginit <=:o p > 1.
29. Fie (xn)n un l;lir strict crescator de numere pozitive a.i.
l}l!J nP(xn+1 - Xn) = a > 0, cu p > 1. Atunci (xn)n este margin it :} p > 1.
30. Daca seria L;lx, -x,_,[ este convergenta, XnER, atunci
este convergent.
llndicatie I
Sn = Irx, -x,,[ este l;lir fundamental. Contraexemplu: Xn =

(-1)"
n
79
. .
31. Fie seria z:U., Un >0 convergenta rn = Sa se
n--1 mn
arate ca: a) seria :t b) seria :t i- converge.
11'"1 rll v r ..
. .
32. Fie seria divergenta L u. , Un >0 Sn = L u, . Atunci:
n-1
a) seria :t diverge; b) seria ! u; converge.
11""1 SD nzl SD
jlndicafje l
Tn problemele 31, 32, divergenta se arata cu criteriul lui
Cauchy, iar convergenta cu primul criteriu de comparatie.
33. 0 serie cu termeni pozitivi :Lx. este convergenta
'"'
definit prin, a1 =1 an+1 = an +..'Sc V n este
a.
convergent.
34. 0 serie Z:x. cu termeni din (0, 1) este divergenta
.,,
(3) (an)n. un strict crescator divergent de numere pozitive astfel
in cat:
a
Xn=1--
an+]
jlndicafje I
"=>" Considerand a1 > 0, an+1 = V n 2, demonstratia
1- x,
este similara celei utilizate in problema rezolvata nr.18.
80
35. Fie seria1
a) lim u, + 2u.J
040 >)
b) :t u,
n( '
.---------, .,)
llndicafje

a) f,:ku, =,
convergent& j
. j
concluzia este imedlJ

l
36. Stabiliti rJ
criteriile de
a) (n!)' ,
(2n)!
d)
g) 1n(1 + :)
)
jlndicatie IJ
Pentru a) ..
pentru g) h) al trti
' l


.,, (2n).. j
j
1
j
..
rn = :L:Um. Sa se
converge.
= L:U,. Atunci:
.. ,
lui
convergenta <::::>
'<! n ;:>:1 este
pozitive astfel
n :?: 2, demonstratia
35. Fie seria Un >0 convergenta. Sa se arate ca:

a)
lim u, + 2u, + ... +nu.
= 0;
n
b)
I U
1
+2u
2
+ ... +nun

""' n(n + 1)
""'
llndicatie
a) t,ku, ns"- t,u,)
convergenta => 3 s = Sn, conform teoremei Stolz;
""
concluzia este imediata; b) sn = I u, +
2
u,+ ... +ku, s.
,., k(k + 1)
36. Stabiliti natura seriilor cu termenul general Un (folosind
criteriile de comparatie):
(n!)' I I n+l
a)--, b) , c) --In--,
(2n)! .Jn(n' +I) n n
I I a"
d)--, p>O, e) , f) .1.' a>O;
(Inn)' (Inn)""' '\1 n!
g)ln(l+2) ,x>O, h)'\la-1, a>l; i)[(
2
n-l)!!]" ,a eR.
n (2n)!!
llndicatie j
Pentru a) ... f) se primul criteriu de comparatie, iar
pentru g) h) al treilea; i)
1
r < (
2
n -
1
)
1 1
< =>
2"1/n (2n)!! "112n
-1)! ']" "" I
L.., are natura cu L.J- .
.,, (2n)!! ""n'
81
R. a) convergenta ( u. < ;. ) ; b) convergenta ( u. < ;J;
c) convergenta ( u. < :,) ; d) divergenta (u. < e) convergenta
( u. < :,) ; f) pentru a < 1, seria este convergenta, iar pentru a ::0: 1
oste d;e.geotO; g) h) d;e<!Jeote [ \0! > o ]
37. Stabiliti convergenta absoluta semiconvergenta
urmatoarelor serii:
a) z: a. JJ . (anln marginit;
.,n(n+ 3)
- (-1)"'' .
b) 2: ' .
n-1 n' n
c) "t (-1)"" .
. , + 1) '
e) L (-l)" a ii"Z\Q, a>O.
u<:l (n+a)"'
R. a) absolut convergenta;
l
s, 0, divergenta
b) a E(0,1], serniconvergentii.;
> 1, absolut convergentii.
d
. t- ) {E(O,l], serniconvergentii.
1vergen a; e a:
> 1, absolut convergentii.
82
c) semiconvergenta; d)
38. Aratati ca
incloncudent, conve
lui Cauchy.
a) azn = aznt
b) azn-1 = (at
39. Fie (an)n>
are limita +oc. Atune
e)
40. Sa sede
K"
a) -, K>l
n!
n!b
h) z:---"-1
, (b+a,)(2b+a
R. Natura
criteriilor de conve
a) converg
c) convergenta

_1_); e)
n--+= un 2
g)
pentru a = b, nat
; e) convergenta
.. semiconvergenta
d)
38. Aratati ca pentru seria , criteriullui D'Aiambert este
""
incloncudent, convergenta putand fi stabilita cu ajutorul criteriului
lui Cauchy.
a) a2n = a2n+1 = a", V n ::::01, a > 0, a * 1;
b) a2n-1 = (ab)"-
1
, a2n = a"'
1
b
39. Fie (an)n,
1
un strict crescator de numere pozitive care
are lim ita +oc. Atunci seria La,., -a. este divergenta.
ao+l
40. Sa se determine natura seriilor cu termen general un:
(
'
K" 1 a"n! , . 11
a) -,K>O;b) 1--) ;c)-,a>O;d)n arcsm-;
n! n nn 2"
( n' +n+1)
La ' '
n=l n
a >0; g)
L l-3-5 ... (2n-1);
"" 2-5-8 ... (3n-l)
n'
f)
e)

lb"
h) L n. , (an)n --+ a.
,., (b +a, )(2b +a,) ... ( nb +a.)
R. Natura seriilor de mai sus se stabile!i)te cu ajutorul
criteriilor de convergenta pentru serii cu termeni pozitivi rezultand:
a) convergenta :: =0); b) convergenta =;);
c) convergenta pentru a < e (Raabe-Duhamel); d) convergenta
(
lim-'!-"'-= I.) ; e) convergenta (lim" 'u =I.) ; f) convergenta a <1
n-"' un 2 n-->oo \1 u, 2
divergenta (lim u.,, = ;
n-"' un 2
h)
HO u b
"''
pentru a = b, natura seriei va depinde de (an)n convergent Ia a.
83
CONVERGENT A SIMPLA, CONVERGENT A
UNIFORMA A I R U R I L O R DE FUNCTII.
Autor: lector.dr. SORIN BAZ
8
CONVERGENT A SIMPLA,
3 CONVERGENTA UNIFORMA
A DE FUNCTII
L_ __________________________ __
3.1. de functii
A. Probleme rezolvate
1.Fie de functii (tn:IR-+IR).,,f"(x)=nPx", pEIN*,
fixat. Sa se determine multimea de convergenta functia limita.
Sa se studieze convergenta uniforma a pe multimea de
convergenta.
olutie.J
Fie x astfel in cat lxl ;:: I . Daca x ;:: I. atunci
iar daca x ( f"(x))" nu are limita.
1
Presupunem lxl < 1. Exista a >0 pentru care 1x1=--.
I+a
Atunci, daca n 2 p putem scrie:
lxl" = I n = I n < I
(I+a) l+na+ ..... +a n(n-I): ... (n-p) aP'
(p +I)!
(p +I)!
= n(n-I) ... (n-p)aP''
. P n (p +I)! n n
dec1 n I xi s; P' 0, de unde lim nPx" = 0.
a n n - I n - p n-.oo
87
. Ar;;adar, pe intervalul ( -1,1), r;;irul converge simplu ditre func(ia
nul a (f(x) = 0).
Vom demonstra ca r;;irul \ fn) nu este uniform convergent
n-1
pe(-1,1). Tntr-adevar, daca alegem x" = -n- vom avea
( )
(
n-1)" (n-1)" 1
f
0
X
0
= nP n ----,---+ 00, deoarece -n- --+ e
r;;i, atunci,
putem afirma: exista &
0
=1 astfel incat, pentru orice n' E IN, exista
n n' r;;i X
0
= n -
1
cu proprietatea lfn(xn)- f(xn)l E
0
.
n
Daca ne restrangem Ia o mul(ime de tipul [-13.13]. din faptul
ca s 13 <I se ob(ine s nP 13 "----,---+ 0, pe baza demonstra(iei
anterioare. Din inegalitatea precedenta, conform criteriului lui
Weierstrass, se deduce convergen(a uniforma a lui f" catre f, pe
orice submul(ime de forma [-13, 13] a lui(-1,1).
2. Pentru r;;irul de func(ii \fn:IR--+ IRlod'
l
sin(x'")
fn(x) = x" ' X* 0
. 0, x=O
, sa se studieze convergen(a simpla,
respectiv cea uniforma, precizand limita r;;i mul(imile de
convergen(a corespunzatoare.

Sa presupunem ca am ales un x * 0, cu lxl <I. Avem
sin(x'") sin
f"(x)= '" x" r;;i, cum x"----,---+0 iar lim--y =1, deducem
X Y-+0 y
tJx)----,---+0, pentru orice XE(-1,1). Pentru x=1 se ob(ine
fo{l)=sin(l), iar daca x=-1, r;;irul f
0
(-1)=(-1)"sin(l) nu are
limita.
88
Daca \xi > 1, atunci I
Tn concluzie, func(i<
{
0, XEIF
f(x)= sin(!), x=1
Alegand r;;irul d1
descrescator Ia 1
ca nu are loc cc
submul(ime de fo
1 I
\fn(x)\ s lx\" sa"-
Analog, cu
nu are loc conve
\x\ s b , folosind
jt
0
(x)\s\x\" sb"-
[- b,b].
ObseM
Concluziil<
[ o, oo ), rezultau
lim ita f nu satisf<
catre functia
uniform convergent
n-1
vom avea
atunci,
demonstratiei
criteriului lui
a lui f, ci'itre f, pe
multimile de
=1, deducem
. x = 1 se obtine
. 1)" sin(!) nu are
sin(x'") 1
Dacalxl > 1, atunci lt,(x)! = x" < 0.
in concluzie, functia lim ita este f: I R- { 1} I R , data de:
f(x)={O, X EIR-{1}.
sin(!), x = 1
Alegand de puncte x, = 'fl( x, = -4%] care tinde
descrescator Ia 1 (respectiv, crescator Ia -1) cand n oo, pentru
1 (-1)"
care avem f,(x,) = fl f,(x,) = fl , se probeaza faptul
ca nu are loc convergenta uniforma pe (-oo,-l)u(l,oo). Pe o
submultime de forma (-oo,-a]u[a,oo), cu a>1, se obtine, insa:
lt,(x)!,.:; ,.:; deci avem convergenta uniforma.
Analog, cu alegerea x, = x, = -'m demonstram ca
nu are loc convergenta uniforma nici pe (-1,1). Daca b E(O,l)
lxl ,.:; b, folosind inegalitatea < !YI , se deduce ca
lt,(x)!,.:; lxl",.:; b" deci (f,), converge uniform Ia 0 pe
[- b,b].
Observatie .
Concluziile negative de mai inainte, referitoare Ia intervalul
[o,oo), rezultau din faptul ca functiile fn sunt continue in x=1, iar
lim ita f nu satisface aceasta proprietate.
89
3. Sa se gaseasca limita de
1
l 2n
2
x
2
+ nx + 3n
\fn:[O,oo)---+ IR
2
, fn(x)=
2
sa se precizeze natura
n, 2n X+ n -I

ISolutiel
Se observa ca, pentru x > o, avem
2n
2
x
2
+ nx + 3n .
lim fn(x) =lim
2
= x, 1ar pentru x = 0,
n-+o:J 2n X + n - 1
lim fn(x) = 3. lim ita irului de este
n-+oo 0--)>00 n - 1
f(x) ={X, X> 0 .
3, x=O
Nu putem avea uniforma pe [0, oo ), deoarece
functiile fn sunt in 0, in timp ce lim ita f nu este.
daca x E [a, oo ), cu a> 0, putem scrie
I I
X+ 3n X+ 3n I 3
f (x)-f(x) = deci fn
n 2n
2
X + n -I - 2n
2
x - 2n
2
2na n '
converge uniform Ia f pe orice multime de forma [a,oo ), cu a> o.
4. Sa se demonstreze ca irul de functii (fn:IR---+ IR),,,,
arctg( .frlx)
f
0
(x) =
3 2
converge uniform pe IR, Ia f(x) = 0.
n +X
lsolutieJ
N(s) = Atunci, folosind
Fie s > 0. Alegem rangul
marginirea arctg, obtinem ca, pentru orice n > N(s) orice
90
x E I R , are loc ir
convergenta uniforr
5. Se da
Folosind criteriullui
irului.
ISolutieJ
Avem, pentr'
I
I
sir
fn+p(x)- fn(x) = l
\sin(n+p)x\
+ (n+p)' n(
(
I I
+ n+p-1-
Fie s > o arbitrar.
orice n > N( s) fixa1
fi x E IR, deci, co1
uniform Ia o functiE
6. Fie
gaseasca limita
convergen\ei.
natura
functii este
deci f
n

, folosind
x E IR , are Joe inegalitatea ltn(x)l <

< ceea ce probeaza


convergenta uniforma a lui fn catre functia nula, pe JR.
5. Se da defunctii {fn:IR IR)n,JN"'fn(x) =I sin<,kx).
k
Folosind criteriullui Cauchy, studiati convergenta uniforma a

Avem, pentru orice n E IN * fixat p E IN arbitrar:
I
( )
- ( )1- sin(n+1)x ... sin(n+p)x < lsin(n+1)x! ...
fn+p X fn X - ( )' + + ( )' _ ( )' +
n+1 n+p n+1
!sin(n + P )xi 1 1 (I 1 )
+ (n+p}' s; n(n+l) +-+ (n+p-l)(n+p) = ;:;-- n+l +-
(
I I) I I I .
+ - ----- =----<-, oricarearf1 XEIR.
n+p-1 n+p n n+p n
Fie E > 0 arbitrar. Daca alegem = [; J +I vom avea pentru
orice fixat p EIN arbitrar lfn+o(x)-fn(x)I<E, oricare ar
fi x EIR, deci, conform criteriului lui Cauchy, {tn) converge
uniform Ia o functie f:IR IR.
6. Fie de functii {fn:IR IR),,, fn(x)=VI+x
2
". Sa se
gaseasca limita sa se probeze uniformitatea
convergentei.
91

Daca lxl < 1, atunci 1 + x'" 1 li!i de aici

Pentru lxl>l,vomavea =
n-+oo
1 + x'" nr:;;; x'" + 1
= lim" '" x'" = x', deoarece x'" 1 . In sfarli!it, lxl = 1
n---+o:. X
conduce Ia lim ifi = 1 . Deci lim ita li!irului va fi functia:
n-+oo
. {x', lxl z 1
f(x) =lim f"(x) = I I .
n-+oo 1, X < 1
Vom demonstra uniform convergenta pe IR a li!irului (fn},
proband-a separat pe (- oo,-1) u (1, oo), respectiv [ -1,1] .
Sa presupunem ca lxl > 1. Are loc relatia:

+ x'" - x' =
1 I
= ( + X
2
" r I + ( + X
20
r
2
X
2
+ + X
20
X
20
-
4
+ Xln-l < fl'
deoarece fiecare din termenii de Ia numitor este mai mare decat
1. Daca lxl 1, se poate scrie 0 < + x'" -1 < ifi -1 . fn ambele
situatii. convergenta Ia 0 a li!irurilor respectiv (ifi -1) va
asigura, conform criteriului Weierstrass, uniformitatea
convergentei lui f" Ia f pe multimile mentionate.
7. Sa se studieze convergenta li!irului de functii
x'
lfn:[1,oo)-+IR} ,f
0
(X)=
4
,.
n:::o X + n
92
\Solutiej
Se observa
x E[l,oo), deci limita
calculam suplf.(x)
X21
x'(3n
2
-x')

(x + n')' '
intervalul [l,oo). Cu
X > X
0
, rezulta Ca )
suplfn(x)- f(x}l = f.(l
QJ
fn(x) =
8. Se da
__!:1)(__ , X E [0
n+x
0, X E[!
converge uniform p

Fie x<l
orice n z nx, inegal
I
<-'
n
de aici
relatia:
mare decat
. in ambele
(Q,i2 -1) va
. de functii
!Solutiej ,
Se observa ca lim f.(x) = lim /
2
= 0, pentru orice
n-+oo n-+oo X + n
XE[1,oo), deci limita irului este functia f(x) = 0, XE[1,oo). Dorim sa
x'
calculam suplf.(x)- f(x)l =sup
4
, . Pentru aceasta, avem
X<':l x ~ t X + n
x'(3n
2
- x
4
)
f;(x) = ( )' , de unde x. = VJr1 este radacina derivatei in
x
4
+ n
2
intervalul [I, oo). Cum f;(x) > 0 pentru x < x. i f;(x) < o pentru
x > x., rezulta ca x. este punct de maxim global pentru f., deci
I I
( )
J3n.Jn J3
supf.(x)-f(x) =f. x. =
2
= c ~ O . Aceasta implica,
x>l 4n 4v n
insa, ca: f . ~ f , uniform pe [1, oo).
8. Se da irul de functii \f.:[O,l] IR) .,, , definit prin
Sa se demonstreze ca irul nu
nx [ n-1]
-- XE 0--
n+X' ' n
[
n-1 ]
XE -n-,1 0,
converge uniform pe [ 0,1] .
!solutieJ
Fie 0 s x < 1 . Alegand nx = [-
1
-] + I , vom avea, pentru
1- X
1t t n -
1
d f ( ) nx L 1
oncenznx,JnegaJaeax<--, ec1 .x=--. a 1m1tase
n n+x
93
obtine: n n: x x. Pentru x = 1, f"(l) = 0, (v)n 2! 1. Va rezulta
l
x, x E[O,!)
lim f"(x) = f(x) = . In plus, daca XE[O, 1), avem:
n-M
0, X= 1
n 2! nx
convergenta nu este uniforma pe
X, n<nx
[0, 1].
9. Sa se verifice, pentru exemplele de ma1 JOS,
aplicabilitatea teoremei de derivare termen cu termen a de
functii:

n , A>O, n2!1.
fn(x)=e-nxcos(nx)
lsolutieJ
a) Functiile f" sunt derivabile pe domeniul comun de
definitie pentru Xo = 0 E [-A, A], fn ( 0) = 0 ----,",..--+ 0 . Din faptul ca

= _ _:_:_:_...:... rezulta: :s; !_, (v)x E IR , ceea ce implica,
n n
folosind criteriul lui Weierstrass, convergenta uniforma a
derivatelor catre functia g = 0. Aplicand teorema de derivare
termen cu termen, gasim ca f" converge uniform pe [-A, A] Ia o
94
functie f, cu proprie
f(O)= limfn(O)=O, rez
n->oo
Observatie.
Se poate ob\in
In care caz se va ilusl
b) Avem f,l
convergent.
Functiile f, aL
se observa ca =
teorema nu este aplic
Observatie.
Daca se lnh
atunci se obtine lf;(l
converge uniform Ia
remarca In plus faptL
concluzia ramanand
B. Problem
1. Sa se precizeze r
domeniul de defini\iE
. Va rezulta
pe
mai jos,
de
de
implica,
A,A) Ia o
functie f, cu proprietatea f'(x) = g(x) = 0, deci f(x) =C. Cum
f(o) =lim f"(o) = 0, rezulta f(x) = 0, x E[- A, A].

Observatie.
Se poate obtine concluzie daca scriem ::; ,
in care caz se va ilustra ideea marginirii domeniului de definitie.
b) Avem f"(O)=l. (v)nEIN, deci (t"(o)l"este
convergent.
Functiile f" au derivatele = -ne-"x[ cos(nx) + sin(nx)]
se observa ca 0) = -n -oo , deci nu converge pe [ 0, A]
teorema nu este aplicabila.
Observatie.
Daca se intervalul [o,A] cu [qfol, o <a< A,
atunci se obtine s (v)x E[qfol, ceea ce ne arata ca
converge uniform Ia g = 0 , aplicarea teoremei fiind posibila. Se
remarca in plus faptul ca marginirea lui [q.Al nu mai este esentiala,
concluzia ramanand valabila pentru intervalul [a, oo), a > 0.
B. Probleme propuse
1. Sa se precizeze natura convergentei (simpla sau uniforma) pe
domeniul de definitie pentru de functii de mai jos:
Inn!
a) IR, fn(x)= -X
n
95
nx
1+n+x
X
x' + n
3
2. Sa se arate ca de functii f":(O,oo) IR, f"(x) = e-nx sin(n'x)
converge simp.lu pe intervalul (O,oo). Sa se determine o multime
de convergenta uniforma.
3. Se da de functii fn:[O.l] IR,
. l(n+l)x", X E[0,1)
fn(x) = . Folosind
0, X= 1
integrarea termen cu
termen, aratati ca (fn) nu converge uniform pe [0,1]. Sa se
gaseasca o submultime a domeniului lui f pe care converge
uniform.
4. Fie fn:[a,b] IR un de functii monotone pe [a,b], care
converge simplu Ia o functie continua Atunci (fn)
converge uniform Ia f, pe [a,b].
96
IRaspunsl
1. a), d), f)
convergenta e vi
neuniform; e) p<'
convergent simpl
f 0, X= 0
f(x)=H. X>O
converge simplu I<
X, XEu[l
k.Z
f(x)= X XEuJ
2' k.Zl
0, X
2.pe[a,oo),a>0
3. pe [o,a], a <1
4. lndicatie: se folc
e-xsin(n'x)
o multime
termen cu
. Sa se
[a,bJ, care
Atunci (f.}
I Raspunsuri.j
1. a), d), f) - converge uniform Ia f(x) = 0; b) multimea de
convergenta e vida; c) converge simplu Ia f(x) = x, dar
neuniform; e) p<1 - uniform convergent Ia f(x) = 0; p < 2-
convergent simplu Ia f(x) = 0; p=2 - convergent simplu Ia
!
0, X= 0
f(x) = x >
0
; p>2 - converge numai in x = 0; g)
converge simplu Ia
X, X E [ krr - : , krr+ J
f(x) = x E { krr
0, X E U ( krr + ."., ku - {(zk + I).".}
4 4 2
2. pe [a, co), a> 0 converge uniform Ia f(x) = 0.
3. pe [ 0, a J, a <I converge uniform Ia f(x) = 0.
4. lndicatie: se continuitatea uniforma a lui f pe [a,bJ .
97
3.2. Serii de functii. Serii de puteri
A. Probleme rezolvate
1. Sa se a rate ca seria de functii f arctg
2
x
3
, x E I R
no! X + n
este absolut convergenta uniform convergenta pe IR.
X
Avem f"(x) = arctg
2 3
, n 1. Folosind inegalitatea
x +n
arctglxl lxJ se obtin majorarile:
If
X
I
<
lxl = 1 < _I_
( ) - pentru orice x E IR. Cum
" - x2 + n3 x' + n' - '
f

este o serie convergenta, aplicand criteriul lui Weierstrass,
""'1 nl2
vom deduce convergenta uniforma convergenta absoluta a
seriei de functii din enunt.
2. Sa se studieze convergenta simpla a seriei de functii
00
L;nxe-"x, x EIR sa se gaseasca o multime pe care seria
n.:..I
converge uniform.
jSolutieJ
Folosind criteriul raportului, vom obtine,
pentru
X*O,
r fn+i(x) _
1
. (n+l)x/e(ncl)X r n+l -X -X
1m t ( ) - 1m I nx = 1m--e = e .
n X nx e n---H.Q n
98
De aici deducem ca
e-x > 1, iar pentru x
Tn x=O seria este in m
Se ca pen
Vom putea scrie, astf
I l
nx 6nx
fn(x) = e"x < (nx)' = r
Tn ipoteza ca X El
lfn(x)l cum :
na
Weierstrass, deduce
interval de forma [a, a
3. Se da seria
i) Sa se afle multim
seriei.
ii) Pentru 1' sa s
jSolutieJ
Se observa ci
Daca x > 0, vom av
serie geometrica cu
rezulta, folosind crit
convergenta pe (O,oo
cum
X
XEIR
inegalitatea
eiR. Cum
absoluta a
de functii
care seria
pentru
De aici deducem ca pentru x < 0 seria este divergenta, deoarece
e-x >I, iar pentru x > 0 seria este absolut convergenta ( e x <I).
ln x=O seria este in mod evident absolut convergenta.
u'
Se tie ca pentru orice u > 0 avem e" > kT, k E IN arbitrar.
Vom putea scrie, astfel: enx > (n;r , daca X> 0, de unde ob\inem
I I
nx 6nx 6
f,(x) = --..x < -( )' =--,--,.
e nx n x
ln ipoteza ca x E [a, oo ), cu a > 0, ajungem . Ia inegalitatea
lt,(x)l ,
6
, i, cum seria :f: este convergenta, din criteriul lui
n a no! n
Weierstrass, deducem convergenta uniforma a seriei pe orice
interval de forma [a, oo ), a > 0.

3. Se da seria de functii :Le-xcos(n+2)x, x EIR.
n-=0
i) Sa se afle multimea de convergenta i sa se determine suma
seriei.
ii) Pentru x I , sa se studieze convergenta uniforma.
ISolutieJ
Se observa ca lt.(x)l = e-xlcos(n + 2)x[ e-x.

00
(1)"
Daca x > 0, vom avea e-x <I i seria = ex este o
serie geometrica cu ratia subunitara, deci convergenta, de unde
00
rezulta, folosind criteriul comparatiei, ca :Lt.(x) este absolut
n=l
convergenta pe (o,oo). ln x = 0 avem f.(o) =I, oricare ar fi n 0
i, cum * 0, seria este divergenta. Pentru x < 0 avem
99
e-nx --.;---+oo :?i, cum lim ita pentru n---+ oo a lui cos(nx) nu exista,
cu excep\ia lui x de forma 2kn, k E Z , seria este divergenta.
Pentru x > o sa notam:
w w
s,(x)=Ie-nxcos(n+2)x :?i s2(x)=Ie-nxsin(n+2)x'
Atunci avem: S(x) = SJx)+ iS
2
(x) = f(e-x)"[cosx+isinxr' =
n==O
(cosx+isinxr cos(2x)+isin(2x)
1- e x(cosx + isinx) = (1- e-x cosx)- ie-x sinx =
[cos(2x) + isin(2x)J[(1- e-x cos x) + ie-x sinx]
=
2
.
2
, deoarece avem o
(1-e-xcosx) +(e-xslnx)
serie geometrica cu ratia e-x(cosx + isinx) de modul complex
egal cu e-x <I. Partea rea/a a lui S(x) este egala cu
S (x) = cos(2x)- e-x[cos(2x)cosx + sin(2x)sinx] =
1
]-2e-xCOSX+e
2
x
cos(2x)-e-xcosx . . - ..
= x
2
x :?1 reprez1nta suma sene1 de functii cautata.
l-2e COSX+e '
Daca presupunem ca x?: I, vom avea jtn(x)j,; e-nx,; e-n.
w 1 1
Cum I
11
e convergenta ca serie geometrica cu ratia - , din
_e e
criteriul lui Weierstrass rezulta convergenta uniforma a seriei pe
intervalu/ [ 1, oo).
@sa se arate ca
convergenta, iar suma ei este
continua.
w 1
seria I
2 2
, x EIR, este
""' n + x
o functie derivabila, cu derivata
100
ISolutieJ
Sa notam cu ~
seriei . Pentru n e
1 I
< ++--"""'
(n+ 1)
2
(n+p)'
Cauchy seriei conver
gasi un N( : ) astfe
(v)x E JR, (v)p E IN. [
seriei de func\ii pe IR
sunt derivabile, c
2lxl I
j t ~ ( x ) j = n' +X' . n' + x
Weierstrass ob\inem '
Aplicand teorema de
functii, se obtine ca ~
S'(x) este continua, c
func\ii continue.
5. Sa se studie
nu exista,
divergenta.
]
n+2
X =
avem o
complex
cautata.
I .
- d1n
e'
seriei pe
este
n
Sa notam cu Sn(x) = L ,
1
2
irul sumelor paf\iale ale
k +X
seriei . Pentru n EIN*, fixat i p EIN, arbitrar se obtine
\Sn+P(x)- S"(x)\ = ( ;, + + ( ;,
n+l +X
2
n+p +X
2
1 I . f. IR A . d
( )' + + ( )' , oncare ar 1 x E . pl1can criteriul
n+l n+p
oo I
Cauchy seriei convergente :L;-, pentru un e > 0 oarecare, vom
n=l
gasi un N(g) astfel !neat daca n > N(E), \Sn+p(x)- Sn(x)\ < g,
(v)x E IR, (v)p E IN. De aici se deduce convergenta uniform a a
seriei de functii pe IR catre o functie S(x). Functiile f"(x) = n' I 2
+X
sunt derivabile, cu derivate continue
pe IR. Cum
2\x\ 1 I 1
lf;(x)l= ' 2. 2 2 ' (v)xEIR,
n +X n +X n +X n
din
criteriul
00 '
Weierstrass obtinem ca L f" (x) este uniform convergenta pe IR.
Aplicand teorema de derivare termen cu termen a seriilor de
00 '
functii, se obtine ca S(x) e derivabila i Lfn (x)= S'(x). ln plus,
n=l
S'(x) este continua, ca suma a unei serii uniform convergente de
functii continue.
oo ' n
5. Sa se studieze convergenta seriei de puteri L .
n-=0 n.
101
\Solutiej
n' . \an.,\ . (n + 1)
2
n!
Cum a" = -
1
, vom avea lim -
1
-
1
= lim (
1
)
1
, = 0.
n. 0--+o::> an n4w n + . n
de unde obtinem ca raza de convergenta este oo . deci seria este
absolut convergenta pe IR uniform convergenta pe orice
interval de forma [-a, a], 0 <a < w .
6. Gasiti multimea de convergenta a seriei de puteri
00 x"
L n
""' n3 Inn
\Solutiej
Avem lim"" Pa I = lim
1
=.!. deoarece lim vn = 1 iar
n--+o::> \fli::ln) n-Ho 3VnViM 3 '
ViM are limita pentru n w ca +
1
) . adica 1. Va
nn
rezulta, conform teoremei Cauchy-Hadamard, ca raza de
comiergenta este R=3, deci seria converge absolut pe (- 3,3).
Studiem convergenta In capetele intervalului. Pentru x = 3 avem
seria numerica f-
1
- care este divergenta, concluzie care
""' nlnn
poate fi obtinuta folosind criteriul integral al lui Cauchy. Oaca
00 1
x = -3, seria devine 2:(-1)" -- observam ca sunt satisfacute
""' nlnn
conditiile din criteriul de convergenta al lui Leibniz: --
1
1
--- > 0 este
n nn .
descrescator (deoarece (xlnx)' = lnx + 1 > 0, pentru x > ..!_)
e
102
convergent Ia zero. i
de puteri este interv;
7. Sa se slue
f 2" '(x+ 1)
2
"
""' (4n+1)
2
\SolutieJ
Avem a,"=
calcula
ro = limvfaJ = lim)
04CO n----tro V
astfel !neat raza
1
x = -1 - J2 sau x
00 1
2:;( ,,care
""'2 4n+ 1)
convergenta).
in concluzie
convergenta pe n
(-w,-1-
= 0,
seria este
pe orice
de puteri
= 1 iar
ladica 1. Va
raza de
pe (- 3,3).
=3 avem
care
> 0 este
1
X>-) ljii
e
convergent Ia zero. Tn concluzie, multimea de convergenta a seriei
de puteri este intervalul [- 3,3).
7. Sa se studieze convergenta seriei de puteri
I 2n-'(x + 1)'n
nol (4n+1)
2
.
Avem ljii a,n-I = 0 pentru orice n z 1. Vom
calcula
- 2n-l 1 1
ro = = lim 2 ( )' = 2"ry
2
n lim-n R = J2
n....ro 4n + 1 !i;4n 1 '
n 1+--
4n
astfel lncat raza de convergenta va fi R = }z . Pentru
1 1
x = -1- J2 sau x = -1 + J2 se obtine aceealjii serie numerica,
1 1
I ( )' , care este convergenta (comparatie cu _I-, ,

convergenta).
Tn concluzie, conform teoremei lui Abel, seria este absolut
convergenta pe multi mea [ -1- }z ,-1 + }z] i divergenta pe
(-oo,-1- .}z)u(-1+ Jz.oo).
103
Observatie.
Aceea!,ii concluzie se putea obtine facand substitutia
ctJ
2
n-l y"
y = (x + 1)
2
!,ii gasind raza de convergenta a seriei cu
n"J(4n+1)
2
ajutorul criteriului raportului.
8. Pentru ce valori x E IR- {-1}, seria L(3n+l) x-_
( l)n
n=O X+ I
este convergenta?
ISolutieJ
f b
. . x-1 bl d
Vom ace su st1tut1a y = --
1
, x *-I, pro ema re ucan-
. X+

du-se Ia studiul convergentei seriei de puteri L(3n+l)yn pe
multi mea lm q>, unde q>(x) = x *-I. Seria in y are raza de
t
- 1- 1 d I. lan.,l I. 3n+4 o
convergen.a ega a cu , eoarece 1m =I. 1n
n-+IXl a n4oo 3n + 1
n
y= x-l se deduce x= y+l, de unde rezulta ca lmq>=IR\{1}.
X+] 1-y
Observam ca [ -1,1] r1 lm q> = [- 1,1). Pentru y = -1 , seria de puteri
este divergenta, deoarece !,iirul ( -1)"(3n + 1) nu are limita.
x-1
Ramane sa determinam valorile lui x pentru care - 1 < --< 1.
X+!
Aceste inegalitati sunt echivalente cu -
2
- > 0 !,ii > 0. Astfel
' X+1 X+]
se obtine
x =(O,oo), deci seria de functii din
enunt converge pe multimea ( 0, oo) .
104
9. Folosind deri
se calculeze sumele se

b) 1 - 3x
2
+ sx _,. = L
11=1
a) Raza de con
1
=---.=-=
lim 1 __
n->oo 2n + 1
I, de
verifica u!,ior ca in x =
f.!_). Prin derivare ter
n=O n

4 """' 2n
+ x + .. = L... x , care 1
Daca notam cu S(x) Sl
derivare termen cu
S'(x) = fx
2
n =

I
n=O 1- X
( arg th este functia i
th(o) = o, rezulta c = (
demonstra ca argthx =
cu
+l)(X-1)"
X+!
reducfm-
+ l)y" pe
' are limita.
X-1
1<--<1
X+ I .
>0. Astfel
func!ii din
9. Folosind derivarea sau integrarea termen cu termen, sa
se calculeze sumele seriilor de puteri de mai jos:
00
b) l-3X
2
+5X
4
-= L(-1) (2n-1)x
2
"-
2
,
n=l
a) Raza de convergenta a seriei este R = .
1
faJ =
hmsup" ani
n-+oo
=
= 1 , deci seria este convergenta pe (- 1,1) . Se
I
. '
1m .. n+

verifica ca in x = 1 x = -1 seria diverge (comparatie cu
f.!.). Prin derivare termen cu termen se obtine seria 1+x
2
+
n"'O n

+X
4
+= l:x
2
", care este o serie geometrica cu ratia x
2
E(0,1).
Daca notam cu S(x) sum a seriei ini!iale pe ( -1,1), din teorema de
derivare termen cu termen a seriilor de puteri obtinem
s(x) = Ix'" = -
1
-,. De aici deducem ca S(x) = argthx + c
n=O 1- X
(argth este functia inversa lui th = ). Cum S(o) = 0
th(o) = 0, rezulta c = 0, deci S(x) = argthx,x E(-1,1) (Se poate
demonstra ca argthx
lOS
b)Deoarece
(
2n-l)'h
lim sup = lim -1 = lim
2
":(;/2n- 2 lim = 1, raza
n---J.oo n---J.oo n-cl-C(J n--+oo 2n - 2
de convergenta a seriei va fi R = 1. Seria este, clar, divergenta Tn
1. Prin integrare termen cu termen pe intervalul Tnchis
[ 0, u ], 0 < u < 1 , vom obtine seria de puteri

a carei suma este


u
n=I
'
Derivand suma de mai Tnainte, obtinem = (
1
- u
2
t i
1+U 1+U2
rezulta ca suma seriei initiale va fi (
1
- x
2
)
2
, pe intervalul (-1,1).
1 + x
2
B. Probleme propuse
1. Pentru seriile de functii de mai jos, determinati multimile de
convergenta, precum i natura convergentei:
) sin(dx) b) _2__ x
2
" .
a n
3
+ 1 ; ;2. 2" 1 + X
2
" '
c) f d)
n=O e n=O
)
a"cos(nx) II .
eL.. ,a<1,
n=l n
n
2
+ 5 ( X )" I
f Xofc-
) n
2
+ 4n + 3 2x -1 ' 2'
lRaspunsuri.l
a), b), e) uniform convergenta pe I R; c) absolut
convergenta pe ( 0, oo), divergenta Tn rest; d) absolut convergenta
106
pe U(2kn,(2k + 1)n),
(- u ( 1, oo ), diver
2. Aratati ca seriile
specificate, iar sum
continue:
)
cos(nx) IR
a_. a,XE,
no\ n
c) tarctg(;,). XEI
3. Sa se determine 1
de mai jos:
n
a) L(-1) (2n+l)
2
x'
n=O
raza
ldiiiP.rr,.,nf-' In
Inch is
puteri
u
I+ u
2

I- u
2

{I+ u
2
Y
pe U(2kn,(2k + I)n), divergenta In rest; f) absolut convergenta pe
kEZ
(- '"" u (I, oo ), divergenta in rest.
2. Aratati ca seriile de functii de mai jos converg pe multimile
specificate, iar sumele lor sunt functii derivabile cu derivate
continue:
f. cos(nx)
a) L., a ' X E IR, a> 2;
n=l n
c) x EIR;
00
b) L e-nx sin(nx), X:?: 1;
n=O
d) sin(nx) IR
L. 2" I X E I
n=l
(- I,1) 3. Sa se determine multi mile de convergenta ale seriilor de puteri
de mai jos:
I
" -
. 2'
absolut
00
n
a) L(-1) (2n+1)
2
x"
n=O
b)
<X) n x3n+l
c) n+1
d oo (x+S)'"-'
) 2n4"
e) f n; (x)"
n=l n 2
f) I(-1)" Vn+2(x-2)"
n=O n +I
x2nt-l
g) (2n+I)!
R:(-1,1)
R:[-1,1)
R{- Jz.Jd
R:(-7,-3)
R:(-2e,2e)
R:(1,3]
R:IR
107
.h) f[1-(-2)"]x"
R=l
R: (- _!_ I..)
2'2
') f. (z- i)" C
IL..,., n,ZE
""' n3
R:lz-il <3
00 z2n
j) ZE C
n=O 2
4. Sa se determine multimile de convergenta lii sumele pentru
urmatoarele serii de puteri:
x"

""' n
R=l
IRaspunsuri: I
1
a)[ -1,1), S(x) =In-;
1-x
2
c) (-1,1), s(x) = ( )';
1-x
oo 2n+l
b)
n=o 2n+1
2"
d)


n=l 3
b) [-1,1], S(x) = arctgx;
(
3 3) 6x
d) - 2 '2 ' S(x) (3- 2x)
2

3.3. Serii Taylor
A. Probleme rezolvate
1. Sa se dezvolte in serie Taylor in jurul punctului a= 0,
functia f:IR IR, f(x) = sinx.
108
fo1u*ie1 ,

Functia f(x) 4
I
Tntr-adevar, (sin)
sin<l(x) = sinx, etc.
Prin inductie se po.
1<20, (sin)
2
k(x) =
'
l
Va rezulta ca
lii deci seria
x' Xs X7. .H
x--+---+ ..
3! 5! 7! 11
. 1/(2n-
oeoarece



convergenti"'a
Taylor este tocmai
argument x astfel .

forma lui
i

intre 0 lii x .
M".fl
avea (n+4
converge Ia 0 1*1

lxl ::;; M , unde
4
j
dezvoltarea in
i
a= 0,
ISolutieJ
Functia f(x) = sinx este indefinit derivabila pe IR.
intr-adevar, (sin)' (x) = cosx, (sin)" (x) = -sinx, (sin)('J(x) = -cosx,
sin('l(x) = sinx, etc.
Prin inductie se poate demonstra ca, in general, pentru orice
(sin)'k(x)=(-1lsinx (sin)'k+
1
(x)=(-1lcosx.
{
0, n = 2k
Va rezulta ca f(nJ ( 0) = ( )k
-1 ,n=2k+1
deci seria Taylor asociata lui f in punctul 0 va fi de forma:
x3 x5 x7 co n x2n+l
x-3!+5!-7!+=

(2n+1)!.
D I
. 1/(2n + 3)!
1
. 1 .
eoarece 1m /(
1
)
1
= 1m ( )( J) = 0, vom avea
1 2n + . n--+oo 2n + 2 2n +
lim = 0, de unde rezulta ca raza de
"""' 00
convergenta a seriei este R = ro. Sa demonstram ca suma seriei
Taylor este tocmai f(x). Alegem o constanta arbitrara M > 0 un
argument x astfelincat lxl :s: M. Restul Taylor de ordin n, scris sub
x"'l
forma lui Lagrange, va fi R"(x) = ( ) sin(ntl(s), cu s cuprins
n + 1 !
intre 0 x. Deoarece l(sin)(")(u)l :s: 1, oricare ar fi u E IR, vom
M"+r M"+l
avea JR"(x)J :s: ( )I cum ( )I rezulta ca R"(x)
n+1. n+1.
converge Ia 0 pentru orice XE[-M,M]; deci f(x)=limT"(x),
n--+oo
n k x2ktl
lxl:s:M, unde T"(x)= L.;(-1) ( k )
1
. M fiind oarecare, se obtine
2 + 1 .
x2n+l
dezvoltarea in serie: sinx = L.;(-1)" -(--)I' x EIR.
n=o 2n + 1 .
109
2. Sa se gaseasca dezvoltarea in serie Maclaurin a
functiei f:(-1,oo)---+IR, f(x)=(1+x)", a EIR.
ISolutieJ
Functia f este indefinit derivabila pe (-1, oo) ca o
compunere de doua functii indefinit derivabile: x---+ x + 1 pe
(-1,oo) x---+ x" pe (O,oo). Se obtine:
f'(x) = a(1 + x)"-
1
, f"(x) =a( a -1)(1 + x)"
2
in general,
t<nl(x) =a( a -1}. (a-n+ 1)(1 + x)"n, cum rezulta folosind
inductia matematica. Deci f(nl( 0) = a( a -1} .. (a - n + 1) seria
oo f(nl(o) a a( a -1)
Maclaurin este t(o) + L xn = 1 + -x + x
2
+ ...
. no1 n! 1! 2!
a(a-1) .. (a-n+1) n
.. + X+
n!.
P t t
- . I' lan+11 ,. n-a d
en ru aceas a sene avem 1m -
1
-
1
= 1m-- = 1 eci raza ei
n-)O'J a 0--).00 n + 1
n
de convergenta este egala cu 1, oricare ar fi a E IR. Restul de
ordinul n sub forma lui Cauchy este:
(
.) x(x-sJ ( ) ( \1 )a-n-1
Rn x = n! a a-1 ... a-n
11
1+sn , cu snintre 0 x,
care, cu notatia sn = enx, o <en < 1, devine:
R ( )
_a( a -1)- .. (a- n)( )a-n-1( )n n+1
n X - I 1 + enx 1- en X .
n.
Rearanjand factorii produsului, obtinem:
R ( )
- (a-1Xa-2) .. (a-1-n+1)xn ( )"-1( 1-en )n
n X - ax 1 + enx
n! 1+enx
Se observa ca (a -
1
)(a-
2
) .. (a -
1
- n +
1
)xn este termenul
n!
general al seriei Maclaurin asociate functiei (1 + x)"-
1
cum
110
aceea este converge
va tinde Ia zero ciind
la,q(1-l,q}"
1
< ]ax(1 +
cu sensu! schimbat
en E (0,1) XE(-1,1)
Va rezulta astfel
este valabila dezvolt
oo a( a-
(1+x)" =1+ I-
0=1
serie binon
de mai sus devine fc
Observatie
Relativ Ia valil
intervalului (-1,1), se
Pentru a 0, st
x = 1 (conform cril
functie continua in
capetele intervalului:
daca -1 <a < o,
egalitatea este vala
Raabe);
daca a :0:-1, ser
criteriul necesar de<
3. Sa se de2
f(x) = 12-+ x .
Maclaurin a
. X-+X+l pe
folosind
seria
1, deci raza ei
e IR. Restul de
X,
aceea este convergenta pentru orice x E (- 1,1), factorul mentionat
va Iinde Ia zero cand n oo . De asemenea, au loc inegalitatile:
<lax(!+ 8nx)l"-' </ax/(!+ /x/r-', daca a 2 I, (respectiv
cu sensu I schimbat daca a < 1) 0 <
1
- :n < 1, Tn ipotezele
I+ nx
9n e(O,J) XE(-1,1), de 0<( l-Sn )n <1, ('v)n21.
1 + enx
Va rezulta astfel ca pentru orice x E(-1,1), adica
este valabila dezvoltarea:
(I+X)"=1+Ia(a-l) ..

/x/<1. Seria obtinuta se


n=l n.
serie binomiala, deoarece, daca a= m, m E IN, formula
de mai sus devine formula binomului lui Newton.
Observatie.
Relativ Ia validitatea dezvoltarii obtinute anterior Tn capetele
intervalului ( -1,1), se pot demonstra urmatoarele:
Pentru a 2 0, seria de puteri este absolut convergenta Tn
x = 1 (conform criteriului Raabe-Duhamel) cum (1 + x)" este
functie continua Tn x = 1 x = -1 , egalitatea este valabila Tn
capetele intervalului;
daca -1 < a < 0, seria converge Tn x = 1 diverge Tn x = -1
egalitatea este valabila Tn x = 1 (se folosesc criteriile Leibniz
Raabe);
daca a s -1 , seria diverge Tn x = 1 x = -I (nu este Tndeplinit
criteriul necesar de convergenta).
3. Sa se dezvolte Tn serie de puteri functia
f(x)= h+x.
111
Observam ca f(x)=[l+(x+l)]
1
. ceea ce ne sugereaza
folosirea seriei binomiale cu a = , in ipoteza -I < x +I < I,
adica pentru x E (- 2,0). Noland y = x +I . se obtine:
( )
"
2


l+y =1+l!y+
21
Y++ n! Y +
I I
2
n-113 .. (2n-3)
0
+ .. =J+z-y- 222!y ++(-!) 2"n! y + ..
Revenind Ia x. vom avea:
I ro n-!(2n-3)11 n
-J2+X = 1+-(X+l)+ ,L(-1) ( )II .. (X+l), X E(-2,0),
2 n=
2
2n ..
dezvoltarea fiind valabila l?i pentru x = -2 l?i x = 0. deoarece
0. De pentru x = 0. primii termeni dau:
r;.; I I 3 35 357
"L "'I+---+ - + = 1.426.
2 24 246 2468 246810
4. Sa se dezvolte in serie Taylor, in jurul punctului a= 0.
functia f: IR \ {- 2,-3}---+ IR. f(x) = ,
3
x .
. X +5X+ 6
ISolutieJ
Descompunand functia rationala f in fractii simple, se
ajunge Ia:
3x 6 9
=---+--
x2 + 5x + 6 x + 2 x + 3
I 12
6 3
Dar,-----
X+2 X
1+-
2
_9_ = _3_ = 3[1
X+3 X
1+-
3
Pe domeniul com
sus, care este mul
x
2
+
3
5:+6


5. Sa se 1
serie de puteri ale
ISolutieJ
Daca notarr
y < I pentru orice
urmatoarea funcjie
g(y) = y/(1-y)

Pentru valori ale
dezvoltari in serii b
Astfel, vom putea!
ne sugereaza
-l<X+l<l,
0, deoarece
a= 0,
simple, se
6 3 [ x x' x" J
Dar, pentru lx/<2 I
X+ 2 X 2 4 2" '
1+- .
2
9 3 [ x x' x" J
X+
3
=--x =31-3+9-+(-!)"y;-+, pentru lx/<3.
1 + --
3
Pe domeniul comun de convergenta al celor doua serii de mai
sus, care este multi mea (- 2,2), vom pule a scrie:
3
x = 3[(.!.- .!.)x- (.!.- .!.)x' + +(- 1)"(-...!:... + __!_)x" + ] =
x' + Sx + 6 2 3 4 9 2" 3"
"( 1 1)
=L)(-1) +11 x".
2 3
X
5. Sa se dezvolte functia f(x)= .J1+X In
1+X
serie de puteri ale lui ___)(____.
I+ X
jSolutieJ
Daca notam y = _x_ vom avea x = _Y_ i se observa ca
l+x 1-y
y < I pentru orice x > -I. Substituind pe x In expresia lui f gasim
urmatoarea functie de y:
g(y)= y/(1-y) = y =-FY+ 1 .
Jl+y/(1-y) FY Fv
Pentru valori ale lui y cuprinse lntre -1 i 1, vom putea folosi
dezvoltari In serii binomiale cu a =.!., respectiv a = -.!..
2 2
Astfel, vom putea scrie:
113
FY =(1-y)l 2-

3
2 2! y 3! - y +. +
+(-!}" y"+=
n!
= 1-l_y __ I_ 2 3 (-1)
2
"-
1
13-(2n- 3)
2 2
2
2! Y 2
3
-3! Y -+ 2"n! y"+
In mod asemanator, avem:
I ( '
FY= 1-yr' =
1
2 2 2 2
2 2! y - 3! y' +. +
+ ( _ I)" (- -1} .(- i -n + 1) y" + .. =
n!
135 , (-Iti3(2n-1)
2 2
2
-21 2
3
-3! Y ++ 2"nl y"+
De aici rezulta: .
( )
- 1 [ l 2 13
gy -Y+2'211+3y +-,-[1+5]y'++
. 2 3!
13(2n-3) ( )
+ n [I+ (2n -l)]y" + .. = y +" I 3. 2n- 3 n
2 n! 2"-
1
( -1)1 Y
n-2 n .
Cum pentru y =
1
: x avem g(y) = f(x), se obtine:
x =-x+f(2n-3)!!( x )"
Jl+X X

(2n-2)!! l+X
114
Conditia y > -I va im
de mai sus fiind valat
pentru
6. Sa se gase
functiei lui Gauss, G:
sa se calculeze G(1)
folutieJ
Ne vom foiOS'
ex, valabila pe IR: e
dezvoltarea:
-t' t' t'
e = !--+--+
!! 2!
Prin integrarea tenr
intre 0 !?i x, se ajung
X
Je-''dt =I;(- I)" I(:
0 n.
00 2
deci G(x) =I--
""o ..[; r
De aici se deduce c
G(l) = _2__ f (-I)"
Fn n-o n!(2n +
Seria obtinuta per
conditiile criteriului I
n
" y +"" ..
++
Conditia y > -I va impune x > - Dezvoltarile in serie binomiala
de mai sus fiind valabile in y = -I , egalitatea va fi adevarata
pentru XE[
6. Sa se gaseasca dezvoltarea in serie Maclaurin a
functiei lui Gauss, G:[O,oo) IR, G(x) = Je-"dt. Cu ajutorul ei,
"1/ n o
sa se calculeze G(1) cu o eroare mai mica decal 0,0001.

Ne vom folosi de dezvoltarea in serie Maclaurin a functiei
oo x"
ex, valabila pe IR: ex= Z:-
1
. Prin inlocuirea lui x cu-t' se obtine
n=O n.
dezvoltarea:
-t' !
2
!
4
n !
2
"
e =1--+--+(-1) -+ t EIR.
I! 2! n! '
Prin integrarea termen cu termen a seriei de puteri de mai sus
intre 0 x, se ajunge Ia egalitatea:
X <Xi X2ntl
Je-"dt = 2:(-1)" I( )' x EIR,
0 n=O n. 2n +I
00 2 (-1)"
deci G(x) = 2:---x'"+'.
n=o -J:;; n!(2n + 1)
De aici se deduce ca
2 oo (-1)" 2 [ I (-1)" ] 2
G(l)= = -J:;; l-3++ n!(2n+l) + -J;R".
Seria ob\inuta pentru x=1 fiind o serie alternata satisfacand
conditiile criteriului Leibniz, avem jRnj < ( ) ( )
n+ I ! 2n+ 3
115
. Trebuie gasit eel mai mic n natural pentru care
2 1 _
4
2 I .
1 ( )t( )<10 . Pentrun=Savem
1
---=0,00012
'1/ n n +I . 2n + 3 '1/ n 6 !13
pentru n=6, '7:-- -
1
1
- = 0,000015. Vom a lege deci n=6 obtinem
'l/1t 7.15
2[ 1 1 1 1 I 1 J
1-3+2!5-3!7+4!9-5!11+6!13 =0,8427, valoare
care reprezinta o aproximare cu o eroare inferioara lui 10-4 .
B. Problema propuse
1. Sa se gaseasca dezvoltarile in serie Taylor in x=O ale functiilor
de mai jos, specificand multimile pe care sunt valabile:
a) f(x) = cosx
b) f(x) = arcctgx
c) f(x) = arcsinx
d) f(x) = sh(x)
e) f(x) = xe'x
f) f(x) - 3x- 5
- x' -4x+3
X dt
g) f(x)= J
0 1- t
4
00 n x2n
R: _L(-1) -( )I' XE IR
o=o 2n .
00 n x2n-l
R: 2::(-J) X E[-1,1]
0=1 2n -1
(2n -1) 11
R X+ 2:: .. X
2
0+l X (-11)
. o=l 2" n!(2n+l) ' E '
OJ x2n+l
R: .Lc )1' x E IR
o=o 2n +I .
w (-1)"2oxo+l
R:x+_L
1
,XEIR
""'] n.
R: - f(1 + :,)X
0
, X E(-1,1)
o=O 3
R
. f. (2n -1)!! 4ool ( )
. X + L., n ( )
1
x , X E - 1,1 .
0=1 2 4n + 1 n.
IIG
2. Sa se dezvolte
jurul punctului X
0
=
R: -78+59(x+4)
3. Cu ce precizie s
cinci termeni ai set
R: err< 0,091.
4. Sa se calculez
R: 0,621.
5. Sa se scrie prirr
puteri ale lui x pen
a) f(x) = tgx
b) f(x) = lncosx
c) f(x) = secx
pentru care
valoare
ale functiilor
2. Sa se dezvolte functia f(x) = x'- 2x
2
- 5x- 2 in serie Taylor, in
jurul punctului X
0
= -4.
R: -78 + 59(x + 4) -14(X+ 4)' + (x + 4)'.
3. Cu ce precizie se calculeaza numarul : cu ajutorul primilor
. . . . . . x' x' x' d - . I 1?
cinci termem a1 sene1 x--+---+ aca x 1a va oarea .
3 5 7
R: err< 0,091.
1
4. Sa se calculeze J
5 1
~ x d x cu o eroare mai mica decat 10-
3

o -vx
R: 0,621.
5. Sa se scrie primii trei termeni nenuli ai dezvoltarii in serie de
puteri ale lui x pentru functiile de mai jos:
x
3
2x
5
a) f(x)=tgx R: x+-+-
3 15
b) f(x) =In cos x
c) f(x) = secx
(
x' x
4
x
6
)
R:- -+-+-
2 12 45
x' 5x
4
R: 1+-+-
2 24
117
FUNCTII DE MAl MULTE VARIABILE
Autor: lector.dr. IFTIMIE BOGDAN
8) FUNCTII DE MAl MULTE VARIABILE
4.1. Domeniu de definitie. Limite. Continuitate
A. Probleme rezolvate
1. Sa se determine domeniul maxim de definitie pentru
functiile:
a) f(x, y) = ln(1 - x?-- I>
I Solutie I
Trebuie pusa conditia: 1 - x?-- I> 0 ~ x?- +I< 1
Dmax = {(x, y) E IR
2
/x?- + i < 1} (interiorul cercului cu centrul
in origine ~ i raza 1 ).
y
b) f(x, y) = "'1-1 x -11 + Ji=TY[
Dmax = {(x, y) E IR
2
/Ix-11 :S 1, IYI :S 1}
lx-11 :S 1 ~ -1 :S X- 1 :S 1 ~ 0 :S X :S 2
IYI :S 1 ~ -1 :S y :S 1
A ~ a d a r , Dmax = {(x, y) E IR
2
/ 0 :S X :S 2,-1 :S y :S1} =
= [0,2] X [-1, 1] (patratul din figura CU interiorul sau)
121
y
0
-1
c) f(x, y) = Jsin(x
2
+ y
2
)
X
Dmax = 2 0} = U {(x, y)/2k 1t::; + j::;
k.'2.0
::; (2k + 1) 1t}
Domeniul maxim de definitie este format din multimea
punctelor apartinand unei familii de coroane circulare.
d) f(x,y,z) =In z+

-x
2
-y
2
Dmax = {(x, y, z) I z > 0 z
2
- -12 0}
Ecuatia x
2
+ I = z
2
in spatiu tridimensional o
suprafata conica. Domeniul de definitie este format din punctele
interioare de pe suprafata conului x
2
+ l = z
2
, cu exceptia
originii.
y
e) f(x, y) = arcsin y
X
z
X
0
Dmax= {(x, y) E IR
2
1X>' < 1} = {(x, y) E IR
2
/x >' 0,
IYI < lxl}
122
IYI < lxl '} -lxl
Domeniul de c
cuprins intre dreptelt
exceptand originea (I
2. Fief: IR \{C
Sa se determi
existenta limitei func1
jsolutie \
Fie <P (x) = lilT
Y->'
a) x = 0
y->

y->
Deci <P (x) =
Atunci lim ( li1
y-
multimea
0
din punctele
cu exceptia
/uO,
IYI < lxl <=} - lxl < Y < lxl <=} -x y ~ x, x > o
sau x ~ y ~ - x, x < 0
y
y =-X y =X
Domeniul de definitie este alcatuit din partea planului x o y
cuprins intre dreptele y = -x, y = +x i care contine dreapta Ox,
exceptand originea (por(iunea haurata).
x' -y'
2. Fief: IR \ {0, 0} , definita prin: f(x, y) = ~ ~
x' + y'
Sa se determine limitele iterate in origine i sa se cerceteze
existenta limitei funqiei in origine.
isolutie I
Fie <p (x) = lim f(x, y)
y->0
2
a ) x = O ~ lim f(x,y)= lim-y =lim (-1)=-1
Y-+0 Y-+0 y
2
y-+0
b) x "- 0 ~ lim f(x, y) = lim x: - Y: = lim ~ = lim 1 = 1
y-+0 y-t>O X + y y-+0 X
2
y-+0
Deci <p (x) = { -
1
x =
0
1, X;toO
Atunci lim (lim f(x, y)) = lim <p (x) = 1
x__,.O Y40 X-4-0
x.O
123
Fie' (y) = lim f (x, y). Distingem cazurile:
X->0
a) y = 0.
. - . x2- y2
lim f(x, y) - lim ,
2
X--J-0 X-40 X ~ + y
. - . x' -y2
b) y * 0. lim f(x, y) - lim
2 2
X-+0 X-+0 X + y
Aadar ' (y) = {-
1
' y =
0
1, y icO
. x2 .
= lim - = lim 1 = 1
X - ~ 0 X 2 X--J-0
2
= ~ =-1
y'
Rezulta ca lim (lim f(x, y)) = lim ' (y) = -1
y-tO X-+0 y---tO
YO
Aadar limitele iterate in origine exista, dar sunt diferite (iau
valorile 1 i -1), deci nu exista limita functiei in punctul (0, 0).
Daca aceasta limita ar fi existat, atunci ar fi existat i
limitele iterate i acestea ar fi fost egale.
Cele doua limite iterate exista, insa sunt diferite.
Pentru a arata direct ca nu exista limita functiei in origine,
vom considera iruri diferite, (xn), (Yn), ce converg Ia 0.
F
.
1
a IR*
1e Xn =-, Yn = -; a E
n n
1 (:( 2
Atunci
2 2 --- - ~ - 2
x -y n
2
n' 1-a .
f(Xn, Yn) = ~ ~ =
2
= --
2
==> lim f(Xn, Yn) =
X + Y 1 (:( 1 + (:( n->oo
n n ~ - + - - -
n2 n
2
1-a
2
=
l+a'
1-a
2
. 1-a' 3
Pentru a = 1,
2
= 0 I pentru a = 2,
2
- --
l+a l+a 5
Cele doua valori difera, deci functia nu are limita in (0, 0)
3. Sa se cerceteze existenta limitelor iterate pentru functia
I
f: IR x IR* -7 IR, f(x, y) = x cos-
y
124
lsolutie I
lim f(x, y) =
X-+0
Nu exista lin
Y-l
lim (lim f(x,
X-+0 y--'t-0
in aceasta s
4. Fie funqil
Sa se decidi
lsolutie I
Fie <p (x) = li
Y
Atunci lim (
X->0
Fie' (y) = I

Deducem: I
l
Exista cele '
are limita in punctu
5. Sa se ara
Y
'+
f(x,y) = -,-
Y -
= lim 1 = 1
-1
X->0
diferite (iau
{0, 0).
fi existat
in origine,
f{Xn, Yn) ::
3
': --
functia
I Solutie
lim f(x, y) :: lim
x-+-0 (
x. cos_l_J = 0 => (::3) lim (lim f(x, y)) = 0
y
Nu exista lim cos_!_, deci nu exista lim f(x, y) nici
y
lim (lim f{x, y)).
X-+0 y-J-0
In aceasta situatie functia f nu ad mite lim ita in origine.
4. Fie functia f: {0, oo)
2
IR, f(x, y) = x sin _I_+ _Y_
X X+y
Sa se decida existenta limitelor in punctul {0, 0).
lsolutie I
Fie <p (x) = lim f{x, y) = x sin _I_+ lim _Y_ = x sin _I_
X X + y X
Atunci lim (lim f{x, y)) = lim <p (x) = 0
X--+0 Y-+0 X-+0
Fie 'I' (y) = lim f(x, y) = lim x sin _I_+ lim _Y_ = 0 + Y = 1
X-+0 X-+0 X X-+0 X+ y y
Deducem : lim (lim f(x, y)) = lim ('I' (y)) = lim 1 = 1
Y-+0 X-+0 Y-+0 Y-+0
Exista cele doua limite iterate dar nu sunt egale, deci f nu
are limita in punctul {0, 0)
5. Sa se arate ca functia f: IR
2
\ {(x, y) 1 y
2
2x} IR,
f( )
y' + 2x
1
. .t_ . .
x,y = , nu are 1m1 a 1n ong1ne
y -2x
125
[ Solutie I
Vom considera (xn), (Yn) convergente Ia 0, de forma:
a 1
xn =-, Yn
n n
Pentru ca f sa fie bine definita,
impunem:
1 2a
7c-2X
0



n n
Tn perechea (Xn, Yn),
(pentru a = 0 este
adevarat; daca a* 0, consideram (xn) pentru n :2: N,
unde N = [L]+1)
1 2a

n' n
Atunci, f(Xn, Yn) =
1 2
a
1+2an
l-2an
n' n
Daca a = 0 f(Xn, Yn) = 1
n->oo
Daca a * 0 lim f(xn, Yn) =_}a = -1
n--)ooo - 2a
Pentru diferite care converg Ia 0 am obtinut valori
diferite pentru lim f(xn, Yn), deci f nu are limita Tn origine.
n--+CIJ
6. Sa se arate ca functia:
1
x
2
y
3
f( )
- 2 2 , x2 + y2 * 0
X, Y - X +y
0, x
2
+y
2
=0
este continua Tn tot planul.
lsolutie I
Pentru punctele (x
0
, y
0
) ce satisfac conditia + * 0, fie
x.

y"


0
arbitrare.
126
Deoarece x;
Sa presupur
Xn *- 0, (V) n
Atunci, pent1
f(Xn, Yn) =
'
spune ca f este em
Sa studiem
Fie s
if(xo,Yn)l =
Folosind im
numere reale a, b,
Dar X -
n
relatie ce exprima
7. Sa se ce
J sir
f(x,y) = l
de forma:
(Xn, Yn),
0 este
valori
;t 0, fie
Deoarece + ;t 0 Xo ;t 0, fie Yo ;t 0.
Sa presupunem x
0
;t 0 exista un rang N astfellncat
Xn ;t 0, (V) n 2: N
Atunci, pentru n ::: N avem: + * 0 deci:
x'y' x'y'
f(Xn, Yn) =
2
", ._,
00
2

0
2
= f(xo, Yo), relatie ce ne
Xo Xo +Yo
spune ca f este continua In (xo, Yo).
Sa studiem acum continuitatea in punctul (0, 0).
Fie arbitrare (xn), (Yn) convergente Ia 0.
= lxoi'IYol
3
= 2lxoiiYol ._!__., 11 I'
lxol
2
+IYl lxl +IYl
2
xo Yo
2 3
If( >1- XoY o
1 Xo' Yo - 2 2
Xo +Yo
Folosind inegalitatea 2ab :s: a
2
+ b
2
adevarata pentru orice
2ab
numere reale a, b, relatie echivalenta cu
2
, :s:l, deducem:
a +b-
if(xo,Yoll
Dar
xo
0->00
0,
Yo
n->oo
0
lxol
0->oo
0,
IY ol


0 limf(Xn,
Yn)
= 0 = f(O, 0),
0->oo 0-+00
0->00
rela\ie ce exprima continuitatea lui fIn (0, 0).
7. Sa se cerceteze continuitatea functiei
1
sin(x3 + y3) daca x' + y' ;t 0
f(x,y) = x
2
+ y
2
'
0, daca x
2
+y
2
=0
127
jsolutie I
In punctele (Xo. Yo) cu + * 0 functia este evident
continua.
Fie x. n->ro 0, y" n->ro 0 oarecare
sin(x' + y') + I
f(x.,y.)
2
" ," =>lf(x.,y.)l
2 2
s
x. +Y. lx.l +IY.I
lx.l' +iY.I'
s I I' I I' s I I' I I' slx.I+IY.I
Xn + Yn Xn + Yn
Am folosit inegalitatea lsinxl s lxl, adevarat pentru orice
x E IR, inegalitatea modulului, precum inegalitatea a
3
+ b
3
s (a
+b) (a
2
+ b
2
), valabila pentru V a, b pozitive.
Dar lim I+ IY" I) = 0 =>lim f(xn. Yn) exista este egala cu
n4oo n4oo
0 = f(O, 0). Cum au fest alese arbitrar, rezulta ca f e
continua 1n (0, 0).
B. Probleme propuse
1. Sa se determine domeniul de definitie pentru urmatoarele
functii, dand eventual interpretare geometrica:
x' y'
a) f(x, y) = 1----
a' b'
-{ x' y' }
R. Dmax-
b) f(x, y) = In (y- x2)
R. Dmax = {(x, y)/y-x2 > 0}
128
c) f(x, y, z)
R. Dmax =
d) f(x, y, z)
R. Dmax =(I
e) f(x, y) =
R. Dmax ={I
f) f(x, y) =
R. Dmax = [
g) f(x, y) =
R. Dmax = {1
h) f(x, y) =II
R. Dmax = {1
2. Sa se arate
f(x, y) = l
este continua 1n raJ
continua 1n (0, 0)
3. Sa se deci1
2X)
f(x, y) = x' +
4. Sa se calc1
a) Jim (x
2

X->0
y->0
R.O
este evident
. este egala cu
rezulta ca t e
urmatoarele
c) f(x, y, z) = arcsin
1
r==z==
-yx'+y2
R. Dmax = y, z) I-

+ Y
2
s Z s

+ y')
1 1 1
d) f(x, y, z) = .JX + jY + .JZ
R. Dmax = (0, oo) X (0, oo) X (0, oo)
e) f(x, y) =
R. Dmax = {(x, y) I X :? 0, y :? 0, x? :? y}
f) f(x, y) = .J4-x
2
1n(1-y
2
)
R. Dmax = [-2, 2] X (-1, 1)
g) f(x, y) = arcsin (x + y)
R. Dmax = {(X, y)lx + y + 1 :? 0, X + y- 1 :o; 0}
h) f(x, y) =In ('/- 4x + 8)
R. Dmax = {(x, y)loj > 4x - 8}
2. Sa se arate ca funqia:
f( )
-
2 2
, X +Y >'0
(
4xy 2 2
X, Y - X +Y
0, X= y = 0
este continua In raport cu fiecare variabila separat, dar nu este
continua In (0, 0)
3. Sa se decida existenta limitei functiei
f(x, y) =
2
2
xy
2
, (x, y) >'(0, 0), In origine.
X +Y
4. Sa se calculeze urmatoarele limite:
a) lim (x
2
+ oj) sin -
1
xy
y-+0
R.O
129
R. 2
1-cos(x
2
+y')
c) lim
x-.o (x2 +y')x' Y'
Y->0
R. Nu exista
5. Sa se cerceteze continuitatea urmatoarelor functii:
1
1- cos(x
3
+ y
3
) 2
2
0
f
' X +y *
a) (x, y) = x' + y'
0, X= y =0
R. Nu este continua In origine
b) f(x, y) =!(1 +

x > o, y > o
1, x=y=O
R. Este cOntinua pe domeniul de definitie
c) f(x, y) = {1- x
2
- y
2
, x
2
+ y' * 0
O, x' + y' = o
R. este continua pe IR
2
xy. --- daca (x, y) * (0, 0)
l
x'- y'
d) f(x,y) = x' + y
2
'
0, daca (x, y) = (0, 0)
R. Este continua pe IR
2
e) f(x,y)={(x' +y')ln(x
2
+y
2
), (x, Y)*(O,O)
0, daca x = y = 0
R. Este continua pe IR
2
f) f(x, y) = {x'ln(x
2
+ y
2
), daca (x, y) * (0, O)
0, daca (x, y) = (0, 0)
R. Este continua pe IR
2
130
4.2. Derivl
functi
Deriv;
Taylo
varial
A. Probleme
1. Fie functia f(
de Ia 1)
I Solutie I
.!_(1, 1) =lim f(J
ax x->1
1
=lim ln(
1
+ x)X:
X->1 2
. 1 1 +.
=lim--ln-
Y->1 Y -1 2
= ln[lim(1 + L
y->1 2
2. Fie funqia
calculeze derivatele p
0)
4.2. Derivate partiale. Diferentiabilitatea
functiilor de mai multe variabile.
Derivate de ordin superior. Formula lui
Taylor pentru functii de mai multe
variabile
A. Probleme rezolvate
1. Fie functia f(x, y) = ln(1 + xl). Sa se calculeze, pornind
de Ia definitie: 1 ), : (1, 1)
I Solutie I
1+X
In--

=lim f(x,1) -f(1,1) =lim ln(1 + x) -ln2


ox x->1 x-1 x-1 x-1
lim
2
X->1 X -1
I
_1 [ 2__]2 1
. 1 +X x-1 _ X -1 x-1 - 1
=lim In(--) =In 11m(1 + ---) =In e
2
=-
X->1 2 X->1 2 2

=lim f(1,y)-f(1,1)_ =lim ln(1+y


2
)-ln2
Eli y->1 y -1 p1 y -1
1
- 1 I 1+y' (1+y
2
)Y':,l
=1m-- n--= n 1m -- =
Y->1 y -1 2 y->1 2
2 -1 y'-1-,-
[
2 y+ll
=In = lne =I
2. Fie functia f(x, y) = .Jx' x , f: IR
2
\ {(0, 0)}. Sa se
x' + y'
calculeze derivatele partiale : (X
0
, y
0
), : (x
0
, Yo),
131
daca (xo. Yo) *- (0, 0)
lsolutie I
132
3. Sa se calcule;
urmatoarele func\ii:
sin!
a) f(x, y) = e x
b) f(x, y) =
c) f(x, y, z) = exz
I Solutie I
a) :(X0 ;Yo)=(

= -e x COS
Of ( sl
Oy(Xo,Yo)= e
I
=-e
Xo
sin.fu '
Xo COS"
'
'
Of (
b) 8x(Xo,Yo)=
1
= ------,===4
+ X
Yo
3. Sa se calculeze derivatele partiale de ordinul intai pentru
urmatoarele functii:
sin.Y
a) f(x, y) = e x
b) f(x, y) = +
c) f(x, y, z) =

_+_y_
2
lsolutie I
slnYo Y y2
= -e x cos_. _2_
2
Xo Xo
133
Xo
XoYo+-
Yo
2
XoYo-Xo
: (Xo,Yo) = [ )XoY + }IY=Yo =
; ( '
2
2Ro Yo XoYo +--
Yo
c) : (X
0
, Y
0
, Z
0
) = (ez,x )x
2
+ Ydlx=x, =
= z ez,x 'x2 + y2 + 2x . ez,xl =
o V o



+ x=x,
ez., (z X
2
+ z y
2
+X )
= 00 00 0

Of (x y . z ) = fex,z, [x2 + y2 )I = e"'"" . Yo
::A, 0' 0' 0 V 0 Y=Yo J 2 2
vy -vxo +Yo
4. Sa se cerceteze daca funqia f(x, y) = )x
2
+ y
2
este
diferentiabila in origine.
lsolufie /
0 conditie necesara pentru ca o functie sa fie diferentiabila
intr-un punct o reprezinta existenta derivatelor paftiale in acel
punct.
Sa vedem, de exemplu, daca exista : (0, 0)
lim f(O, y)- f(O,O) =lim Jyz -
0
=lim hi= -I
y-.0 y - 0 Y->0 y y-.0 y
y<O Y<O Y<O
134
I i m -'f (,__0'-'-,y"-) -_f_,_( o_,.c
y->0 y-0
y>O
Cele doua lirr
derivabila partial in ra
este diferentiabila in c
5. y)l
Sa se arate ci
partiale in origine, ins
I Solutie I
Fie (x, y) e IR
2
,
0 :.:; [f(x, y)[ = .p
1 1 2 '
=--vx +Y =
2
=> f este conti1
2ab s a
2
+ b
2
).
Sa cercetam e:
I
. f(x,O)- f(O,'
1m -'-'-'----:'-1
x->0 X- 0
in mod analog,
Of 0) ..
exista -(0, v
fJy
Sa presupuner
Exista atunci
valoarea 0 in punctul
+ y' este
lim f(O, y)- f(O,O) =lim .JY2-
0
= limh'[ = 1
y->0 y - 0 y->0 y y-+0 y
y>O Y>O Y>O
Cele doua limite nefiind egale, deducem ca f nu este
derivabila partial in raport cu variabila yin punctul (0, 0), deci f nu
este diferentiabila in origine.
. xy , daca (x, y) * (0, O)
5. F1e f(x, y)= xz + y2
0, daca (x, y) = (0, 0)
Sa se arate ca f este continua in origine, admite derivate
partiale in origine, insa nu este diferentiabila in origine.
Jsolutie J
Fie (x, y) e IR
2
, (x, y) (0, 0)
l
xyl 1 xz + yz
0 if(x, Yll = < - =
+ yz - 2 + yz
=

_+_y_
2
=> limf(x,y) = 0 = f(O,O) =>
2 X->0
y-.o
=> f este continua in punctul (0, 0) (am folosit inegalitatea
2ab s a
2
+ b
2
).
Sa cercetam existenta derivatelor paf"\iale in origine:
lim f(x,O)- f(O, 0) = lim 0- 0 = 0 => 0) = 0
xO X - 0 x-.0 X Ox
f(O y) - f(O 0)
In mod analog, lim ' ' = 0, de unde deducem ca
y-+0 y-0
exista (0, 0) i ia valoarea 0.
iN
Sa presupunem ca f ar fi diferentiabila in origine.
Exista atunci o functie

continua i care ia
valoarea 0 in punctul (0, 0) astfel incat:
135
f(x, y)- f(O, 0) = 8f {0, 0) x + 8f {0, 0) y + w{x, y). x
2
+ y
2
=>
Ox 8y o(
=> w(x, y) = f(x, y) = xy , (\i){x, y) * {0, 0) M 2.
1 2 2 xz +Yz _
-vx +y 1K
2
Dar functia ro{x, y) nu are limita in origine (este safii::ient sa
consideram l?iruri de forma Xn =.!_, Yn = a, a E IR, deci f nu este
n n
diferentiabila in origine.
0 conditie suficienta ca f sa fie diferentiabila in punctul {0,0)
este sa existe derivatele partiale ale lui f p_e o vecinatate a
punctului {0, 0) l?i sa fie continue in (0, 0). lntrucat f nu este
diferentiabila in origine l?i deoarece exista derivatele partiale ale lui
f pe tot IR
2
, deducem ca ori 8f , ori 8f nu este continua in (0, 0)
fJx 8y
(aceasta se poate observa printr-un calcul direct).
! xz _ yz
6. Fief: IR
2
-tiR, f(x, y) = xy xz + Yz daca {x, Y) * (0, 0)
0, daca (x, y) = (0, 0)
a) Sa se calculeze derivatele partiale de ordinul I. Este f
diferentiabila in origine ?
b) Sa se calculeze derivatele partiale de ordinul II in origine
l?i diferentiala de ordinul II a functiei fin origine.
/solutie /
a) Fie (Xo. Yo) E IR
2
, (xo.Yo) * (0, 0)
136
Tn mod an
Fie (xo.Yo)
8f
Ox (0,0) =
8f
fly (0,0) =
Al?adar:
l?i : (x,y)
Fie (x, y),
l:(x,y)l=
2x
4
:>IYI-l
=> lim
(x.y)->(0.0)
Tn mod an
Deducem
este diferen(iabil
Of ( x' - y
2
J
In mod analog, --(X
0
, Y
0
) = X
0
Y
0
2
lv=v =
fJy x; + y "
x
4
- 4x'y' - y
4
-x.o oo o
- 0
(x;

Fie (xo,Yo) = (0, 0) ==>
= lim f(x, O)- f(O, O) = lim
0
-
0
= 0
Ox X->0 X - 0 X->0 X - 0
=lim f(O, y)- f(O, O) =lim
0
-
0
= o
fJy y->0 y - 0 Y->0 0 - Y

(
x
4
+ 4x'y
2
- y
4
Of y . -----;,------'c;;--:;-''-
Ox (x,y) = (x' + y')'
0,
. Of X
(x, y) (0, 0)
(x, y) = (0, 0)
(
x
4
- 4x
2
y
2
- y
i?l -(x, y) = (x
2
+ y
2
}
2
8y 0,
(x, y) (0, 0)
(x, y) = (0, 0)
Fie (x, y) (0, 0). Atunci
I
Of x I= .lx' + 4x'y2 - y'l < . x' + 4x'y' + y' <
,.,< ,y) !Y! < 2 'l' -!Y! < 2 ')' -
Uh X + y X + y -
!Y! 2x' + 4x'y2 : 2y' = 2!Y! (x' + y2 l' = 2!Y! ==>
(x'+y')- (x'+y
2
}
2
==> lim Of (x,y) = 0
(x,y)->(0,0) fJx
Tn mod analog, rezulta: lim Of (0,0} = o
(x,y)->(0,0) fJy
Deducem ca functiile : : sunt continue Tn origine deci f
este diferentiabila Tn origine.
137
8f 8f x4
a't . ay(x,O)- ay(O,O) . X- -O
b)
--(0,0) =lim =lim x
4
=
axay x-.o x - o x->o x
I
. X
=1m-= I
X
8f 8f (-y4J
2 -(O,y)---(0,0) Y --.- -0
=lim ax ax = lim-"-'-Y--'--- =
ayax Y->0 Y - 0 Y->0 y
-y
=lim-=-!
y->0 y
8f 8f
a't -(x,O)- -(0,0)
0
-, (O,O) =lim ax ax =lim-= o
ax x-.o x - o x ... o x
8f 8f
2 -(0, y)- -(0,0)
=lim ay ay 0
8y
2
Y->0 y - 0 Y->0 y
a't a't a't
d
2
f = -(00) dx' + --(00) dx dy + --(00) dy. dx +
ax' ' axay ' ayax
+ (0,0) dy
2
= 0 dx
2
+I dx dy -I dy dx + 0 dy' = 0
7. Fie o functie derivabila. Sa se arate ca functia

Z (x,y) = <p(x2 + /) verifica ecuatia:


az az
y--x- =0
ax ay
I Solutie I
az . 2 2 au ' 2 2
-(x, y) = <p (x + y ) -(x, y) = 2x <p (x + y ),
ax ax
unde am notat prin u(x,y) = x
2
+ y
2
.
138
az.
Analog -(x,y)
ay
Atunci:
az '
y ax(x,y)-x;
8. Fie <p: IR
2
I
ca functia 'f':


satisface relatia:
a'f' mv
XZ---+
ax ay
lsolutie I
Sa notam u(x, y
8' iJ<p
ax(x,y,z)= au
= y . ocp (u, v) + 2
au
a' acp
fij(x, y,z) = au
= x. acp (u, v) + 2
au
a' (x, y, z) = iJ<p
az au
= -2z acp (u, v)
av
Obtinem, in fina
a'f'
XZ--YZ
ax
= x. y. z iJ<p (u,
au
+ (-y' + x')(-2:
y
ca funqia
oz 2 ,au 2 2
Analog +y ),
ay ay
Atunci:
az iJZ ' 2 2 ' 2 '
Y ax (X, y)- X 8y (X, y) = y 2X <p (X + y ) -X 2y <p (X + y-) = 0,
8. Fie q>: IR
2
IR o functie diferentiabila pe IR
2
. Sa se arate
ca funqia ':

definita prin '(x, y) = q>(xy, + y


2
- z
2
)
satisface relatia:
x. z a':P - aiJI + (x
2
- y
2
= 0
ax ay az
I Solutie I
Sa notam u(x, y, z) = xy, v(x, y, z) = + i-z
2
a':P (x, y,z) = a<p (u, v)' au (x, y,z) + 8<p (u, v)' av (x, y,z) =
ax au ax av ax
a<p 8<p
= Y au (u,v)+2X av (u,v)
a':P a'P au a<p av
-;;y(x,y,z)= au(u,v) ay(x,y,z)+ av(u,v) ay(x,y,z)=
(u,v) + 2y (u,v)
8'P a<p au a<p av
y,z) = -(u, v) -(x, y,z) + -(u, v) -(x, y,z) =
iJZ au az av az
a<p
= -2z' av (u, v)
Obtinem, in final:
x z a':P - y z a':P + (x
2
- y
2
). a':P =
ax ay oz
8<p( ) 2 a<p( ) 8<p( ) 2 a<p
=XYZ au u,v +2x zav u,v -xyzau u,v -2y z. av(u,v)+
+(-y
2
+X
2
)(-2z)a<p(u,v) = 0
av
139
9. Fie g: IR
2
---?IR o functie de clasa r/- (derivabila de ordinul
I II cu derivatele partiale de ordinul II continue) ce satisface
ecuatia lui Laplace:
a'g a'g 2
(x, y) + -
2
(x, y) = 0, (\f) (x, y) E IR
ax ay
Sa se arate ca functia f: IR
2
\{0, 0}---?IR,
f(x, y) = g(
2
x
2
,
2
y
2
J satisface, de asemenea,
X +y X +y
ecuatia lui Laplace.
I Solufie I
Fie u(x, y) =
2
x
2
, v(x, y) =
2
y
2
X +y X +Y
at ag au ag av
ax (x, y) = au (u, v) ax (x, y) + av (u, v). ax (x, y) =
8g Y'-x' 8g ( 2xy J
= v) + -(u v) -
au ' (x
2
+ y
2
)
2
av ' (X
2
+ y
2
)
2
a't a (agJ y' -x' 8g a ( y' -x
2
J
ax' (x, y) = ax 8u (u, v). (x2 + y' )2 +au (u, v). ax (x' + y
2
)
2
+
J+ ;( J=
a'g au y' - x' a'g av y' - x'
= au' (u, v). ax (x, y). (x' + y' )2 + avau (u, v). ax (x, y). (x
2
+ y' )
2
+
8g - 2x(x
2
+ y' )
2
- 2(X
2
+ y' )2x(y
2
- X
2
) a'g au
+-(U,V)
au (x' + y' )' auav ax
(
2xy J a'g av ( 2xy J a9
' 2' +-, (u,v)-(x,y) , , , +-(u,v)
(x + y ) av ax (x + y ) av
- 2y(x' + y
2
)
2
+ 4x(x
2
+ y
2
) 2xy
=
(x'+y')'
140
a'g (y'- x')' '
+-
- au' (x' + y')' i'.
a'g ( 2xy(y'- x
+ auav - (x' + y')'
Se calculea.
a't a't
ax2 (x, y) + 8y2 (x, y)
(folosind faptul ca
(
10. Sa se eli
<p 'P de clasa 'ff
a) Z(x,y)= cp(:
b) Z(x, y)= cp(
c) Z(x,y)= cp(:
I Solufie I
az .
a) - = cp (x)
ax
b) az = cp'(x
ax
az.=q>(X)q
8y
a'z .
-=cp(x)
axay
de ordinul
ce satisface
asemenea
'
Y
2 2
-x
(x' + y' )' +
(u,v).au
Ox
a'g (y' - x' )
2
a'g ( 2xy(y' - x' >) ag x' - 3y'
(X
2
+ y' )
4
+ avau .. - (x' + y' )
4
-au . lx (x' + y
2
)
3
+
a'g ( 2xy(y'- x' >) a'g 4x'y' ag 3x'- y'
+auav- (x'+y')' +av, (x'+y')'-+av
2
Y(x'+y')'
a't
Se calculeaza Tn mod analog ay' (x, y) l?i se va obtine
a'f a'f
-
2
(x,y) + -
2
(x,y) = 0
Ox Oy
a' a'
(folosind faptul ca ___!,! (u, v) + ___!,! (u, v) = 0)
au av
10. Sa se elimine, prin derivari succesive, functiile arbitrare
cp 'I' de clasa '(;!- pe I R:
a) Z(x,y)= <p(x)+ 'I'(y)
b) Z(x,y)= <p(X) 'I'(y)
c) Z(x,y)= cp(x+y)+ 'I'(x-y)
lsolutie I
Oz. . a'z
a)
Ox OyOx
Oz. .
b) - = q> (x) 'f'(y)
Ox
Oz. .
- = q>(X). 'f' (y)
Oy
a'z . .
- = q> (X). 'f' (y)
axay
141
Oz. Oz. ' '
Rezulta ca - -= <p (x) 'I'(y) <p(X) ' (y) =
axay
. . a'z
= <p(x) lj!{y) <p (x) 'I' {y) = z -
axay
az az a'z
--=Z--
, ax ay axay
Oz. ' '
c) ax = <p (x + y) + 'l' (x- y)
a'z .. ..
ax' = <p (x + y) +'I' (x- y)
Oz. ' '
ay = <p (x + y)- 'I' (x- y)
a'z .. .. .. ..
ay' = <p .(x + y)- (-' (x- y)) = <p (x + y) + 'l' (x- y) :::::>
a'z a'z
:::::>- =-
ax' ay'
11. Sa se calculeze diferentialele de ordinul I II pentru
functia F: IR
2
--> IR, F(x, y) = + I. -1. 2xy)
unde f: IR
3
--> IR, este o functie diferentiabila de doua ori
J
Fie u{x, +I. v{x, y) = ro(x, y) = 2xy. Atunci:
aF at au at av at am at at at
-{x,y) =--+--+ --= 2x- + 2x-+ 2y-
Ox. au ax av ax aro ax au av aro
aF at au at av at aro at at at
-{x,y) =--+--+--- = 2y--2y-+ 2x-
ay au ay av ay aro ay au av aro
142
Diferentiala de or
aF aF
dF=-dX+-
Ox. fJy
+(2y at
au av
Derivatele partial
a'F a (c
Ox.' (X, y) = ax B
+ :(:}:+
at ( EJ
2
1
2X+2-+-
au BuB
+ (_.'_!__ . au +
auaro ax av
, a't
4
,
= 4x -+ x -
au' i
a't
+ 4xy --- + 4xy
aroav
a't '
= 4x
2
-+4x'-
au' i
a't at
+8xy--+2-
avaro BL
(y) =
'f'" (x- Y) ==>
I II pentru
ori
2xy. Atunci:
2xE!f +2y-"!
&v 00)
'2f)f Of
- Y-+2x-
&v 00)
Diferentiala de ordinul I a functiei F se poate scrie:
dF =oF =(2x Of +2X Of
ax ay au av ow
(
Of Of OfJ
+ 2y--2y-+2X- dy
au av aw
Derivatele partiale de ordinul II ale functiei F sunt
+a'v .. Of +
ax' ' ax au ax ax' au ax av ax ax' av

ax ow ax ax' oro au' ax avau ax awau ax
2X+2-+ ---+--+--- 2x+2-+
Of ( a'f au a'f av a'f awJ Of
au auav ax av' ax awav ax av
av + a't. oroJ
2
y+O Of=
auaro Ox av oro Ox oro
2
Ox oro
, a't , a't a't , a't , a't
=4X -+4X --+4xy--+4X --+4X -+
au' avau aroau auav av'
a't a't a't a't Of Of
+4xy--+ 4xy---+4xy-- +4y
2
--+ 2-+2- =
ow iJv ouoro iJv oro oro
2
au iJv
, o
2
f , a't 2 a't , a't a't
=4X -+4X -+4y --+8X --+8xy--+
au' av' oro' auav auav
a't Of Of
+Sxy--+2-+2-
avaw au av
143
B'F (x y) 8f . a'u + ()v + a'v +
ay' ' ay au ay au ay' ay av ay av 8y
2
a'w =
2
Y( a'f . av + __!}___'.!__. aw) +
8y aw 8y aw ay' au' 8y avau 8y awau 8y
+
2
at

au+ a't. av_+ a't .


au auav 8y av' 8y awav 8y 8v

av + a't. aw)+O at =
auaw ay avaw ay aw' ay aw
, a't 2 a't , a't , a't a't
=4y -+4y -+4x ---By --+8xy---
au2 av' aw' auav auaw
a't at at
-Bxy--+2--2-
avaw au av
a'F = _!}___( 8f). a'u + _!}___( 8f). 8v + 8f . a'v +
axay ax au ay au axay ax av ay av axay
+_!}___(at). aw a'w = (a'f. av aw).
ax am ay am axay au' ax avau ax awau ax
(
a'f au a'f 8v a'f aw)
-2y+0+ ---+--+---- (-2y)+0+
auav ax av' ax awav ax
+ a't _aw)
2
x+
2
at =
4
xya't _
auaw ax avaw ax aw' ax aw au'
a't a't , , a't 2 , a't 2at
-4xy--+4xy-+4(x +Y )--+4(x -y )--+-
av' aw' auaw avaw aw
Pentru a afla acum diferentiala de ordinul II utilizam
a'F a'F a'F
formula: d'F = -
2
dx' + 2--dxdy + -
2
lnlocuim
ax axay ay
derivatele paftiale cu rezultatele gasite.
12. Sa se calculeze derivatele partiale de orice ordin,
precum diferentialele de orice ordin pentru functia
f: IR
2
--7IR, f(x, y) = eax+by; a, bE IR
144
[solutie I
8f
ax (x, y) =a'
df = aeax+by
a't
-, (x,y)=
ax
a't
-
2
(x,y)=l
8y
Prin induC\it
a"t
axayn-k (x
Rezulta ca:
n
d"F = L(
:::: eax+by . ,
I
= eax+by I
Diferentials
este: d"F((x, y);(x
13. Sa se
punctul ( 1 , 1) pen
I Solutie I
Avem, sue
8f
ax (x,y) =
+
Ot
28f
aw
II utilizam
lnlocuim
ordin,
I Solutie I
: (x, y) = aex+by; : (x, y) = be ax. by
df = aeax+by. dx + beax+by dy = ex+by (adx + bdy
a't a't
- (x y) = a
2
eax+by --(x y) = abex+by
Ox2 ' 'EJx&y '
a't
&y2 (x, y) = b2eax+by
Prin inductie, se arata imediat ca:
a"t
-;,---:-;:- (x y) = ak . b"-keax+by
Oxk&yn k '
Rezulta ca:
d"F = a"f -dxk -d n-k =
n Y
k""O VA vy
n
= L . dxk . dyn-k =
k-oO
n
= eax+by . :L . (adx)k . (bdy)"-k =
k=O
= eax+by . (adx + bdy)"
Diferentiala de ordin n In punctul (xa. Yo). ca/culata In (x, y)
este: d"F((x, y);(x
0
, y
0
)) = exo+by"+cz" [a(x- X
0
) + b(y- y
0
)]"
13. Sa se scrie polinomul lui Taylor de gradul a/ treilea In
punctul (1, 1) pentru funqia f:(O, oo) x (0, co)-;/R, f(x, y) = xY
lsolutie I
Avem, succesiv:
8f 8f
--(x, y) = yxY-
1
;-(x, y) = xY lnx
Ox &y
145
a't a't 1
-
2
(x, y) = y(y --(x, y) = yxY-
1
In x + xY . - =
ax ayax x
=

+ 1)
a't
-
2
(x, y) = xY (lnx)'
ay
a't
ax' (x, y) = y(y -1){y- 2)xY-
3
a't a
-
2
-(x, y) =-{y-In x xY-
1
+ xY-
1
) = y xy-z + y(y -1)xY-'Inx +
axay ax
+ (y -1)xY-'
a'f a 1
--, (x, y) = -[xY (lnx)'] = y xY-
1
(lnx)' + xY. 21nx - =
axay ax x
=Xy-
1
-lnx[y-lnx+2]
a't
ay' (x, y) = y (y -1) (y- 2) xY-
3
Tnlocuind (x, y) prin (1, 1 ), obtinem:
ar ar a'f
ax (1, 1) = 1; ay (1, 1) = o; ax' (1, 1) = o;
a'f a'r
-(1,1)=1;-, (1,1)=0;
ayax ay
a'f a'f
-
3
(1,1) = 0;-
2
-(1,1) =I;
ax axay
a'f a'r
--, (1,1)=0;-3 (1,1)=0;
axay ay
Polinomul Taylor de gradul Ill ataat functiei f, In punctul
(1, 1) are forma:
146
1 [ o
3
f
T,(x, y) = 3! i3x3 (
o
3
f
+3--
2
(1,1)(X
axay
1 [a'f
+ 2! ax' (1,1). (x-
Of
+ax (l,l)(x-1)+
2
+-(x-l)(y-1)+
2!
B. Probleme J
1. Pornind de Ia d
Of Of
a) -(4,1), -(4,
ax ay
R _!
. 3' 3
a't
b) -(2,2), pe
axay
R.I.
9
c)
-
'
x +sin' y
J21
R -- -
. 2 ' 2
. 1
txY - =
X
-l)xY-' In x +
1
-21nx-=
X
f, in punctul
1 1 o
3
f o
3
f
T,(x,y)=
3
Jlilx
3
(1,1)-(x-1)
3
+3 ilx
2
Dy (1,1) (x-1)
2
(y-1)+
+3---(11)(x-1)(y-1)
2
+-(11)-(y-1)
3
+
o
3
f a't J
ilx()y2 ' Dy' '
1 I a't a't a't J
+
21
lOx:' (!,!) (x -1)
2
+ 2 ilx()y (1,1) (x -1)(y -1) + Dy' (1,1). (y -1)
2
+
3
+ ilx (1,1) (x -1) + Dy (1,1) (y -1) + f(l,l) =
31
(x -1)
2
(y -1) +
2 1 2
+ -(x -l)(y -1) + (x -1) + 1 = -(x -1) (y -!) + (x -1)(y -1) + x
2! 2
B. Probleme propuse
1. Pornind de Ia definitie, sa se calculeze:

a) -(4,1), -(4,1)pentru f:D-tiR, f(x, y) = ln(x-/);
Ox Dy
D = {(x, y) E IR
2
/x > /}
1 2
R. 3' -3
o
2
f
b) --(-2,2), pentru f: IR
2
-tiR, f(x, y) = Vx'y
ilx()y
R._l_
9
c) :(:.:} pentru f: IR
2
-t IR, f(x, y) =


x+sin
2
y
R fi _1
. 2 , 2
147
2. Sa se calculeze derivatele paf\iale de ordinul Tntfli doi
pentru urmatoarele functii:
a) f(x, y) = (x2 + Vl arctg x, y to 0
y
b) f(x, y) = ln(x + .J'x-,-
2


);x>O, y to 0
c) f(x, y) = ln(x + LJ; x"' 0, x + l to 0
2x 2x
d) f(x, y) = arcsin x' - y' ; x2 + l ::::: 1
e) f(x, y, z) = ex'+y' sin' z
f) f{x, y, z) =ln(0 + y
3
+ z
3
- 3xyz)
g) f(x, y, z) = exz

+ y
2
h) f{x, y, z) = arctg l
xz
i) f{x, y, z) =xY + yz- 2zx; x > 0, y > 0, z > 0
3. Sa se arate ca functia f(x, y) = arctg '!_ satisface ecuatia
X
. a't at
lu1 Laplace, -
2
+ -
2
= 0
ax ay
4. Sa se calculeze
a
3
t . 8
3
t
ax
2
8y axay2 pentru functia f(X, y) =
cos( ax+ eY)
8
3
f
5. Sa se calculeze
2
axay
pentru f(x, y) =
X
4
-8xy
3
X-- 2y
8
3
f
6. Sa se calculeze
2
pentru f{x, y) = arctg xy
ayax
148
7. Fie f(x, y) =
a't a't ,
relatia: -+--+
ax' axf)y 1
8. Sa se arate
f derivabila verif
9. Sa se arate
f, g derivabile 1
, a'F i
X -+xy-
8x2 a
10. Sa se
au au = 0 atune
2 + 2 '
ax fJy
x2+ltoOsat
11. Sa se ara
satisface relatia: x
este o functie deriva
12. Daca F (x
f diferentiabili
1ntai doi
ISf.ace ecuati a
f(x. y) =
7. Fie f(x. y) = _!'!__; x y. Sa se a rate ca este adevarata
x-y
. 8
2
f 8
2
f 8
2
f 2
relat1a: - + +-=
. Ox2 OxOy Oy2 X- y
8. Sa se arate ca F(x. y) = + y' -a)- cu x 0
fd
. b'l" .fi - t iJF iJF +Y' ( )
enva 1aven1caecua,1a:x--+y-= Fx,y
iJx iJy -a
9. Sa se arate ca F(x, y) = x f(y) + cu x 0, y 0,
y y X
f, g derivabile de doua ori verifica ecuatia:
o'F o'F oF
x' 0
Ox
2
iJxiJy Ox
10. Sa se arate ca daca u(x, y) satisface ecuatia
8
2
U 8
2
u t . . f
1
. ( ) _ ( X Y )
- + -
2
= 0. a unc1 unc,1a v x, y - u
2 2
,
2 2
cu
Ox
2
Oy X + y X + y
-2 + / 0 satisface ecuatie.
11. Sa se arate ca functia v (x. y) = yu ( y . x
2
; L} x 0
satisface relatia: x(x'+y
2
): +2y'(x: +y: -v)=o unde u
este o functie derivabila
12. Daca F (x. y, z) = x cu 0, k E IR
f d
.f . bl- t . iJF iJF iJF kF( )
1 erent1a 1 a, a unc1 x iJx + y iJy + z iJz = x, y,z
149
13. Fie '1': IR
2
---7IR, f(x, y) = x cp(x + y) + y 'l'(x+y), unde cp,
'I' sunt derivabile de doua ori. Sa se arate ca are loc relatia:
a't a't a't
--2--+-=0
ax' axay ay'
14.Aratati ca funqia
f( )
1
2
2
xy
2
, daca x
2
+ y
2
"' o
x,y = x +y
0, daca x = y = o
admite derivate partiale in punctul (0, 0), ! (0, 0) i ~ (0, 0), d e ~ i
funqia este discontinua in aces! punct.
15.Sa se arate ca, d e ~ i funqia:
f( )
l(
x
2
+ y
2
)sin
2
1
2
, pentru x
2
+ y
2
;t 0
x,y = x + y
0, pentru x = y = 0
are in vecinatatea punctului (0, 0) derivate partiale discontinue in
punctul (0, 0) ~ i nemarginite in orice vecinatate a acestui punct,
t o t u ~ i ea este diferentiabila in punctul (0, 0).
16. Fief: IR
2
---7IR, f(x,y) = J y'ln(l + ;: J Y "'
0
l 0, y = 0
Sa se arate ca a'f (O O) = a't (O O) dec:.i nu sunt
axay ' ayax ' ' "
satisfacute criteriile lui Schwarz ~ i Young
150
17. Sa se ara1
admite derivate par
acest punct.
18. Sa se calc
~ + y
2
+ z
2
), unde f l
19. Sa se calc
a) d
3
f, daca 1
b) d2t, daca 1
c) d2t, daca 1
d) d"f, daca 1
e) d"f, daca 1
f) d"f, daca fl
20. Sa se dE
formulei lui Taylor, i1
sa se gaseasca rest1
21. Sa se dE
punctului (0, 0), pan<
22. Sa se de:
jurul punctului (1, 1,
Y 'P(x+y), unde cp,
loc relatia
..
.Of
f' ay (0, 0),
discontinue in
a acestui punct,
nu sunt
17. Sa sea rate functia f(x, y) =
daca x = 0 sau y = 0
daca x.., o, y * o
admite derivate partiale in origine, dar nu este diferentiabila in
acest punct.
18. Sa se calculeze dF d
2
F pentru F(x, y, z) = f(x + y + z,
i- + I + z
2
), unde f admite derivate partiale de ordinul II continue
19. Sa se calculeze diferentialele de ordinul indicat:
a) d
3
f, daca f(x, y) =

b) d2t, daca f(x, y) = ln(x- y); x > y
c) d2t, daca f(x, y, z) = + 21 + 3z
2
- 2xy + 4xz + 2yz
d) d"f, daca f(x, y, z) = ex +by+ cz
e) d"f, daca f(x, y, z) = cos (x + 2y + 3z)
f) dnf, daca f(x, y, z) = In (ax+ by+ cz), ax+ by+ cz > 0
20. Sa se dezvolte functia f(x, y) = ln(x - y) cu ajutorul
formulei lui Taylor, in punctul (0, -1), pana Ia termenii de ordin II
sa se gaseasca restul corespunzator.
21. Sa se dezvolte functia f(x, y) = e
2
x ln(1 + y) in jurul
punctului (0, 0), pana Ia termenii de ordin Ill.
22. Sa se dezvolte functia f(x, y, z) = x
3
+ l + z
2
- 3xyz in
jurul punctului (1, 1, 1 ), pana Ia termenii de ordin II.
151
4.3. Extremele functiilor de mai multe variabile.
Extreme cu legaturi.
A.1. Probleme rezolvate
1. Sa se determine extremele functiei f: IR
2
---* IR,
t( x, y) = x' + y' - x' - y'.

f admite derivate partiale de ordinul II continue pe IR'. Sa
determinam punctele stationare ale functiei. Acestea se gasesc
rezolvand sistemul:
Of
ax=O {4x'-2x=O {x(2x'-1)=0
at =O y{2y'-1)=o
fJy
Rezulta urmatoarele puncte din plan: (0,0), (0,- ), (0, ),
1 11 11 1 1111
(- Ji ,0), (- J2 ,- J2 ), (- J2 ,+ J2 ), ( J2 ,0), ( J2 ,- J2 ), ( J2, J2 ).
Fie
a't a't
ax,(a,b) axfJy(a,b) 12a'-2 0 2 2
L'1 = a't a't = , = 4( 6a - 1 )( 6b - 1).
fJyox (a, b) fJy' (a, b) I o 12b - 2
1) (a, b)= (o,o) t1 = 4 > o (o,o) este punct de extrem;
a't
ax' (o,o) = -2 < o (o,o) este punct de maxim.
!52
2) (a,b)=(o.-
de extrem (este p1
3)
nu este punct de I
4) (a, b)= (- r
5) nL
(
1 1
6) (a,b)= - J2'--:J
a't (
extrem; ax' -
minim.
(
1 1
7) (a, b)= - J2' .[2
a't ( 1 1 )
ax' - J2' J2 ,
in mod analog ded
sunt puncte de mini!
2. Sa se
f(x, y) = (x+ y)e+'+l
olutieJ
f admite deri
Aflam punctele stat
varia bile.
f:IR'---+ IR,
pe IR'. Sa
se gasesc
-1)(6b
2
-1).
2) (a,b)=(o.- )z) =:>Ll=4(-1)2<0=:>(o.- )z) nu este punct
de extrem (este punct l?a).
3) (a,b)=(o . .Jz)=:>Ll=4(-1)2<0, deci nici punctul (o.]2)
nu este punct de extrem (este punct l?a).
4) (a, b)= (- ~ .o) nu este punct de extrem.
5) (a,b)=(.Jz.o) nuestepunctdeextrem.
6) (a, b)= (- .Jz ,- )z) =:> Ll = 8> 0 =:> (- .Jz ,- )z) este punct de
8
2
f ( I I ) ( I 1 )
extrem; ax' - 12,- 12 = 4 > 0 =:> - 12,- 12 punct de
minim.
7) (a, b)= (- .Jz, )z) =:> t\ = 8 > o;
a't ( 1 1 ) ( 1 1 J
ax' - 12'12 =
4
=:> - 12'12
punct de minim.
A (' 1) ,(1 1)
In mod analog deducem ca l?i punctele
12
.-
12
l?l
12
.72
sunt puncte de minim.
2. Sa se determine extremele func\iei f: IR
2
---+ IR,
f(x,y)=(x+y)e-(x'+v').
olutie.l
f admite derivate par(iale de ordinulll continue pe IR'.
Aflam punctele stationare rezolvand sistemul:
153
at
ax=
0
\e-(><'+v') -2x(x+y)e{x'+v') = 0 {1-2X
2
-2xy = 0
at =0 <=> e-(x'+v') -2y(x+y)e-(x'+v') =0 <=> 1-2xy--2y' =0
ay
Scazand cele doua ecua\ii ob\inem x
2
= y' <=> ( x- y )( x + y) = 0.
a) x- y = 0 => x = y => 1- 2x
2
- 2x
2
= 0 => 4x
2
= 1 => x' = de
- 1 1
unde rezulta x = y = - sau x = y = --.
2 2
b) x + y = 0 => y = -x => 1- 2x
2
+ 2x' = 0- contradic\ie. Deci
punctele sta\ionare sunt punctele (
Sa calculam derivatele par\iale de ordinulll.
= -2xe-(><'+l) -(4x+2y)e-(>'+v') +4x'(x+y)e-(><'+v') =
= 2(2x
3
+ 2x'y- 3x- y )e -(>'+v')
a'f (x'+ ') (x'+ ') (x'+ ')
Y -2xe- Y +4xy(x+y)e- Y =
axay
= 2e -(x'+y')(2x'y + 2y'x- 2x- 2y)

a'f (x'+Y') ( ) (x'+y') ( ) lx'+y')
ay' = -2ye- - 4y+2X e- +4y
2
x+y e' =
=2(2xy' +2y
3
-3y-x)e-(x'+Y')
a't a't
. ax' axay
Fie A = a'f a'f
ayax ay'
154
3
= 4.Je1
2 2
a't (-_1_ _ _1_\
ax' 2 2) 2
ii)
_1_ I =
ax' 2 2) 2
3. Sa se ga!
f: ( o, w) x (o,oo) x (O,o
y' 2
f(x, y,z) = x+
4
x +-
olutie1
f admite der
de defini\ie.
y'
at -0 1--.
ax- 4x
at =0<=> j_ __
0y 2X '
Of = O 2Z
az Y
Din prima ecua\iE
-2xy 0

' I
x de
4'
Deci
=
3 I
= 4-.Je
2 2
= s..Je > o
I 3
2 2
I
- -+ 2
2 -4-.Je =
I I
----+2
4 4
a'f (- = > 0 => punct de minim.
ax' 2'2 2 2'2
ii) (a, t. = s..Je > o
a'f = < 0 => punct de maxim.
ax' 2'2 2 2'2
3. Sa se gaseasca extremele functiei f,
f:(o,oo) x (o,oo) x (o,oo) IR ,
y' z' 2
f(x,y,z) = x+-+-+-.
4x y z
olutieJ
f admite derivate partiale de ordinulll continue pe domeniul
de definitie.
at y'
-=0 1--,=0
iJx 4x !(2x- y)(2x + y) = 0
at y z' , ,
y =2XZ
iJy 2x y
2
3
Of 2z 2 z = Y
-=0 ----;=0
iJz y z
Din prima ecuatie rezulta 2x = y (x > 0, y > o).
155
y
3
= yz' => y' = z' => z = y (y > O,z > o)
z' = y => y' = y => y(y -1)(y + 1) = 0 => y = 1
1
Atunci z=1 x =
2
.
Singurul punct stationar este Sa calculam derivatele
partiale de ordinul II.
a'f =L
ax' 2x'
a'f y
--=--
axay 2x'
a'f
--=0
axaz
a'f 1 2z'
-=-+-
ay' 2x y'
a'f 2z
ayaz =--yz
a'f 2 4
--=-+-
ilz' y z'
Atunci diferentiala de ordinul II calculata in punctul (-i ,1,1) este
urmatoarea functionala patratica:
2 ( 1 ) il'f ( 1 ) 2 il'f ( 1 ) 2 il'f ( 1 ) 2
d f 2 ,1,1 = ax' 2 ,1,1 dx + ay' 2 ,1,1 dy + ilz' 2 ,1,1 dz +
a'f ( 1 ) a'f ( 1 ) a'f ( 1 )
+2-- -11 dxdy+2-- -11 dxdz+2-- -11 dydz=
axay 2' ' axaz 2 ' ayaz 2' '
= 4dx' + 3dy
2
+ 6dz' - 4dxdy- 2dydz.
156
4
t., = 4,t., = -2
-2
3
este punct de minim p1
4. fntr-un plan 1
triunghi drept baza s1
inaltime h. Si
mica arie laterala S.
Varful piramidei se all
triunghiul, Ia distanta I
proiectiei M a vartului 1
Pozi!ia lui este perf,
perpendicularelor dus'
din aceste marimi est'
se afla de pal
vom pune semnul mir
triunghiului, este inde
28
4
- ax- by A.
z = . na
c
1
l?i X=-
2"
derivatele
( ~ , 1 , 1 ) este
4
4 -2
fl.\ = 4, fl., =
-2
=8fl.,=-2
3 '
0
este punct de minim pentru f.
-2
3
-I
0
-1 =52=>
6
punctul ( ~ .u)
4. lntr-un plan se da un triunghi cu laturile a, b, c. Pe acest
triunghi drept baza se pot construi o infinitate de piramide cu
aceeal?i lnaltime h. Sa se determine piramida care are cea mai
mica arie laterala S.
olutieJ
v
B
A
Varful piramidei se afla In planul paralel cu planul in care se afla
triunghiul, Ia distanta h de acesta. Problema se reduce Ia aflarea
proiectiei M a varfului piramidei pe planul in care se afla triunghiul.
Pozitia lui este perfect determinata prin marimile x, y, z a
perpendicularelor duse din punctul M pe laturile a, b, c. Fiecare
din aceste marimi este "pozitiva" (are semnul plus) daca punctul
se afla de aceeal?i parte a laturii ca al treilea varf al triunghiului ~ ? i
vom pune semnul minus in caz contrar. Daca notam prin 84 aria
triunghiului, este lndeplinita relatia: ax+ by+ cz = 28
4
, de unde
28
4
-ax- by A .
1 1
_ . _
z = -- . na atera a se expnma prin:
c
157
a b c
1
a "--- b 1
S=-.Jx'+h' +--vz'+h' =--vx'+h
2
+---yy
2
+h
2
+
2 2 2 2 2
c J{28-,- ax- by)'+ c'h' ( )
+ =Sx,y
2 c
Numarul variabilelor s-a redus Ia doua: x y. Avem:
as 1 ax 1 cz a . as 1 by 1 cz b
ax = 2 .Jx' + h' - 2 Z
2
+ h
2
C 8y = 2

+ h' - 2 Z
2
+ h
2
C .
as=
0
ax x y z
Din sistemul: obtinem: = = -.===
as= 0 .Jx' +h' +h' .Jz' +h
2
'
ay
de unde x = y = z. Punctul corespunzator este centrul cercului
inscris in triunghiul dat.
5. Sa . se determine extremele functiei
f(x, y) = x' + y'- y- x, conditionata de ecuatia x + y = 1.
Avem de rezolvat o problema de extrem cu o singura
legatura: x + y -1 = o. Formam functia ajutatoare:
<t>(x, y; A.)= x' + y'- y- x + A.(x + y -1). Rezolvam sistemul:
a<I>
-=0
ax l2x -1 + A. = 0
a<I>
ay = o 2y -1 +A. = o, sistem ce ad mite solutia unica:
a<I> x+y-1=0
-=0
Dz
158
1 o
X : y : 2' A = 0. I
( pentru A = 0
a' <I> (1
d'<I>(x,y)= ax' 2':
= 2dx' +2dy'
Diferentiind legatl
dx = -dy.
(
_!_ .!. l este punct <
2' i)

Faptul ca A
gasirea punctului <
de fapt o problemi
6. Sa se g
ca X + Y + Z

Este o pro
{
x+y+z-5=0
xy+yz+zx-8
1 ntroducem func1
<t>(x, y,z; A"A,) =
Formam sistemL
+h' +
b
c
z
+ h' '
cercului
functiei
1
x = y = -, A= 0. Diferentiala de ordinul II a functiei <!> In punctul
2
pentru A= 0 este:
d'<I>(x y) = a' <I>(__!_ Loldx' + a' <I> (__!_ _l_.o\dxdy +a' <I>(__!_ Loldy' =
ax' 2 2 ') axay 2 2 ') ay' 2 2 ')
= 2dx
2
+ 2dy'
Diferentiind legatura x + y -1 = 0 obtinem dx + dy = 0, sau
dx = -dy. d'<l>(x, y) = 4dx' > o, de unde rezulta ca punctul
este punct de minim.
pbservatie.\
Faptul ca A= 0 exprima faptul ca legatura nu intervine In
gasirea punctului de extrem deci, In acest caz problema este
de fapt o problema clasica de aflare a extremelor.
6. Sa se gaseasca extremele functiei f( x, y, z) = x y z ,
ca X + y + Z = 5 xy + yz + ZX = 8.
Este o problema de extrem cu doua legaturi:
{
x+y+z-5=0
xy + yz + zx - 8 = o
lntroducem functia ajutatoare:
<!>(x, y,z; A, A,)= xyz + A
1
(x + y + z- 5) + A
2
(xy + yz + zx- s).
Formam sistemul:
!59
8<1>
~ = 0
ox
8<1>
~ - = 0
8y
8<1>
~ = 0 <=>
8z
8<1>
-=0
OAI
8<1>
-=0
OA,
yz+A
1
+A
2
(y+z)=O
XZ+A
1
+A
2
(x+z)=0
xy+A
1
+A
2
(y+x)=O
X+Y+Z=5
xy+ZX+YZ=8
Scazand primele trei ecuatii, doua cate doua, obtinem:
(y- x)(A
2
+Z) = 0
(z- y)(A, +X)= 0
(z-xXA,+y)=o
x+y+z=5
xy+ZX+YZ = 8
in cazul primei ecuatii putem avea doua situatii:
a) y- x = o
b) _y-x;tO
a) y- x = 0 => x = y. Coreland cu z- y = 0, din a doua ecuatie,
precum ~ i cu a patra ecuatie, rezulta: z = y = x =%, care nu
satisface a cincea ecuatie. Deci z- y * 0 ~ i rezulta: x =-A,,
y =-A,. inlocuind in ultimele doua ecuatii, rezulta:
{
z- 2A
2
= 5
2 => z = 2A, + 5
A
2
- 2ZA
2
= 8
A,'- 2A
2
(2A
2
+ s) = 8 <=> 3A,' + 10A
2
+ 8 = 0
160
~ = 100-96 = 4, "'' =
i)
4
A =--=>X=Y
2 3
sa calculam diferen
~ , ..
o'<t> . u "
--= z +A.,, ""'"
ox8y v
1
u.
2(447.
Atunc1 d <t> 3'3'3'
Diferen\iind legiUur
{
dx+dy+dZ=0
4 4 7
-dy+-dx+-dY
3 3 3
{
dX+dy +dZ =I
<=> .!..!. dx + dy +
3 3
Rezulta 2dxdy =
maxim (local).
ii) A,
2
= -2 =>X=
= -dxdy = dx' (di
Rezulta ca punctL
b) y-xot-0, Y*'
Rezolvand, se
precedent, ~ i anu
ecuatie,
, care nu
X= -A.,'
4
-102 -3
6 =(
-2
4 4 8 7
i) A.
2
=-3=>X=Y=3, Z=-3+5=3
16
A., =-yz-t..,(y+z)=-
9
Sa calculam diferentiala de ordinul II in punctul (-, _, 2) .
. 3 3 3

ax' = o; ay' = o; fJz' = o;

axay =z+A.,; ayaz =x+t..,; axaz =Y+"-,.
Atunci _ 2
16
2) = 2dxdy.
3'3'3'9'3
Diferentiind legaturile, ob!inem:
l
dx+dy+dz=O
4 4 7 4 7 4
-dy + -dx+-dy +-dz +-dx+-dz = 0
3 3 3 3 3 3
l
dx+dy+dz=O
11 11 8 dz = 0, dy = -dx.
-dx+-dy+-dz = 0
3 3 3
Rezulta 2dxdy = -2dx
2
< 0 => punctul ( j)
este punct de
maxim (local).
ii) /..
2
= -2 => x = y = 2, z = 1 A., = 4. Atunci

=
= -dxdy = dx' (diferentiind legaturile se ob!ine tot dz=O, dy=-dx).
Rezulta ca punctul (2,2, 1) este punct de minim (local).
b) y- X ,t 0, y ,t X Z = -A.
2

Rezolvand, se obtin permutarile solutiilor gasite in cazul
precedent, anume:
161
(
4 7 4) . (7 4 4) .
punctele
3
,3,-
3

3
.
3
.
3
- puncte de max1m;
punctele (2,1,2) (1,2,2) - puncte de minim.
7. Sa se determine extremele functiei f:(o,oot IR,
t(x, y, z, t) = x + y + z + t, cu conditia xyzt- c = 0, c > 0.
olutieJ
/Metoda I / (schitare)
Vom reduce problema Ia o problema de extrem clasic,
reducand numarul variabilelor.
c
xyzt=c
4
=>t=-.
xyz
Trebuie sa gasfm acum extremele functiei de trei variabile, x, y, z,
4
definite fn primul octant al spatiului, g(x, y, z) = x + y + z +
xyz
Punctele stationare le gasim rezolvand sistemul:
ag_= o
ax
ag
ay
ag
-=0
az
c'
1---=0
x'yz
4
1- = 0 => x
2
yz = xy'z = xyz
2
= c
4
=> x = y = z = c,
xy z
c4
1---=0
xyz'
4
ceea ce implica t = .;- = c.
c
Se poate arata ca punctul (c,c,c,c) este punct de m1mm
(local), iar valoarea minima a functiei f este 4c. Se poate arata
chiar ca acest punct este punct de minim absolut (folosind
inegalitatea mediilor, x + y + z + t ;o: 41../xyzt ).
162
\Metoda II\
o rezolvam c
Sa introducem func
F(x, y,z, t) = x+y+l
oF aF aF
ay=O, az=
0
'at
1 + A.yzt = 0
1+A.xzt=0
1+/..xyt=O
1 + AxyZ = 0
xyzt = C
4
Tnmultind primele P
X= y = Z = t = -'Axy:
a'F a'F a'F i
ax' = ay' = az.' -
a'F a'F
-=
ayaz , ayat.
Atunci, diferentiala
calculata pentru 'A
2
d
2
F(c,c, c,c) = -c:(c
Diferentiind legatur
c'(dx + dy + dz+dt
dt = -(dx+dy +dz
)'
. extrem clasic,
X, y, Z,
c
Y+Z+-.
xyz
X=Y=Z=C,
de minim
poate arata
(folosind
jMetoda 111
0 rezolvam ca o problema de extrem cu legaturi.
Sa introducem functia ajutatoare
F(x,y,z,t)=x+y+z+t+ t..(xyzt-c). Punand conditiile
i!F i!F i!F i!F . .
= 0 = o = 0 = 0 se obt1ne s1stemul:
ay 'az 'at 'at..
1 + A.yzt = 0
l+A.xzt=O
l+Axyl=O
l+AxyZ=O
xyzt = c
4
i!F

Ox '
inmultind primele patru ecuatii cu variabila care lipsel}te, rezulta
x=y=z=t=-Axyzt. Deducem x = y = z = t = c l}i
c
i!'F i!'F 8
2
F 8
2
F i!'F 8
2
F
- - - - - 0 "i --= Azt, ""'"- = A.yt,
Ox
2
- 0y
2
- OZ
2
- at
2
- y 0x0y UJ<.UL
8
2
F 8
2
F o'F
i!y i!z = t..xt; i!y at = A.xz; i!zat = Axy.
Atunci, diferentiala de ordinul II a functiei F in punctul (c,c,c,c),
calculata pentru va fi:
c
2
d
2
F( c, c, c, c) = --( dxdy + dxdz + dxdt + dydz + dydt + dzdt) .
c
Diferentiind legatura xyzt = c
4
, obtinem:
c'(dx + dy + dz + dt) = 0 (::;> dx + dy + dz + dt = 0. inlocuindu-1 pe
dt = -( dx + dy + dz) in relatia de mai sus, se obtine:
163
. d
2
F(c, c,c,c) = 2dxdy- 2dxdz- 2dydz + 2(dx +dy + dzt] =
= l(2dx
2
+ 2dy' + 2dz
2
+ 2dxdy + 2dxdz + 2dydz) =
c
+(dx+dz)' punctul (c,c,c,c) este
punct de minim (local), chiar absolut, conform celor constatate Ia
metoda L
x
2
y
2
z
2
8. Elipsoidul cu trei axe: (a> b >c)
a
2
b' c
2
intersecteaza planul lx +my+ nz = 0 , care trece prin centrul sau
(originea sistemului de axe). Sa se determine semiaxa elipsei
obtinuta ca sectiune, i.e. sa se gaseasca valorile extreme ale
functiei f(x, y, z) = x' + y' + z' daca variabilele verifica ecuatiile de
mai sus.
Metoda numarului de variabile pentru a reduce
problema Ia o problema de extrem clasic duce Ia calcule
complicate; vom aplica metoda lui Lagrange. Fie
(
x
2
y' z'J
F(x, y,z; 1.
10
1.
2
) = X
2
+ y' + z' + 1.
1
82 + 1)2 + CT + 1.
2
(1x +my+ nz).
lmpunem condiliile:
8F 8F 8F 8F 8F
-=0 -=0 -=0 -=0 -=0
ax "'' az ' 81. 81. VI I 2
164
+ 1) + 1.
2
1 =
+ 1) +A 2m
+ 1) + A
2
n
x' y
2
z'

a' b c'
lx+my+nz =0
Tnmultind prima e1
cu z, obtinem: A
1
I. Ia'
2 '
x=-2a'-t'
De aici se obtine 1
sus lnmultite resp
l'a' m'b' n
--+--+-
a'- f b'- f c'
extreme ale func\i
9. Sa se g
n
= z: xk' , ca
k==l
olutieJ
Fie F(xl'x:
derivatele paf\iale
8F A
- = 2xk +-=0
axk ak
l(a>b>c)
+nz).
+ 1) + 1..,1 = o
+1)+A2m=O
+ 1) + A2n = o
x' y' z'
-+-+-=1
a' b c
2
lx+my+nz= 0
fnmultind prima ecuatie cu x, a doua ecuatie cu y i a treia ecuatie
cu z, obtinem: A
1
= -(x' + y
2
+ z'} = -f(x, y, z). Se mai poate scrie:
A Ia' A mb' A nc'
x ' y , z ,
=-2a'-f' =-2b'-f' =-2c'-f.
De aici se obtine uor A
2
i apoi x,y,z. Adunand egalitatile de mai
sus inmultite respectiv prin I, m, n se obtine ecuatia:
l
2
a
2
m'b' n'c'
-,-- + -, - + --,- = o, de unde se determina cele doua valori
a -f b -f c -f
extreme ale functiei f (se ajunge Ia o ecuatie de gradulll in f).
9. Sa se gaseasca extremele functiei t(xl'x,,.,xn} =
n n X --
= 'l:x,', tiind ca 2.:-' = 1, unde a,> 0, Vk = 1,n.
k""\ k=l ak
olutie.l
Fie F(xl'x,, ... Egaland cu 0
ko=l k=t ak
derivatele partiale ale functiei F obtinem:
oF A _ A
- = 2x, +- = 0, k = 1,n; L..- = 1. Rezulta: x, = ---,
ax, a, kl a, 2a,
165
\fk =I, n.
lnlocuind in ultima ecuatie, deducem:
(
"-) n ]
-- L:-,-=l<=l
2 =I a,
2
<=>A= --n--
L:a,2
k=l
Rezulta, x
1
o
2
F
-=2'
8x 2 '
k
I n
n , 1=1,n. SanotamcuA=:L;a,'.
a\. L;a,' =I
k=l
Diferentiala de ordinul II in punctul considerat este:
" 8
2
F
d'F = 2 L --, = 4n > 0 => punctul de coordonate
'=I ax,
este punct de minim (local). Cu ajutorul inegalitatii Cauchy-
Buniakovski - Schwartz se arata imediat ca punctul este de minim
global. Valoarea minima a functiei este m =
2
1
, =
=I A a,
1 n 1
=-, L:-,.
A-,=la,
A.2. Aplicatii in economie
1. Productia saptamanala de branza a unui producator rural este
data prin Q = 30L
0

25
K
0
'
75
, unde Q reprezinta cantitatea de branza
exprimata in pounds, L reprezinta orele de munca necesare iar K
reprezinta capitalul investit exprimat in sute de dolari. Daca
fabrica propus sa produca 300 pounds pe saptamana, gasiti
valorile lui K L ce minimizeaza cestui saptamanal, in conditiile in
care fiecare ora de munca este platita cu 10 $/h.
166

Cestui total e:
este exprimat in sutE
I 3
<=>30-L
4
-K
4

Formam functia ajut
F(K,L;A) = IOL+ 1001
Egalam derivatele 1
Obtinem:
oF = 100 + 3ALK
2
'
8K
oF =10+AK
3
=0
8L
oF =LK' -10
4
=0
8A
!OK
L=-3-
<=> K
4
= 3-10
3
10
A=--,
K
Din a doua ecuatie
Cestui minim este ;
2. Producal
produsele in patru
(Dallas). Pentru u
publicitate suma c
(
A) n 1
-- I-i =I<=?
2 ,,, a,
n
cuA= Ia.'.
k=l
(
_I -!)
a, A' 'a"A
Cauchy-
este de minim
=
olutieJ
Costul total exprimat in dolari este: C(K,L)=10L+100K. (K
este exprimat in sute de dolari). Legiitura este: 0=300 <=?
I 3 1 3
<=?30L" K' =300<=?L4 -K' =10<=?LK
3
=10
4
Formam functia ajutatoare F, data prin:
F(K,L;A) = IOL +lOOK+ A(LK
3
-10
4
).
Egalam derivatele partiale ale acestei functii cu 0.
Obtinem:
aF =100+3ALK'=O
8K
L 10
---
K 3
aF =10+AK
3
=0
8L
::> AK
3
=-10 <=? LK' = 10
4
<:o>
aF =LK
3
-I0
4
=0
8A
L= !OK
3
<=?K
4
=310
3
10
A =-K'
LK
3
= 10
4
AK
3
= -10
7
" 1 o
Din a doua ecuatie obtinem K = 3
4
I 0
4
L = -
3
.
3'
3 3 4
-- -
Costul minim este atunci C = 100 3
4
10
4
+ 100.3
4
-10
3
= 9870$.
2. Producatorii Chevelure produc vand
produsele in patru centre: A (Atlanta), B (Boston), C (Chicago), D
(Dallas). Pentru urmatorul an ei planifica sa cheltuiasca pe
publicitate suma de 507 I 0
3
$. Datele culese indica urmatoarea
167
legatura lntre vanzari !;>i sumele folosite pe reclame, in fiecare din
cele patru centre:
I
qA =500-XA2,
I
q
8
=600-X
8
2,
I
qc = 800 Xc2,
I
q
0
= 720-X
0
2,
unde q reprezinta numarul de vanzari, iar X suma cheltuielilor pe
publicitate. Folosind preturile de vanzare, costurile variabile (pe
unitatea de produs) i costurile anuale fixe date in tabelul urmator,
gasi\i valorile XA, Xs, Xc i Xo ce maximizeaza profitul.
Piata Pretul de vanzare Costul variabil Costul fix
A 1,60$ 0,80$ 40000$
B 2,00$ 1,00$ 60000$
c 1,70$ 1,00$ 80000$
D 1,50$ 0,50$ 30000$
Suma ob\inuta din vanzari este:
R = 1,6qA + 2q
8
+I, 7qc + 1,5q
0
, in timp ce costul total se obtine
adunand costul fix cu costul variabil !;>i cheltuielile publicitare, fiind
dat de:
C = 0,8qA +q
8
+ qc + 0.5q
0
+ XA + X
8
+ Xc + X
0
+ 210000.
Beneficiul rezultat are forma:
B = R- C = 0,8qA + q
8
+ 0,7qc + q
0
- XA - X
8
- Xc- X
0
--210000 =
I I 1 1
= 400XA 2 + 600X
8
2 + 560Xc 2 + 720X
0
2 - XA - X
8
- Xc - X
0
-
-210000.
Trebuie sa \inem cont de restrictia:
168
XA +X a + Xc +X.
Formam functia a
F(XA,X
8
,Xc,Xo;'
- X
8
-Xc -X
0
-
Egaland cu 0 deri

ax A
8F X _1_
--=300 B 2
8X
8
aF =
axe .
8F ..!
--=360X
0
2
8X
0
8F
-=XA +X
8
+
8A
Rezulta 1- i\
exprimand in fun
X
8
= 2,25XA
Xc = 1,96XA
X
0
= 3,24XA
Tnlocuind in ultin
XA=60000, de u1
3. Optim
costa
Consideram un
productie, x i Y
de a determina
agentul econon
notata cu T i c1
exprimate taate
utilizate din cei
in fiecare din
;;uen1ule11110r pe
(pe
ue1u1 urmi:itor,
obtine
fiind
XA + X
8
+ Xc + X
0
= 507000.
Formam functia ajutatoare:
l 1 I 1
F(XA, X
8
, Xc, X
0
; A)= 400XA 2 + 600X
8
2 + 560Xc2 + 720X
0
2- XA-
- X
8
- Xc- X
0
- 210000 +A( XA + X
8
+ Xc + X
0
- 507000)
Egaland cu 0 derivatele partiale de ordinul I se obtine sistemul:
oF
1
--=200XA-, -1+A=0
oX A
oF
1
--= 300X
8
-,-1 +A= 0
oX
8
oF I
--= 280 Xc -, -1 + A = 0
oXc
oF X _
1
--= 360. D 2 -I +A = 0
oX
0
oF
oA = XA + X
8
+ Xc + X
0
- 507000 = 0
I I 1 1
Rezulta 1- A= 200XA -:; = 300X
8
-:; = 280Xc -, = 360X
0
-'
exprimand in functie de XA:
XB = 2,25XA
Xc = 1,96XA
X
0
= 3,24XA
inlocuind in ultima ecuatie a sistemului, se obtine:
XA=60000, de unde X
8
=135000$,Xc=117600$, Xo=194400$.
3. Optimu/ producatorului: maximizarea produc(iei Ia un
cost dat
Consideram un producator ce are Ia dispozitie doi factori de
productie, x y, pentru a produce anumite bunuri. Problema este
de a determina volumul maxim al productiei in situatia in care
agentul economic dispune de o cantitate totala fixa a resurselor,
notata cuT preturile Px py ale factorilor de productie,
exprimate toate in franci francezi. Daca notam cu x, y cantitatile
utilizate din cei doi factori cu Q(x,y) volumul productiei rezultate
169
din aceasta alocare, atunci se cere maximul functiei de productie
Q(x,y), tinand cont de restrictia T=x Px+Y Py

[Metoda 1.\
Curba x Px+Y py=T este, in planul xOy, o dreapta. Dar
x <o o, y <o 0 , deci in plan ne va interesa doar un segment al
acestei drepte.
y
B
A X
Op\iunea producatorului este undeva in triunghiul OAB, el
nefiind obligat sa cheltuiasca tot bugetul. Combinatiile de factori
de productie ce realizeaza acela!?i volum al productiei se numesc
curbe de izoprodus sau izocuante. Pe desen sunt figurate trei
170
asemenea izocu
Q2, Q3.
Se obser
este tangenta
euristica, intuitivi
\Metoda
T=x Px+Y
x). Trebuie sa
Anulam derivat<
\Metod;
Conside
singura legatur
F(x, y) = a(x,y)
Sa calculam d'
oF aa
ox.= ox-A!
iJF = iJQ -AI
ay ay
XPx + YP, = 1
Pornind de Ia
de productie
o dreapta. Dar
un segment al
Px T
=--x+-
Py Py
X
JTIUnQhiiUI OAB, el
lbim1tiiiE de factori
se numesc
sun! figurate !rei
asemenea izocuante, corespunzatoare nivelelor de productie 01,
Oz, Q3.
Se observa ca maximul este atins pentru izocuanta care
este tangenta Ia dreapta bugetului. Aceasta este o solutie
euristica, intuitiva.
T=x Px+Y py=> y = ..!._- x& = f{x) (o functie f depinzand de
Py Py
x). Trebuie sa determinam extremele functiei g(x) = O(x, f{x)).
Anulam derivata lui g: g(x) = 0 etc.
IMetoda IIU
, ____ _
Consideram problema ca o problema de extrem cu o
singura legatura. Fie functia ajutatoare:
F(x, y) = Q(x, y) + A(T- xpx- YPy ).
Sa calculam derivatele partiale ale functiei F ~ i sa le egalam cu 0.
oF oQ
-=--Ap =0
0X OX X
oF oQ
-=--Ap =0
8y oy Y
XPx + YPy = T
Pornind de Ia primele doua ecuatii, deducem:
171
8Q 8Q 8Q
8x = & c:i A = ox = 8y
8Q y .
-- Py Px Py
8y
Deducem ca in starea de echilibru raportul productivitatilor
marginale este egal cu raportul preturilor, sau exista egalitate intre
productivitatile marginale impartite Ia preturi.
Diferentiind functiile Q obtinem:
8Q 8Q .
dQ = 8x dx + 8y dy dT = Pxdx + Pydy.
Dar in starea de echilibru 8Q = Apx oQ = ApY; deducem astfel
8x By
dQ = A(Pxdx + pydy) = AdT.
dT = 1 => dQ = A.
A mascara astfel productia suplimentara obtinuta atunci cand
marim bugetul.cu o unitate monetara.
Vom rezolva in continuare explicit problema de extrem,
alegand o forma particulara pentru functia de productie. Vom
utiliza functia de productie de tip Cobb - Douglas. Astfel, daca
factorii de productie sunt cantitatile de munca L (exprimate in ore
munca) de capital K (exprimate, de exemplu, in mii de franci),
functia se scrie:
Q(K, L) =A K" L!, unde A, a, 13 sunt parametrii pozitivi.
lpoteza suplimentara a+ 13 = 1 este frecventa. Tn acest caz,
functia Cobb-Douglas devine:
Q(K,L)=AK"L'-".
Revenim Ia sistemul obtinut prin egalarea derivatelor de
ordinul I cu 0.
= aA(-'=-)1-a 8Q = aA(K)'a
8K K '8L L
Sistemul devine :
172
= Ap,
=APK
KpK +Lp, = T
Vom continua calculel
Tinand cont de primel
K =Eb_=>K=E.t,_L
L PK PK
Atunci, din a treia ec1
cantitatilor folosite di
raportul preturilor res
1 A
A=2TKL
Derivatele partiale dE
a'F A FL. a'
8K
2
= -4 Kv'K; 81
Diferentiala de ordin1
' A jL dK'
d F = -4 Kv'K '
atunci cfmd
(
K)l-a
aA -L = ApL
'" = ApK
KpK +LpL = T
Vom continua calculele pentru a =
Tin and cont de primele doua relatii
K p'-L
L PK PK
A
. d" . . It- L T d . K T
tunc1, 1n a tre1a ecuat1e rezu a =-, ec1 = Raportul
' 2pl 2pK
cantitatilor folosite din fiecare resursa este invers proportional cu
raportul preturilor resurselor.
1 A
A=2TKC
Derivatele paftiale de ordinul II ale functiei F sunt:
8
2
F A Jl 8
2
F A -JK 8
2
F A 1
aK' 8L
2
LJl; 8K8L =4-JLK.
Diferentiala de ordinulll este:

-JK dL'.
4 K-JK 4 .JlK 4 LJl
173
Diferen\iind legatura KpK + LpL = T, rezulta pKdK + PL dl = 0, sau
dK = _!'t,_dl. Obtinem:
PK '
2 A p,
2
2 A 1 PL 2 A .JK 2
d F = ---"-dl - ----dl - ---dl < 0 => punctul
4 pK
2
2 -JLK PK 4 L.Jl
considerat este punct de maxim.
Qmax = A-.JK . .JL =A IT )
2
T =A kP:
v2P: PL 2 PKPL
Observatie.
Problema se poate generaliza in cazul in care sun! utiliza\i
n factori de productie, rezultatele principale ob\inute ramanand
adevarate ~ i in cazul general.
B. Probleme propuse
1. Sa se determine extremele urmatoarelor func\ii:
a) f(x, y) = x' + y
3
- 3xy
R. fm;n = -1, pentru x=1, y=1
b) f(x, y) =ax'+ 2bxy + cy'- ex- fy
( )
(
ce-bf af-bc )
R. M x,,y, =M ( ')' ( ') , daca ac-b
2
,cO, ~ i
2 ac-b 2 ac-b
avem:
i) ac- b' > O, a > 0 => M este punct de minim
ii) ac- b
2
> 0, a < 0 => M este punct de maxim
iii) ac- b' < 0 => nu exista puncte de extrem
174
c) f(x, y) = x' + y' -4X)
R. (1,1) ~ i (-1,-1) pur
d) f(x, y) = xy'ex-y
R. (-1 ,2) punct de m
e) f(x,y)=sinx+siny
R. (
3
;,
3
;) punct
maxim
f) f(x, y) = (x' + y')e-(x
R. fm;n=O, pentru x=
X
g) f(x, y) = e
2
(X+ y')
2
R. fm;n=--, pentru
e
h) t( x, y, z) = 2x
2
+ y' -+
R. (2, 1 , 7) este pun1
i) f(x, y,z) = 31nx + 211
R. (6,4, 1 0) este pUI
j) f(x,z, y) = x
2
+ y' +
4
R. fm;n=-
3
, pentn
x' + y' +
k) f(x,y,z)= xyz
R. (a,a,a); a>O
2. Sa se gaseasca p1
a) f(x, y) = (x-1)
2
+Y
R.M(0,1)
X y
b) f(x,y)=--+-, da
a b
dl = 0, sau
utilizati
raman and
b
2
;t 0, i
c) f(x,y)=x'+l-4xy
R. (1,1) i (-1,-1) puncte de minim
d) f(x, y) = xyV-Y
R. (-1 ,2) punct de minim
e) f(x, y) =sin X+ siny +cos( X+ y); (x, y) E[ 0,
3
;] x [ O,
3
;]
R
(
37t 37t) d . . (7t 7t)
.
2
.
2
punct e mtntm,
6
,
6
maxim
f) t(x, y) = (x' + y')e-(x'+y')- (x' + y')
R. fm;n=O, pentru x=O, y=O
X
g) f(x,y)=e2(x+y')
2
R. fm;n=--, pentru x=-2, y=O
e
h) f(x,y,z)=2x
2
+Y
2
+2z--xy-xz
R. (2, 1, 7) este punct a
. (57t 57t)
I
6
,
6
puncte de
i) f(x,y,z)=31nx+21ny+51nz+ln(22-x-y-z)
R. (6,4, 1 0) este punct de maxim
j) f(x,z, y) = x
2
+ y' + z'- xy + x- 2z
4 2 I .
R.fm;n=-3, pentrux=-3, y=--;;; Z=l
x
3
+ y' + z'
k) f(x,y,z)= , X>O, y>O, Z>O
xyz
R. (a, a, a); a> 0 sunt puncte de minim
2. Sa se gaseasca punctele de extrem conditional:
a) f(x, y) = (x -1)' + y' cu legatura x'- y' = 1
R. M(O, 1)
b) f( )
X Y d ~ 2 ,
X, y = a+ b, aca X + ~ = 1
175
c) f(x, y) =Ax'+ 2Bxy + Cy', dacii x
2
+ y' = 1
R. fm;n = "-, fmax =A.,, unde A
1
< "-
2
sun! radacinile ecuatiei:
(A-A.)(C-A.)-8
2
=0
d) t(x,y,z)=xy'z', daca x+2y+3z=a; x>O, y>O, z>O.
R. x = y = z = f =(a)
6
6 max
6
e) t(x, y,z) = xyz, daca x
2
+ y
2
+ z' = 1, x + y + z = o.
1 (1 1 2) (1 2 1)
R. fm;n = -
3
..[6 , pentru .J6 , .J6 , - .J6 , .J6 , - .J6 , .J6
(
2 1 1 )
- .J6 ' .J6 , .J6
f) f(x, y, z, t) = xyzt, daca x + y + z + t = 4c (x>O, y>O, z>O, t>O)
R. fmax =C
4
, in (c,c,c,c)
()
11 _1 1 1
g) f x,y =-+-, daca -
2
+-
2
=-, (a>O)
X Y X y a
R. {- afi,- avf2) punct de minim
{ afi,avf2) punct de maxim
h) f(x,y,z)=x'+y
2
+Z
2
, dacaax+by+cz=1
R. (a
2
+:, + c' 'a' + :, + c
2
'a' + :
2
+ c'-)
daca a, b, c nu sun! simultan nule
i) f(x, y, z) = x' + y'- z', cu legaturile:
x+y+z=1, x
2
+y
2
+Z
2
=4.
n n
punct de minim,
j) t( x,. x,, .. x") = L x," (p > 1 ), daca L x. = a (a > 0)
k--'1 k=l
R. .. , punct de minim
n n n
176
3. Sa se a rate
4. Dintr-un fir dE
unui acvariu pa
paralelipipedului a
R.
5. Sa se deten
x' y'
-+-=1 cupl<
a' b
2
A'-
R. S=TI-
Problema
F(x, y,z) =
\
x' +i_=
a' b
2

AriaS v
ale func\ie
6. Vezi problem
Pentru Q=Oo
determina\i mini
tip Cobb - Doug!
7. Rezolva\i pro
Cobb - Douglas
ecuatiei:
1 ) .
'.J6
z>O, t>O)
de minim,
3. Sa se arate ca daca X
1
X
2
x, = 1, atunci X
1
+ x, ++X, 2: n.
4. Dintr-un fir de cornier de lungime 4a se rama
unui acvariu paralelipipedic. Sa se determine dimensiunile
paralelipipedului astfel ca acesta sa aiba velum maxim.
R.
5. Sa se determine aria elipsei de intersectie a cilindrului
x' y'
2 + -
2
= 1 cu pia nul Ax+ By+ Cz = 0
a b
A
2
+B
2
+C
2
R. S = IT . C ab
Problema se reduce Ia a determina extremele functiei
F(x, y, z} = x
2
+ y
2
+ z' cu legaturile:


a' b
2
Ax+By+Cz= 0
Aria S va fi S = unde a sunt valorile extreme
ale functiei F.
6. Vezi problema rezolvata cu aplicatie in economie 3.
Pentru Q=Q
0
constant (nivelul productiei este constant),
determinati minimul bugetului T. Puteti lua functia de productie de
tip Cobb - Douglas.
7. Rezolvati problema cu aplicatie in economie 3, in cazul functiei
Cobb- Douglas generale (a=

E[0,1]).
177
4.4. Functii implicit definite. Transformari
regulate. Dependenta functionala.
A. Probleme rezolvate
1. Sa se calculeze derivatele partiale de ordinul I l?i
derivata partiala mixta de ordinul II pentru functia z(x,y) definita
implicit de ecuatia: (x + y)ez - xy- z = 0, In jurul punctului {x
0
, Yo)=
(2,2) pentru z
0
=0.

Sa definim F:IR'--+IR, F(x,y,z)=(x+y)ez-xy-z l?i fie
{x
0
, y
0
,z
0
) = (2,2,0). F are derivate partiale de ordinul I continue
(chiar de orice ordin), date prin formulele:
8F z 8F z 8F ( ) z
-=e -y -=e-x -= x+ye -1
ax ay az
In mod evident F{x
0
,y
0
,z
0
)=4e
0
-4-0=0 oF(2,2,0)=3;tO.
8z
Suntem atunci In conditiile de aplicabilitate a teoremei functiilor
implicite deducem existenta unei unice functii q> definite pe o
vecinatate Vo a. punctului (2,2), cu valori lntr-o vecinatate U
0
a
punctului 0 astfellncat sa fie lndeplinite conditiile:
1) q>(2,2) = 0
2) F(x,y,q>(x,y))=O, (v)(x,y) EV
0
3) q> are derivate partiale de ordinul I continue, de orice ordin (ca
functia F).
Vom folosi notatia q>(x, y) = z(x, y). Derivand partial relatia 2),
obtinem:
178
iJF(
az axx,
ax (x, y) =-
az

az( ) ay
8y x,y =- iJF(x,
az
Daca derivam par
8
2
z (x, y) =
axay
(ez
In punctul (x
0
, Yo)
az e
0
- 2
-(2,2)=-4 0 1
8y e-
2. Se da
extremele func!i'
precizand pun
F admite'
8F
2
3 -=3X - ay
ax
aF = 3y
2
- 3ax.
8y
ordinul I l?i
z{x,y) definita
{xo, Yo)=
xy-z l?i fie
I continue
iJz ez(x,y)_y ez-y _
8x(x,y)=- oF( ( ))=(X+y)ez(xy)_l =(x+y)ez-1!?
1
oz x,y,z x,y
oF ( )
OZ oy X, y,z(x, y) ez- X
ay(x,y)=- oF (x+y)ez-1
oz (x, y,z(x, y))
Daca derivam partial in raport cu x a doua relatie, obtinem:
8
2
z
axay (x, y) =
(
z oz )f( ) z J ( z z iJz z az)( z )
e ax-! x+y e -I- e +xe ax+ye ax e -x
=
[(x + y)ez -1r
{ ) ( ) 8z ( ) e
0
- 2 I
In punctul X
0
, Yo, z X
0
Yo = Z
0
= 0 2,2 = -
0
-- = -- l?i
a-x 4e-l 3
oz e
0
- 2 I

ay ' 4e
0
-1 3
2. Se da ecuatia: F(x, y) = x' + y
3
- 3axy = 0; a E IR, a> 0. Se cer
extremele functiei implicite y de x, definita de aceasta ecuatie,
cinat::lt" Uo a precizand i punctul {x
0
, Yo) respectiv.
relatia 2),

F admite derivate pal\iale continue de orice ordin pe IR
2
.
oF
2
-=3x -3ay i
Ox
oF
2
- = 3y -3ax.
Oy
179
Fie ( x
0
, y
0
) e I R
2
astfellndit F( X
0
, y
0
) = 0 l?i : ( X
0
, y
0
) * 0 => (3) o
unica functie <p: V
0
~ U
0
ce satisface conditiile:
<r(x
0
) = y
0
; F(x
0
,<p(x
0
)) = 0, (v)x
0
eV
0
<p are derivate partiale
continue de orice ordin (Vo ,U
0
sunt vecinatati ale lui x0, respectiv
y
0
). Notam functia <p tot cu y. Punctul X
0
e punct de extrem pentru
y=> y(x
0
) = 0.
8F
Dar y'(x
0
) =
ax (xo, Yo) 8F
8F =0=>-(Xo,Yo)=O.
--(x Y ) ax
8y o o
8F ( ) 2
Dar ax x
0
_Yo = 3x
0
- 3ay
0
l?i. tinand cont l?i de conditia
F(x
0
,Y
0
) = X
0
3
+Y
0
3
-3aX
0
Y
0
= 0, deducem:
2 6
Xo . ' Xo 3 d d '( 3 ')
Y
0
= - l?l X
0
+ -
2
- 3X
0
= 0, e un e X
0
X
0
- 2a = 0.
a a
Atunci: a) X
0
= 0, Yo= 0
b) X
0
= alfi. Yo = al./4.
8F
Dar, in punctul (x
0
, Yo)= (o.o) avem ay (o,o) = 0 (nu este
satisfacuta conditia : (x
0
, Yo)"' 0, deci nu putem aplica teorema
functiilor implicite pentru (x
0
, Yo)= (o,O)), deci ramane sa studiem
punctul (x
0
, Yo)= (alfi,al./4). Pentru aceasta vom calcula y"(x
0
).
&F 8F &F 8F
&z (x, y) a:y(x, y)- &aY'(x, y) i3x (x, y)
y"(x) = [aF J' =
8y(x,y)
180
=
6x (3y
2
- 3ax)-
(3y'-
4l
Atunci y"(xo) = --
=> punctul X
0
= ~
implicit definita de'
3. Se considera
<p:IR ~ IR este
a,b,c eiR. Fie ( ~
aplicabilitate a
F(x. y,z) = x' + y'
solutia gasita, sa s
az
(cy-bz) ax +(az
olutieJ
Fie z(x,y)
vecinatatea Vo a
punctului z
0
. zest'
I continue (deoa1
Derivatele partiale
aF = 2x- a<p'(ax +
ax
aF = 2y- b<r'(ax +
ay
iJF (
-=2Z-e<p'aX+
8z
Atunci:
(3) 0
paf'\iale
pentru
a
6x(3y' -3ax)-(-3a)(3x
2
-3ay) 2xy' -ax' -a'y

(3y'- 3ax)' V- ax)'
. " 4a'V2- a'(2l./4- 4V2)
Atunc1 y (x,) = ( )' = ( )' < 0:::;,
2a'V2- a'lfi a'lfi
:::;, punctul X
0
= alfi este punct de maxim pentru unica functie
implicit definita de ecuatia F(x, y) = 0 In jurul punctului x
0
= ai.fi.
3. Se considera ecuatia x' + y' + z
2
= cp(ax +by+ cz), unde
cp: IR IR este o functie derivabila cu derivata continua,
a,b,cEIR. Fie (x
0
,y
0
,z
0
) un punct ce satisface conditiile de
aplicabilitate a teoremei functiilor implicite pentru functia
F(x, y,z) = x' + y' + z'- cp(ax +by+ cz). Daca z = z(x, y) este
solutia gasita, sa se verifice relatia:
)
az az
(cy-bz -+(az-cx)-=bx-ay.
ax ay

Fie z{x,y) unica solutie a ecuatiei date, definita pe
vecinatatea Vo a punctului (xo.Yo) cu valori In vecinatatea Uo a
punctului zo. z este o functie ce admite derivate partiale de ordinul
I continue (deoarece q> este derivabila cu derivata continua).
Derivatele partiale ale lui F sunt:

= 2x- acp'(ax +by+ cz);
aF
ay =2y-bcp'(ax+by+cz);
aF
- = 2z- ccp'(ax +by+ cz).
az
Atunci:
181
aF
uz ax 2x-aq>'(ax+by+cz) .
------ l?l
i3x- - 2z- cq>'(ax +by+ cz)
az
8y
az
8F
8y
oF=
uz
2y- bq>'(ax +by+ cz) d .
( )
, ec1:
2Z-Cq>' ax+by+CZ
)
az az
(cy-bz Ox +(az-cx) 8y =
(bz- cy)(2x- aq>'(u)) +(ex- az)(2y- bq>'(u))
= =
2z- cq>'(u)
2bxz- abzq>'(u)- 2cxy + acyq>'(u)
- +
- 2z- cq>'(u)
2cxy- bcxq>'(u)- 2azy + abzq>'(u)
+ -
2z- C<Jl'(u) -
2z(bx- ay) + cq>'(u)(ay- bx) (bx- ay)(2z- cq>'(u))
= 2z- C<Jl'(u) = 2z- cq>'(u) = bx- ay,

unde am notal u =ax+ by + cz.
4. Sa se calculeze derivatele partiale de ordinul I ale functiilor
u{x,y) l?i v(x,y) definite prin sistemul:
{
F,(x, y, u, v) = x + 2y- u + v- 2
( )
, {x
0
,Y
0
,U
0
,V
0
)=(1,1,1,0)
F, x, y, u, v = x' + y' + u' - 3xyu- 3v

Functiile F, l?i F2 admit derivate partiale continue de orice
ordin pe intreg domeniul de definitie.
F,{x
0
, Y
0
,U
0
, V
0
) = 1 + 2-1 + 0-2 = 0
182
F,(x
0
, y
0
,U
0
, V
0
) = 1+H
Derivatele partiale de
aF, =
1
. aF, =
2
.
ox ' 8y '
oF
-
2
= 3x
2
- 3yu;
Ox
oF
-
2
= 3u
2
- 3xy;
au
lacobianul
= 3 - 3+ 3 = 3 * 0.
8F
-
fPJ
8F
a..
Aplicand teorema fL
deducem ca exista
vecinatate V
0
a pun1
punctului (uo, vo), astf1
a) <Jl(X
0
, Yo)= (u
0
, Vo)
b) F,(x,y,q>(x,y))=O
c) Componentele q>
continue de orice on
l?i q>,(x, y) = v(x, y).
partiale de ordinul
formulele:
D(F,, F,)
au D(x, v)
ax (x, y) =- o(F F)
, 2
o(u:v)
-ay,
orice
Derivatele partiale de ordinul I
aF, oF, aF,
-=t -=2 -=-1
ax' iJy' au'
ale functiilor F1 :?i F2 au forma:
aF, =
1
.
av ,
aF
-' = 3x
2
- 3yu
ax ,
= 3 - 3 + 3 = 3 "' 0.
aF
-
2
= 3y
2
- 3xu:
iJy '
aF, = _
3
av .
au
aF,
av _ -1 1 =
oF, - 3u
0
2
- 3x
0
y
0
-3
av
Aplicand teorema functiilor implicite pentru sisteme de functii,
deducem ca exista o unica functie vectoriala q> definita pe o
vecinatate V0 a punctului (xo,Yo) cu valori intr-o vecinatate U
0
a
punctului (uo,vo), astfel incat:
a) rp(x
0
,y
0
)=(u
0
,V
0
}
b) F,(x, y,rp(x, y)) = 0 :?i F,(x, y,rp(x, y)) = 0, (v)(x, y) E V
0
c) Componentele q>
1
:?i q>
2
ale functiei q> au derivate partiale
continue de orice ordin pe Vo. Vom folosi notatia rp,(x,y)=u(x,y)
:?i <p
2
(x, y) = v(x, y). Atunci rp(x, y) = (u(x, y), v(x, y)) :?i derivatele
partiale de ordinul I ale func\iilor u :?i v se calculeaza dupa
formulele:
au )
ax(x,y =
D(F" F,)
-D{X:v)
D(F,F,) =
Dtl.l, vf
OF
1
OF,
ax av
l
aF, a ~
ax av
( 3 - 3u
2
+ 3xy) =
183
I I
3x
2
- 3yu -3
(3-3u' +3xy) =
3+3X
2
-3yu 1+X
2
-yu
= 3 - 3u
2
+ 3xy = 1 - u' + xy
Dl aF!
ay av
aF, aF,_ 2 1
6 +3y
2
- 3XU
- 3- 3u
2
+ 3xy

D(FJ,)
o(U,X)
D(FJ,) =
D{u,v)
ay av 3y
2
-3xu -3
!--a--'F'--
1
-a-F_--:-1 = - ( 3 - 3u
2
+ 3xy) =
au av
aF, aF,
au av


oF
1
au ax
aF._ aF
1
au ax
( 3 - 3u
2
+ 3xy) =
-1 1 I
3u
2
- 3xy 3x' - 3yul
( 3 - 3u
2
+ 3xy)
3x
2
- 3yu + 3u' - 3xy x' + u' - uy - xy
=
- 3 - 3u
2
+ 3xy 1 - u' + xy


oF
1
au ay
=
oF._ oF
2
-1 2 J
av D(u, y) au BY _ 3u
2
- 3xy 3y
2
- 3x =
Oy =- D(F
1
,F,) =- (3-3u
2
+ 3xy)-- (3-3U
2
+ 3xy)
-D(u,vT
y
2
+UX+2U
2
-2Xy
=
1-u' +xy
!84
5. Sa se arate c
{
U=X
2
+Y
2
v = x
2
- y'
este regulata
se gaseasca e)
unde

Transforr
u(x, y) = x' +Y
2
,
au o<
D(u, v) ax i1
o(x, y) = av a
ax a
deci transform!
oricare ar fi pu
punctului (x
0
, y,
astfellncat F /V
proprietate loc
necesar un cal<
Fie (x]'y,),(x,.l
{
u(x]' y,) = u
v(x]'y
1
)=\
Din aceste
x, = x,, y, = y,
Sa determinarr
(u, v) E F(A) :)
5. Sa se arate ca transformarea F:
{
u = x' + y
2
v = x'- y
2
este regulata ~ i injectiva pe multimea A= {(x, y) I x > o, y > o}. Sa
se gaseasca expresia inversei acestei transformari, i.e. F-' =?,
unde F : A ~ F(A) este bijectiva.
olutieJ
Transformarea data F are componentele u ~ i v,
u(x, y) = x' + y
2
, v(x, y) = X
2
- y
2
Jacobianul transformarii este
au au
D(u, v) ax ay 2x 2y
( )="' ,.,= =-Sxy,.,O,(VXx,y)E(O,oo)x(O,oo),
D x,y ~ ~ _uv 2x -2y
ax ay
deci transformarea este regulata pe multimea A. Deducem ca
oricare ar fi punctul (x
0
, Yo) E(O,oo) x (o,oo), (3) o vecinatate V
0
a
punctului (x
0
, Yo) ~ i o vecinatate Uo a punctului (u
0
, v
0
) = F(x
0
, Yo)
astfel incat F/V
0
:V
0
~ U
0
este o aplicatie bijectiva. Aceasta este o
proprietate locala; pentru a obtine o proprietate globala este
necesar un calcul direct.
Fie ( x,, y, ), ( x,, y
2
) E A astfel inca! F(x,, y,) = F(x,, y
2
) ~
{
u(x,. y,) = u(x,, y,) {x,
2
+ y
1
2
= x,' + y,Z
~ ( ) ( ) ~ 2 2 2 ,.
vx,.y,=vx
2
,y, x,-y,=x,-y,
D. t l't"t' d d
2 2 2 2
1n aces e ega 1 a.1 e ucem: x, = x
2
, y, = y
2
,
adica
x, = x
2
, y
1
= y, (toate numerele sunt strict pozitive).
Sa determinam acum multimea imagine F(A).
(u, v) EF(A) (3)(x, y) E A astfel inca! (u, v) = F((x, y))
IRS
(3)x > o, y > 0 astfel Inca! u = x' + y', v = x' - y'. Deducem ca
x' = u; v y
2
= u; v . Conditia necesara suficienta ca
(u, v) EF(A) este:
U+V>O, U-V>O=> F(A)={(u,v)EIR
2
/U+V>0,U-V>0}.
Transformarea inversa F-': F(A) --t A are expresia:
F-'(u, v)= Uu; v )u;v J
6. Fie transformarea F data prin:
{
u = ex+y
3
,x,y EIR.
V=X
Sa se arate ca In orice vecinatate a punctului (0,0) transformarea
nu este regulata, lnsa este injectiva.
olutieJ
Jacobianul transformarii este:
au au
D(u, v) ax ;,., . e eY e eY
vy-
D(x,y) = av av- 3x
2
o
ax ay
fn punctele de forma (O,y) cu y E IR jacobianul este nul, deci In
aceste puncte transformarea nu este regulata.
Fie (x, y,),(x,, yo) EIR' astfellncat F(x, y,) = F(x,, y,)
{
ex,+y, = ex,+y, {x, + y, = x, + y 2
3 3 x, = x,, y, = y,.
x, = x, x, = x,
Fie (u, v) EF(IR'). Atunci exista x, y EIR astfel Inca!
u = e+Y, v = y
3
. Trebuie impusa conditia u > 0 => x + y = In u,
y=Vv. Deci x=lnu-VV, y=Vv.
186
Deducem ca F:IR'
expresia: F-'(u, v) =
7. Se dau transfom
F1:
W=Z+X
Sa se gaseasca trl
relatia:
D( C,, TJ, 1i) D(
D(x, y,z) (x,.y,,z,) = D(
(v)(x
0
,y
0
,z
0
) EIR'-
olutieJ
Transformare
c,(x, y,z) =(X+ y)' +
TJ(X. y, z) = (y + z)' 1
o(x, y,z) = (z + x)' +
8
8
D(u,v,w)
D(x,y,z) ( ) - 8
XoYo,Zo fj
8
ca
ca
deci in
incat
X+y=lnu,
Deducem ca F:IR' este bijectiva inversa lui Fare
expresia: F-'(u, v) = (lnu-VV,VV).
7. Se dau transformarile:
F1: J F2:



, definite pe IR
3
.
l w = z + x 8 = w
2
+ u'
Sa se gaseasca transformarea compusa F
2
oF1 sa se verifice
relatia:
TJ,o) D(u, v, w) TJ,s)
D(x, y,z) (x,.y,.z,) = D(x, y,z) (x,.y,.z,). D(u, v, w) F,(x,.y,.z,)'
(v)(x
0
, y
0
,z
0
) E IR
3
Transformarea F2oF1 are componentele:
+(y+z)' =2y' +X
2
+z' +2xy+2yz
TJ(x,y,z)=(y+z)' +(z+x? =2Z
2
+X
2
+y' +2xz+2yz
o(x, y,z) = (z + x)' + (x + y)' = 2x
2
+ y' + z' + 2xz + 2xy
au au au
-
ax ay az
I I 0 1 0 0
D(u, v, w) av av av
D(x, y, z) (x,.v,.z,)
=
-
= 0 1 1 = 0 1 I = 2
ax ay az
aw aw aw
1 0 I 1 -I I
-
ax ay
az
187
i31; i31; i31;
o(s. 11. 8)
au i3v Ow
2u
0
2v
0
0
Uo
i3T) i3T) i3T)
0 2V
0
2w
0
=8 0
D( U, V, W) (u,,v,,w,)
= =
au i3v Ow
i38 i38 i38
2u
0
0 2w
0
0

au i3v Ow
= 16U
0
V
0
W
0
= 16(x
0
+ y
0
)(y
0
+z,){x
0
+ z,),
unde {u
0
, V
0
, W
0
) = FJx,, y,,z
0
) = {xo + Y
0
, Yo+ Z
0
,Z
0
+ X
0
)
i31; i31; i31;
i3x i3y i3z
o(s. 11. 8) i3T) i3T) i3T)
= =
D(x, y,z) (x,,y,,z,)
i3x i3y i3z
i38 i38 i38

i3x i3y oz
2(xo +Yo) ' 4y
0
+ 2X
0
+ 2Z
0 2{zo+Yo)
= 2(x, + z
0
) 2(y
0
+z
0
)
4Z
0
+ 2X
0
+ 2Yo
4X
0
+ 2y
0
+ 2Z
0 2(Yo +Xo) 2(z
0
+ x,)
Xo +Yo
Zo +Yo
= 8. X
0
+Z
0
Yo +Z
0
2z
0
+X
0
+y
0
=
2Xo + Yo + z, Yo + Xo Zo + Xo
= 32(xo +Yo)(Yo + zo)(xo + zo)
Deducem astfel egalitatea din enunt.
Transformarea F
2
este regulata pe multimea
=
Vo 0
Vo Wo
- v,
Wo
A= {(x, y,z) E IR' I x + y"' 0, x + z "'0, y + z"' o} atunci, conform
relatiei demonstrate, rezulta ca transformarea compusa F
2
oF
1
este regulata pe multimf A (am folosit faptul ca
transformarea F1 este regulata pe IR ).
188
=
8. Transformati ec
coordonate polare:
Vom conside
calcula derivatele p
derivam partial in
obtinem:
l
or .
1 = i3x cos<p-rsmq
or .
0 = -SJn<jl +rCOS<
i3x
or i3<p
Ox= COS<jl, Ox= -j
Derivand partial in r
or .
0 = -COS<p-rSJn<
i3y
or .
1 = -s1nm
i3y 't' '
or . o<p 1
i3y = s1n<p, i3y = ;:c
Atunci putem scrie:
au = (u(r( x, Y ), <p(:
i3x i3x
au= y),<p(:
i3y i3y
Deducem:
8. Transformati ecuatia lui
trecand Ia
coordonate polare: x = rcos<p, y = rsin<p.
Vom considera u ca functie de noile variabile, r <p . Vom
calcula derivatele partiale ale lui r <p in raport cu x y. Daca
derivam partial in raport cu x in relatiile x = r cos <p, y = r sin <p ,
obtinem:
1
1= ar COS<p-rsin<p a<p
ax ax , de unde rezulta imediat
ar . a<p
o = +rcos<p-
ax ax
ar a<p 1.
= COS<p - = --Sin<p.
ax ' ax r
Derivand partial in raport cu y se obtine:
ar . a<p
0 = -rs1n<p -
ay ay
deducem:
ar . a<p
1 = +rcos<p-
ay ay
ar. a<p1
-=-COS<p.
ay ' ay r
Atunci putem scrie:
au au. EJr +au. a<p
ax ax ar ax o<p ax
au a(( ( )) au ar au o<p
ay = ay u r(x,y),<p x,y) =or. ay + o<p. ay
Deducem:
189
= (a'u _ ar + a'u _ 8cp) _ ar +au a'r +( a'u . ar + a'u _ acp) __ acp +
ar' ax aracp ax ax ar ax' aracp ax acp' ax ax
au a'cp
+---=
acp ax'
. 2 a'u 1 . a'u au a'r 1 . a'u
=cos <p ---s1ncp coscp --+ ----s1ncp coscp-- +
ar' r aracp ar ax' r aracp
1 . , a'u au a'cp
+ r's
1
n cp acp' + acp ax'
D
a'r . acp 1 . , .
ar -=-Sincp--=-sln cp
ax' ax r
a'cp 1 . ar ( 1 acp) 2
ax' = r' s1ncp ax+ - ax = ;:-;-sincp coscp.
= = + + + =
= ( a'u . + _ acp) . ar + au . a'r + ( a'u . + a'u . acp) . acp +
ar' ay aracp ay ay ar ay' aracp ay acp' ay ay
au a'cp
+---=
acp ay'
. , a'u 2 . a'u 1 , a'u a'r au
=Sin cp--
2
+-Sincpcoscp--+
2
cos cp-
2
+-
2
--+
ar r aray r acp ay ar
a'cp au
+---
ay' acp.
a'r acp 1
2

Dar ay' =-cos<p ay cp
&cp 1 & 1 . acp 2 .
ay' =- r' coscp ay ay = -;:-;-smcpcoscp
Facand lnlocuirile lnsumand, obtinem:
190
a'u a'u a'u
-+-=--
ax' ay' ar'
a'u 1 a'u 1 i
-+--+--
ar' r' acp' r '
9. Fie functiile:
f(x,y,z)=x+y+
(x. y, z) E IR'- Sa
a) f, g, h nu sun
b) h depinde de

i)f
ax
D(f,g,h) ilg
D(x,y,z)- ax
ilh
ax
(are doua coloa1
Deducem ca fur
din IR
3
.
Sa consideram
functiilor f 9i g.
rangul 2 (: _
independente In
[
Din faptul ca C
punct (x
0
, y,,z,:
functie ' astfel
au a'<r
+---
acp ax' -
o
2
u O<pJ 8<p
&p' ax ax+
a
2
u
8r8<p +
au a'<r
+---
acp ay' -
:J.:+
au
-+
ar
a'u a'u a'u 1 a'u 1 au . . .
eeuatra devrne:
ax' 8y
2
8r
2
r
2
a<p
2
r ar
a'u 1 a'u 1 au

ar' r' a<p' r ar
9. Fie funetiile:
f{x,y,z}=x+y+z, g(x,y,z}=x-y+z, h{x,y,z}=4(xy+yz};
(x, y,z} E IR
3
Sa se arate ea:
a) f, g, h nu sunt independente In nici un punct din IR
3
.
b) h depinde de f g pe IR
3
.

at at at

ax ay az
1 1
D(f,g, h} ag ag ag
1 -1 1 = o, (v)(x, y, z} E IR'
D(x,y,z}
=
ax
ay az
ah ah ah
4y 4(x + z) 4y

ax
ay az
(are doua eoloane egale).
Deducem ea functiile f, g, h nu sunt independente In nici un punet
din IRa
Sa eonsideram matrieea fermata din derivatele partiale ale
funetiilor f g. Aeeasta este G
1
:J are In mod evident
rangul 2 (
1 1
= -2"" o). Rezulta ca fune\iile f g sunt
1 -1
independente In oriee punet din IR
3
.
D(f,g,h) ( )( ) 3
Din faptul ea D( ) o, \f x, y, z E IR rezulta ea pentru oriee
x,y,z
punet (x
0
,y
0
,z
0
) EIR' exista o veeinatate V
0
a acestui punet o
funetie 'I' astfel !neat:
191
h(x, y, z) = 'f'(t(x, y,z), g(x, y, z)), (v)(x, y, z) E V
0
.
Aceasta proprietate este locala. Rezulta lnsa imediat (facfmd o
simpla verificare) ca h(x, y, z) = f'(x, y, z)- g
2
(x, y,z),
(vXx. y,z) E IR
3
10. Sa se arate ca functiile f(x, y) = x
2
+ 4y
2
,g(x, y) = sin(x+ 2y),
h(x, y) = x- 2y, definite pe IR
2
sunt In dependenta functional a pe
multimea (- J x [- ;). Sa se gaseasca legatura dintre ele.

Matricea formata cu derivatele partiale ale functiilor f, g, h
este:
Of Of
Ox 8y
A= ag ag
Ox 8y
8h 8h
Ox 8y
[
2x
=
8y J
2cos(X+ 2y)
-2
D(g,h) cos(x + 2y) 2cos(x + 2y) = _
4
cos(x +
2
y)
D(x,y) = 1 -2
1t +ktt, k EZ.
0 x,y 2
(
1t 1t] ( 1t 1t] ( 1t 3tt)
Dar x E -- - y E -- - x + 2y E -- - .
6'8' 6'8 2'8
Oeci X+ 2y
nu poate fi de forma ; + kn pentru nici o valoare k E Z; rezulta:
D(g,h) ( ) ( n n] ( n n]
D(x,y)'"o, (v) x,y E -68 x -68.
192
Oeducem ca g
data, rang A=2,
globala, daca ob
11. Sa se deten
u(x, y) = f(x+y)
este relatia dintr<
olutieJ
Pentru CE
D(u,v) =O
o(x, y)
8L
D(u,v) w
Oar D(x, y) = 8\
iJ>
av f'(xX1- f(>
-
Ox
av _ t(yX1+f
2
'
8y - [1-
f'(l
D(u, v) = f(xX1
o(x,y) [1-f(
f'(x+y)
= [ 1- f(x)f(y)j'
(facand o
. X+2y
; rezulta:
Deducem ca g h sunt independente In orice punct din multimea
data, rang A=2, f depinde de g h. Dependenta este chiar
globala, daca observam ca f(x, y) g(x, y) + h'(x, y)].
11. Sa se determine functia derivabila f astfel ca functiile:
( )
_ f(x) + f(y)
u(x,y)=f(x+y) v x,y -
1
-f(x)f(y) sa fie dependente. Care
este relatia dintre u v?
olutieJ
Pentru ca u v sa fie dependente (pe IR
2
) este necesar ca
.,.,..,o()--0-u,-+v) =
0
D(x, y)
au au
D(u,v) ax ay f'(x+y) f'(x+y)
Dar ( ) = a av = av av
o x,y v ax -
ax ay ay
av- f'(x)[1- f(x)f(y)] + f'(x)f(y)(t(x) +f(y))- f'(x)(1 + f'(y))
ax- [1- f(x)f(y)j' - [1- f(x)f(y)j'
av f'(y)(1+f'(x)) .
- = , . Rezulta.
ay [1-f(x)f(y)j
f'(x+y) f'(x+y)
D(u, v) = f'(x)(1 + f'(y)) f'(y)(1 + f'(x)) =
D(x, y) [1-f(x)f(y)J' [1-f(x)f(y)j'
f'(X+ y) 1
= [1-f(x)f(y)J'. f'(xX1+f'(y))
1 2
f'(y)(
1
+ f'(x)) = o, (v)x, y EIR .
193
Presupunem ca derivata lui f nu se anuleaza In nici un punct din
IR
2
. Atunci, trebuie ca:

(v) (x,y} EIR


2
. . f'(x)
functra g.IR IR, g(x) =
1
+ f'(x) este constanta.
Atunci, exista o constanta C1, astfel lncat g(x)=C
1
. Integrand,
obtinem: arctgf(x) = Clx + c,, c, E IR. Deci, f(x) = tg{ Clx + c,).
Pentru a gasi relatia dintre u v, scriem:
( )
tg(C
1
X+C,)+tg(C
1
y+C
2
) ( )
v x,y = ( ) ( ) =tg C
1
x+C
1
y+2C
2
=
1-tg C
1
x+C
2
tg C,y+C
2
[( ( ) )
l
tg[C
1
(x+y)+C,)+tgC, u+tgc,
= tg C1 X+ y + C2 + C2 = [ ( ) ] =
1-tg C
1
x+y +C,. -tgC, 1-utgc,
Relatia dintre u v este
U+tgC,
V= -
1- utgC,
B. Probleme propuse
1. Calculati dy pentru y(x) definit implicit de ecuatia
dx dx
(x' + y')'- 2{x' + y') + 1 = 0, In jurul punctului {x
0
, Yo)= (
dy{xo) x
0
R. =--=-1
dx Yo
d'y(xo) y '+X'
_:-'-;,;-'- = 0 3 0 = -2 -fi
dx Yo
194
2
. az. .
. Calculatr -
. Ox y
x' -2y' +3Z
2
-yz-+
solutie a ecuatiei.
az(x
0
, Yo)
R. Ox
az(xo, Yo)
ay
3. Calculati ddy
, X
1 + xy -In( exy + e-xy
dy y
R. dx = -x, dx
2
. az. 82
4. Calculatr 8x ,
x' y' z
2
-, +-, +-, =1
a b- c
az. c'x
R. 8x =- a'z;
a'z c':
axay =- a'b
5. Sa se calculeze
z' - xeY - ye' - ze'
az. 0;
R. Ox =-
1
; ill
nici un punct din
, (v) (x,y) EIR
2
Integrand,
= tg(CIX+ C,).
U+tgC,
1- utgC,
=(fi .fi.J
2 ' 2 .
2. Calculati : ~ i ~ pentru z(x, y) data de ecuatia
x
2
- 2y
2
+ 3z
2
- yz + y = 0 In jurul unui punct (x
0
, y
0
,z
0
) care este
solutie a ecuatiei.
az( Xo, Yo) 2Xo
R. ""
uA Yo -6Zo
az(x
0
,Yo) l-4y
0
-Z
0
=
8y Yo -6Z
0
3. Calculati dy ~ i d'; pentru y(x) definit de:
dx dx
I + xy - in( exy + e x y ) = 0
y
R dy = _'!_ d'y
dx x' dx
2
(1- y)'
R az = _ c'x . az _
ax a'z' ay
a'z
c'xy
- - ~
axay a'b'z' '
c'y
b'z'
daca:
a'z c'(b'- y').
=
ax'
a'z c'(a'-x')
8;2 = - --a'"b'T
5. Sa se calculeze : ~ i : ~ pentru z( x, y) dat de ecuatia:
z' - xeY - yez - zex = 0, In jurul punctului ( X
0
, y
0
) = ( 0,0 ); Z
0
= 0.
az az
R. ax = -1 ; ay = -I
195
6. Daca y {x + z)- (y + z)f{z) = 0, unde f este o functie derivabila
cu derivata continua, atunci functia implicita z(x, y) care este data
de ecuatia de mai sus verifica ecuatia:
z(x + z>(:)- y(y + z)(:J = 0
7. Fie functia z = <r( x, y ), unde y este functia implicita de variabila
x, data de ecuatia: '-V(x, y) = 0. Calculati : ( <p l?i '-V sunt functii
derivabile).
&p . 81j1 - 8\jl . &p
R dz = ax ay ax ay
dx 8lj1
ay
8. Sa se arate ca functia z(x,y), definita de ecuatia
F( x- az, y- bz) = 0, unde F este o functie ce ad mite derivate
. . 'f . ~ b ~
par(1ale cont1nue, sat1s ace ecuat1a: a ax + ay = I.
9. Fie z(x, y) definita de ecuatia y = x<p{z) + '-V(z); <p l?i '-V sunt
functii derivabile. Sa se arate ca functia z satisface ecuatia:
:: -(:r -2:.:. ~ + :: -r:r =0
10. Functiile y, z de variabila x sun! definite de sistemul de
ecuatii:
{
x' + y' - z' = 0
, (x
0
, Y
0
Z
0
) = {1,0,1}
x' + 2y' + 3z' = 4
. dy dz d'y d'z
Calculall -d , -d , -, , -
2

xxdxdx
196
.iJz
11. a) Calcula\1 ax ~
Z=CV, CEIR.
I
. i)z
b) Calcu at1 ax :
c) Calculati dz, c
az csim
R. a) ax = --u-
az 1
b) ax =2(v+L
1 [ u
c) dz=- e
2e'"
12. Functiile z l?i u de
{
X+ y + Z + U -1 = 0
x' + y' + z' + u' -1 =
Sa se calculeze deriv
u.
az x-u a
R. - = ~ ; -
ax u-z a
13. Sa se calculeze c
u v date de sistemul:
'
r xy- 2y
2
+ yu +X+ Y
t arctg( ) + in(u' + v':
In jurul punctului ( X
0
, l
derivabila
este data
variabila
functii
ecuatia
derivate
~ i ljl sunt
de
11. a) Calculati az coj az' daca X= ucos v, y = usinv,
ax"' ay
Z=CV, CEIR.
b)cl I
.az .az d _
a cu at1 ax ~ ~ ay , aca x = u + v, y = u- v, z = uv.
c) Calculati dz, daca x = e"v, y = e"-v, y = u. v.
R. a) az = csinv. az = ccos v
ax u'ay u
az 1 az 1
b) ax =2(v+u); ay =2(v-u)
1
c) dz=
2
e'" [e"-v(v+u)dx+e"v(v-u)dy]
12. Functiile z ~ i u de variabile x ~ i y sun! definite de:
{
X+Y+Z+U-1=0
x' + y
2
+ z' + u
2
- 1 = o
Sa se calculeze derivatele partiale de ordinul I pentru functiile z ~ i
u.
az x-u
R. ax--
u-z
az z-u au_z-x au=z-y
ay u-z ax u-z ay u-z
13. Sa se calculeze derivatele partiale de ordinul I pentru functiile
u, v date de sistemul:
J xy- 2y' + yu + x + y + v = 0
1 arctg( ) +In( u
2
+ v
2
) = 0
In jurul punctului ( X
0
, y
0
, U
0
, v
0
) = ( 0,1,1,0).
197
14. Sa se calculeze derivatele partiale de ordinul I, pentru
functiile u, v definite prin sistemul:

<J (v) y . daca {xo, Y0 ,U
0
, V0 ) = (uo. :).
ex sin y - J2 = 0
15 f
. . d . "b t a'u 2 a'u ( )
Trans ormat1 ecuat1a coar e1 VI ran e: at' =a ax' a> 0
tredlnd Ia noile variabile: a = x- at, j3 = x +at .
R. a'u = o
8a8j3
16
. . az az ( ) . t d d .
Transformat1 ecuat1a: y--x-= y-x z 1n ro ucan nolle
ax ay
2 2 1 1
variabile: u = x + y , v = - +-, precum !?i noua functie
X y
w=lnz-(x+y), unde w=w(u,v)
&w
R. -=0
av
17. Transformati
U = X+ y, V = X- y,
a'w 1
R. -=-
au' 2
a'z a'z a'z
ecuat.ia: - + 2 -- + -- = o
ax' axc;y c;y'
w=xy-z, unde w=w(u.v)
18. Sa se transforme ecuatia lui Laplace:
punand
a'u o'u a'u 0 d - 1 . b"l 8 t d t d
t.u = -
2
+-
2
+-
2
= , aca no1 e vana 1 e a, ,p sun a e e
ax c;y az
rela!iile:
198
l
x = pcos8sinq>
y=psin8sinq> (c
Z = pCOSq>
p>O, 8E(0,n), q>E
19. Sa se arate ca
( )
- {Y y E[O.I)
<py- ["
y
2
, y E 1,2_
multimea [ 1.4] x [ 0,2
20. Sa se arate ca
{
u=sin(x+y) ( ,
3 ' X,'
V=Y
(-:.:)x(-%%)
21. Se considera t
{
X= apCOS
2
8
; p?:
y = bpsin
2
8
polare generalizate
a) Sa se arate ca
b) Determinati mu
A= {(p,8)/ p E[0,1
c) Determinati F-
1

B = {(x,y)tFx+J
D(x,y)
R. a)-(-)=!
D 8,p
I, pentru
(a> o)
functie
pun and
date de
l
x = pcos e sin<p
y = psine sin<p (coordonate sferice),
Z = pCOS<p
p> 0, 8 E(O,rt), <p E(0,2rt).
19. Sa se arate ca transformarea u = <p(y), v = \jf(x), unde
( )
{
y, y E(0,1) ( ) {X', X E(1,2)
<Jl Y = , [
1
i \jf x = x [
1
, nu este regulata pe
y , y E 1,2 2 , X E 2,4
multimea (1,41 x (0,21, dar este injectiva pe aceasta multi me.
20. Sa se arate ca transformarea F definita de:
{
u = sin(x + y) ( )
2
, x, y E I R nu este regulata
v = y'
pe multi mea
(
nn)(nn)d t- - 1
- 4 '4 x - 4, 4 , ar es e lnjec!iva pe aceasta mu t1me.
21. Se considera transformarea F:
{
x = apcos' e n

2
; p 0, 0 e -, a > 0, b > 0, a > 0 ( coordonate
y = bps1n e 2
polare generalizate).
a) Sa sea rate ca F este regulata in punctul (1, ~ }
b) Determinati multimea F(A), unde
A = { (p, 8) I p E ( 0,11, 8 E ( 0, ;) } .
c) Determinati F"\B), unde
B = { ( x, y) I .JX + JY ~ a, x ~ 0, y ~ 0, a > 0} , cand a = b, a = 4.
D(x, y) ,
R. a) = aabpcos"- 8sin"-' e
o(e,p)
199
b)F(A) + y)t(::): + (::) (<I, x>. y> o]
c) F-'(B)={(p,e)tp E[O.I], e E[o.
{
I x,; 0
22. Sa se arate ca functiile f(x} = x', g(x} = ' definite pe
X, X> 0
IR i cu valori reale sunt independente in x = 0, dar nu sunt
independente in x ' o .
\lndicatie: \
Se folosete metoda reducerii Ia absurd.
. xy , x' + y' ' 0
23. Sa se arate ca functiile f(x, y) =

+ y
2
,
0, X=y=O
g(x, y) = x- y sunt independente in punctul (0,0).
24. Fie functiile f, g, h definite pe IR
3
, date prin:
f(x, y, z) = (x + y + z)',
g(x, y,z) = 2X+ y- 2z,
h(x, y,z) = 3x' -12xz -18zy.
Sa se arate ca:
a) f, g, h nu sunt independente in punctul (0,0,0).
b) Exista o vecinatate a punctului (-1,0,1) pe care g depinde de f
i h.
200
25. Studia\i dep1
z
f(x, y,z) = x
a) g(x, y,z) = t.
h(x, y,z) = z
f(x, y,z) = x
b)
g(x, y,z) = x
h(x, y,z) = x
c)

g(x, y,z)-.
h(x, y,z) = 2
26. Sa se aratE
\
f(x,y,z)=x+}
g(x, y,z) = x' +
h(x, y,z) = xy-
sunt in depe
dependenta fie
R. h depind
27. Sa se detE
u=f(x+y), Vo
R. f(x}=C
0
0
definite pe
dar nu sunt
;tO
0
depinde de f
25. Studia\i dependen\a sau independenta func\iilor:
z
t(x, y,z) =-
a)
b)
X
y-x'

X
YX
2
h(x, y,z) = z- xe"-
t(x, y,z) = x
, In punctele (1 ,0,0) (1, 1 ,0).
g( x, y, z) = x I - y , pe 0 = I R
3
\ { (X, y, z) I X - y = 0} .
x-y
h(x, y,z) =X l-z
x-y
c) g(x,y,z) = sinx-siny
l
f(x, y,z) = z- siny
h{x, y,z) = z- 2sinx + siny
26. Sa se arate ca functiile
l
f(x,y,z)=x+y-z
g(x,y,z)=x
2
+y
2
+Z
2
, definitepeiR
3
,
h(x, y, z) = xy - xz - yz
sunt in dependenta functionala pe IR
3
. Sa se cerceteze
dependenta fiecarei functii de celelalte doua.
R. h depinde de f g
27. Sa se determine functia f neconstanta, astfel ca functiile:
u =t(x + y), v = f(x)f(y) sa fie In dependenta.
R. f(x) = cex; a, C constante reale
201
28. Sa se cerceteze dependenta functiilor:
n
f{x" X
2
, , xn) =I xk
k=l
n
g(x" x,, .. . , x") =I xk,
k=l
n
h{x"x
2
, ,xn) = Ix,x
1
i,J=l
kj
R. g=f'-2h
202
Compartiment
Redactor: LID
Pocesare: SIM<
MIC
VIO
'
;
Compartimentul editorial-publicistic
Redactor: LUMINITA FALB
Pocesare: SIMONA U ~ O I
MIOARA GAMULEA
VIOLET A ROGOJAN
ISBN 973- 9462- 55- 3

S-ar putea să vă placă și